Bullets

HI there....welcome To test visual acuity, the nurse should ask the patient to cover each eye separately and read the eye chart with glasses and without, as appropriate. Fundamental of Nursing Before teaching any procedure to the patient, the nurse must first assess the patient’s willingness to learn and his current knowledge. Fundamentals of Nursing A blood pressure cuff that is too narrow can cause a falsely elevated blood pressure reading. Fundamentals of Nursing When preparing a single injection for a patient who takes regular and NPH insulin, the nurse should draw the regular insulin into the syringe first because it is clear and can be measured more accurately than the NPH insulin, which is turbid. Fundamentals of Nursing Rhonchi refers to the rumbling sounds heard on lung auscultation; they are more pronounced during expiration than during inspiration. Fundamentals of Nursing Gavage refers to forced feeding, usually through a gastric tube (a tube passed into the stomach by way of the mouth). Fundamentals of Nursing According to Maslow’s hierarchy of needs, physiologic needs (air, water, food, shelter, sex, activity, and comfort) have the highest priority. Fundamentals of Nursing Checking the identification band on a patient’s wrist is the safest and surest way to verify a patient’s identity. Fundamentals of Nursing A patient’s safety is the priority concern in developing a therapeutic environment. Fundamentals of Nursing The nurse should place the patient with a Sengstaken-Blakemore tube in semi-Fowler’s position. Fundamentals of Nursing The nurse can elicit Trousseau’s sign by occluding the brachial or radial artery; hand and finger spasms during occlusion indicate Trousseau’s sign and suggest hypocalcemia. Fundamentals of Nursing For blood transfusion in an adult, the appropriate needle size is 16 to 20G. Fundamentals of Nursing Pain that incapacities a patient and can’t be relieved by drugs is called intractable pain. Fundamentals of Nursing In an emergency, consent for treatment can be obtained by fax, telephone, or other telegraphic transmission. Fundamentals of Nursing Decibel is the unit of measurement of sound. Fundamentals of Nursing Informed consent is required for any invasive procedure. Fundamentals of Nursing A patient who can’t write his or her name to give consent for treatment must have his or her X witnessed by two persons, such as a nurse, priest, or doctor. Fundamentals of Nursing The Z-track I.M. injection technique seals medication deep into the muscle, thereby minimizing skin irritation and staining. It requires a needle that is 1’’ (2.5 cm) or longer. Fundamentals of Nursing A registered nurse (RN) should assign a licensed vocational nurse (LVN) or licensed practical nurse (LPN) to perform bedside care, such as suctioning and medication administration. Fundamentals of Nursing The therapeutic purposed of a mist tent is to increase hydration of secretions. Fundamentals of Nursing If a patient can’t void, the first nursing action should be bladder palpation to assess for bladder distention. Fundamentals of Nursing The patient who uses a cane should carry it on the unaffected side and advance it at the same time as the affected extremity. Fundamentals of Nursing To fit a supine patient for crutches, the nurse should measure from the axilla to the sole and add 2” (5 cm) to that measurement. Fundamentals of Nursing Assessment begins with the nurse’s first encounter with the patient and continues throughout the patient’s stay. The nurse obtains assessment data through the health history, physical examination, and review of diagnostic studies. Fundamentals of Nursing The appropriate needle size for an insulin injection is 25G and ⅝" (1.5 cm) long. Fundamentals of Nursing Residual urine refers to urine that remains in the bladder after voiding. The amount of residual urine normally ranges from 50 to 100 ml. Fundamentals of Nursing The five stages of the nursing process are assessment, nursing diagnosis, planning, implementation, and evaluation. Fundamentals of Nursing Planning refers to the stage of the nursing process in which the nurse assigns priorities to nursing diagnoses, defines short-term and long-term goals and expected outcomes, and establishes the nursing care plan. Fundamentals of Nursing Implementation refers to the stage of the nursing process in which the nurse puts the nursing care plan into action, delegates specific nursing interventions to members of the nursing team, and charts patient responses to nursing interventions. Fundamentals of Nursing Evaluation refers to the stage of the nursing process in which the nurse compares objective and subjective data with the outcome criteria and, if needed, modifies the nursing care plan, making the nursing process circular. Fundamentals of Nursing In the event of fire, the nurse should (1) remove the patient, (2) call the fire department, (3) attempt to contain the fire by closing the door, and (4) extinguish the fire, if it can be done safely. Fundamentals of Nursing Before administering any as need pain medication, the nurse should ask the patient to indicate the pain’s location. Fundamentals of Nursing Jehovah’s Witnesses believe that they shouldn’t receive blood components donated by other people. Fundamentals of Nursing When providing oral care for an unconscious patient, the nurse should position the patient on the side to minimize the risk of aspiration. Fundamentals of Nursing During assessment of distance vision, the patient should stand 20’ (6.1 m) from the chart. Fundamentals of Nursing The ideal room temperature for a geriatric patient or one who is extremely ill ranges form 66º to 76º F (18.8º to 24.4º C). Fundamentals of Nursing Normal room humidity ranges from 30% to 60%. Fundamentals of Nursing Hand washing is the single best method of limiting the spread of microorganisms. Hands should be washed for 10 seconds after routine contact with a patient and after gloves are removed. Fundamentals of Nursing To catheterize a female patient, the nurse should place her in the dorsal recumbent position. Fundamentals of Nursing A positive Homan’s sign may indicate thrombophlebitis. Fundamentals of Nursing Electrolytes in a solution are measured in milliequivalents per liter (mEq/L). A milliequivalent equals the number of milligrams per 100 milliliters of a solution. Fundamentals of Nursing Metabolism takes a place in two phases: anabolism (the constructive phase) and catabolism (the destructive phase). Fundamentals of Nursing The basal metabolic rate represents the amount of energy needed to maintain essential body functions. It is measured when the patient is awake and resting, hasn’t eaten for 14 to 18 hours, and is in a comfortable, warm environment. Fundamentals of Nursing Dietary fiber (roughage), which is derived from cellulose, supplies bulk, maintains adequate intestinal motility, and helps establish regular bowel habits. Fundamentals of Nursing Alcohol is metabolized primarily in the liver. Smaller amounts are metabolized by the kidneys and lungs. Fundamentals of Nursing Petechiae refers to tiny, round, purplish red spots that appear on the skin and mucous membranes as a result of intradermal or submucosal hemorrhage. Fundamentals of Nursing Purpura refers to a purple skin discoloration caused by blood extravasation. Fundamentals of Nursing Glucose-6-phosphate dehydrogenase (C6PD) deficiency is an inherited metabolic disorder characterized by red blood cells that are deficient in G6PD, a critical enzyme in aerobic glycolysis. Fundamentals of Nursing According to the standard precautions recommended by the Centers for Disease Control and Prevention, the nurse shouldn’t recap needles after use because most needle sticks result from missed needle recapping. Fundamentals of Nursing The nurse administers a drug by I.V. push by delivering the dose directly into a vein, I.V. tubing, or catheter with a needle and syringe. Fundamentals of Nursing When changing the ties on a tracheostomy tube, the nurse should leave the old ties in place until the new ones are applied. Fundamentals of Nursing A nurse should have assistance when changing the ties on a tracheostomy tube. Fundamentals of Nursing A filter is always used for blood transfusions. Fundamentals of Nursing A four-point (quad) cane is indicated when a patient needs more stability than a regular cane can provide. Fundamentals of Nursing The patient should carry a cane on the unaffected side to promote a reciprocal gait pattern and distribute weight away from the affected leg. Fundamentals of Nursing A good way to begin a patient interview is to ask “What made you seek medical help?” Fundamentals of Nursing The nurse should adhere to standard precautions for blood and body fluids when caring for all patients. Fundamentals of Nursing Potassium (K+) is the most abundant cation in intracellular fluid. Fundamentals of Nursing In the four-point gait (or alternating gait), the patient first moves the right crutch followed by the left foot and then the left crutch followed by the right foot. Fundamentals of Nursing In the three-point gait, the patient moves two crutches and the affected leg simultaneously and then moves the unaffected leg. Fundamentals of Nursing In the two-point gait, the patient moves the right leg and the left crutch simultaneously and then moves the left leg and the right crutch. Fundamentals of Nursing Vitamin B complex, the water-soluble vitamins essential for metabolism, include thiamine (B1), riboflavin (B2), niacin (B3), pyridoxine (B6), and cyanocobalamin (B12). Fundamentals of Nursing When being weighed, an adult patient should be lightly dressed and shoeless. Fundamentals of Nursing Before taking an adult’s oral temperature, the nurse should ensure that the patient hasn’t smoked or consumed hot or cold substances in the past 15 minutes. Fundamentals of Nursing The nurse shouldn’t take a rectal temperature on an adult patient if the patient has a cardiac disorder; anal lesions, or bleeding hemorrhoids or has recently undergone rectal surgery. Fundamentals of Nursing In a patient with cardiac problems, rectal temperature measurement may stimulate a vagal response, leading to vasodilation and decreased cardiac output. Fundamentals of Nursing When recording pulse amplitude and rhythm, the nurse should use these descriptive measures: +3 indicates a bounding pulse (readily palpable and forceful); +2, a normal pulse (easily palpable); +1, a thready or weak pulse (difficult to detect); and 0, an absent pulse (not detectable). Fundamentals of Nursing The intraoperative period begins when a patient is transferred to the operating room bed and ends when the patient is admitted to the postanesthesia recovery unit. Fundamentals of Nursing On the morning of surgery, the nurse should ensure that the informed consent form has been signed; that the patient hasn’t taken anything by mouth since midnight, has taken a shower with antimicrobial soap, has had mouth care (without swallowing the water, has removed common jewelry, and has received preoperative medication as prescribed; and that vital signs have taken and recorded. Artificial limbs and other prostheses are usually removed. Fundamentals of Nursing Comfort measures, such as positioning the patient, performing backrubs, and providing a restful environment, may decrease the patient’s need for analgesics or may enhance their effectiveness. Fundamentals of Nursing A drug has three names: its generic name, which is used in official publications; its trade name or brand name (such as Tylenol), which is selected by the drug company; and its chemical name, which describes the drug’s chemical composition. Fundamentals of Nursing The patient should take a liquid iron preparation through a straw to avoid staining the teeth. Fundamentals of Nursing The nurse should use the Z-track method to administer an I.M. injection of iron dextran (Imferon). Fundamentals of Nursing An organism may enter the body through the nose, mouth, rectum, urinary or reproductive tract, or skin. Fundamentals of Nursing In descending order, the levels of consciousness are alertness, lethargy, stupor, light coma, and deep coma. Fundamentals of Nursing To turn a patient by logrolling, the nurse folds the patient’s arms across the chest; extends the patient’s legs and inserts a pillow between them, if indicated; places a draw sheet under the patient; and turns the patient by slowly and gently pulling on the draw sheet. Fundamentals of Nursing The diaphragm of the stethoscope is used to hear high-pitched sounds such as breath sounds. Fundamentals of Nursing A slight blood pressure difference (5 to 10 mm Hg) between right and left arms is normal. Fundamentals of Nursing The nurse should place the blood pressure cuff 1'' (2.5 cm) above the antecubital fossa. Fundamentals of Nursing When instilling ophthalmic ointments, waste the first bed of ointment and then apply from the inner canthus to the outer canthus; twist the medication tube to detach the ointment. Fundamentals of Nursing The nurse should use a leg cuff to measure blood pressure in an obese patient. Fundamentals of Nursing If the blood pressure cuff is applied too loosely, the reading will be falsely elevated. Fundamentals of Nursing Ptosis refers to eyelid drooping. Fundamentals of Nursing A tilt table is useful for a patient with a spinal cord injury, orthostatic hypotension, or brain damage because it can move the patient gradually from a horizontal to a vertical (upright) position. Fundamentals of Nursing To perform venipuncture with the least injury to the vessel, the nurse should turn the bevel upward when the vessel’s lumen is larger than the needle and turn it downward when the lumen is only slightly larger than the needle. Fundamentals of Nursing To move the patient to the edge of the bed for transfer, follow these steps: (1) Move the patient’s head and shoulders toward the edge of the bed. (2) Move the patient’s feet and legs to the edge of the bed (crescent position). (3) Place both the arms well under the patient’s hips and straighten the back while moving the patient toward the edge of the bed. Fundamentals of Nursing When being measured for crutches, a patient should wear his or her shoes. Fundamentals of Nursing The nurse should attach a restraint to a part of the bed frame that moves with the head, not to the mattress or side rails. Fundamentals of Nursing The mist in a mist tent should never become so dense that it obscures clear visualization of the patient’s respiratory pattern. Fundamentals of Nursing To administer heparin subcutaneously, the nurse should follow these steps: (1) Clean, but don’t rub, the site with alcohol. (2) Stretch the skin taut or pick up a well-defined skin fold. (3)Hold the shaft of the needle in a dart position. (4)Insert the needle into the skin at a right (90-degree) angle. (5)Firmly depress the plunger; but don’t aspirate. (6)Leave the needle in place for 10 seconds. (7)Withdraw the needle gently at the same angle it was inserted. (8)Apply pressure to the injection site with an alcohol pad. Fundamentals of Nursing For a sigmoidoscopy, the nurse should place the patient in a knee-chest or Sims’ position, depending on the doctor’s preference. Fundamentals of Nursing Maslow’s hierarchy of needs must be met in the following order: physiologic (oxygen, food, water, sex, rest, and comfort) safety and security, love and belonging, self-esteem and recognition, and self-actualization. Fundamentals of Nursing When caring for patient with a nasogastric tube, the nurse should apply a water-soluble lubricant to the nostril to prevent soreness. Fundamentals of Nursing During gastric lavage, a nasogastric tube is inserted, the stomach is flushed, and ingested substances are removed through the tube. Fundamentals of Nursing In documenting drainage on a surgical dressing, the nurse should include the size, color, and consistency of the drainage, for example, “10 mm of brown mucoid drainage noted on dressing.” Fundamentals of Nursing To elicit Babinski’s reflex, the nurse strokes the sole of the patient’s foot with a moderately sharp object, such as thumbnail. Fundamentals of Nursing In a positive Babinski’s reflex, the great toe dorsiflexes and the other toes fan out. Fundamentals of Nursing When assessing a patient for bladder distention, the nurse should check the contour of the lower abdomen for a rounded mass above the symphysis pubis. Fundamentals of Nursing The best way to prevent pressure ulcers is to reposition the bedridden patient at least every 2 hours. Fundamentals of Nursing Antiembolism stockings decompress the superficial blood vessels, thereby reducing the risk of thrombus formation. Fundamentals of Nursing The most convenient veins for venipuncture in a adult patient are the basilic and median cubital veins in the antecubital space. Fundamentals of Nursing From 2 to 3 hours before beginning a tube feeding, the nurse should aspirate the patient’s stomach contents to verify adequate gastric emptying. Fundamentals of Nursing People with type O blood are considered to be universal donors. Fundamentals of Nursing People with type AB blood are considered to be universal recipients. Fundamentals of Nursing Herts (Hz) refers to the unit of measurement of sound frequency. Fundamentals of Nursing Hearing protection is required when the sound intensity exceeds 84 dB; double hearing protection is required if it exceeds 104 dB. Fundamentals of Nursing Prothrombin, a clotting factor, is produced in the liver. Fundamentals of Nursing If a patient is menstruating when a urine sample is collected, the nurse should note this on the laboratory slip. Fundamentals of Nursing During lumbar puncture, the nurse must note the initial intracranial pressure and the cerebrospinal fluid color. Fundamentals of Nursing A patient who can’t cough to provide a sputum sample for culture may require a heated aerosol treatment to facilitate removal of a sample. Fundamentals of Nursing If eye ointment and eyedrops must be instilled in the same eye, the eyedrops should be instilled first. Fundamentals of Nursing When leaving an isolation room, the nurse should remove the gloves before the mask because fewer pathogens are on the mask. Fundamentals of Nursing Skeletal traction is applied to a bone using wire pins or tons. It is the most effective means of traction. Fundamentals of Nursing The total parenteral nutrition solution should be stored in a refrigerator and removed 30 to 60 minutes before use because delivery of a chilled solution can cause pain, hypothermia, venous spasm, and venous constriction. Fundamentals of Nursing Medication isn’t routinely injected I.M. into edematous tissue because it may not be absorbed. Fundamentals of Nursing When caring for a comatose patient, the nurse should explain each action to the patient in a normal voice. Fundamentals of Nursing When cleaning dentures, the sink should be lined with a washcloth. Fundamentals of Nursing A patient should void within 8 hours after surgery. Fundamentals of Nursing An EEG identifies normal and abnormal brain waves. Fundamentals of Nursing Stool samples for ova and parasite tests should be delivered to the laboratory without delay or refrigeration. Fundamentals of Nursing The autonomic nervous system regulates the cardiovascular and respiratory systems. Fundamentals of Nursing When providing tracheostomy care, the nurse should insert the catheter gently into the tracheostomy tube. When withdrawing the catheter, the nurse should apply intermittent suction for no more than 15 seconds and use a slight twisting motion. Fundamentals of Nursing A low-residue diet includes such as foods as roasted chicken, rice, and pasta. Fundamentals of Nursing A rectal tube should not be inserted for longer than 20 minutes; it can irritate the mucosa of the rectum and cause a loss of sphincter control. Fundamentals of Nursing A patient’s bed bath should proceed in this order: face, neck, arms, hands, chest, abdomen, back, legs, perineum. Fundamentals of Nursing When lifting and moving a patient, the nurse should use the upper leg muscles most to prevent injury. Fundamentals of Nursing Patient preparation for cholecystography includes ingestion of a contrast medium and a low-fat evening meal. Fundamentals of Nursing During occupied bed changes, the patient should be covered with a black blanket to promote warmth and prevent exposure. Fundamentals of Nursing Anticipatory grief refers to mourning that occurs for an extended time when one realizes that death is inevitable. Fundamentals of Nursing The following foods can alter stool color: beets (red), cocoa (dark red or brown), licorice (black), spinach (green), and meat protein (dark brown). Fundamentals of Nursing When preparing a patient for a skull X-ray, have the patient remove all jewelry and dentures. Fundamentals of Nursing The fight-or-flight response is a sympathetic nervous system response. Fundamentals of Nursing Bronchovesicular breath sounds in peripheral lung fields are abnormal and suggest pneumonia. Fundamentals of Nursing Wheezing refers to an abnormal, high-pitched breath sound that is accentuated on expiration. Fundamentals of Nursing Wax or a foreign body in the ear should be gently flushed out by irrigation with warm saline solution. Fundamentals of Nursing If a patient complains that his hearing aid is “not working,” the nurse should check the switch first to see if it’s turned on and then check the batteries. Fundamentals of Nursing The nurse should grade hyperactive biceps and triceps reflexes +4. Fundamentals of Nursing If two eye medications are prescribed for twice-daily instillation, they should be administered 5 minutes apart. Fundamentals of Nursing In a postoperative patient, forcing fluids helps prevent constipation. Fundamentals of Nursing The nurse must administer care in accordance with standards of care established by the American Nurses Association, state regulations, and facility policy. Fundamentals of Nursing The kilocalorie (kcal) is a unit of energy measurement that represents the amount of heat needed to raise the temperature of 1 kilogram of water 1º C. Fundamentals of Nursing As nutrients move through the body, they undergo ingestion, digestion, absorption, transport, cell metabolism, and excretion. Fundamentals of Nursing The body metabolizes alcohol at a fixed rate regardless of serum concentration. Fundamentals of Nursing In an alcoholic beverage, its proof reflects its percentage of alcohol multiplied by 2. For example, a 100-proof beverage contains 50% alcohol. Fundamentals of Nursing A living will is a witnessed document that states a patient’s desire for certain types of care and treatment, which depends on the patient’s wishes and views and quality of life. Fundamentals of Nursing The nurse should flush a peripheral heparin lock every 8 hours (if it wasn’t used during the previous 8 hours) and as needed with normal saline solution to maintain patency. Fundamentals of Nursing Quality assurance is a method of determining whether nursing actions and practices meet established standards. Fundamentals of Nursing The five rights of medication administration are the right patient, right medication, right dose, right route of administration, and the right time. Fundamentals of Nursing Outside of the hospital setting, only the sublingual and transligual forms of nitroglycerin should be used to relieve acute anginal attacks. Fundamentals of Nursing The implementation phase of the nursing process involves recording the patient’s response to the nursing plan, putting the nursing plan into action, delegating specific nursing interventions, and coordinating the patient’s activities. Fundamentals of Nursing The Patient’s Bill of Rights offers guidance and protection to patients by stating the responsibilities of the hospital and its staff toward patients and their families during hospitalization. Fundamentals of Nursing To minimize the omissions and distortion of facts, the nurse should record information as soon as it is gathered. Fundamentals of Nursing When assessing a patient’s health history, the nurse should record the current illness chronologically, beginning with the onset of the problem and continuing to the present. Fundamentals of Nursing Drug administration is a dependent activity. The nurse can administer or withhold a drug only with the doctor’s permission. Fundamentals of Nursing The nurse shouldn’t give false assurance to a patient. Fundamentals of Nursing After receiving preoperative medication, a patient isn’t competent to sign an informed consent form. Fundamentals of Nursing When lifting a patient, a nurse uses the weight of her body instead of the strength in her arms. Fundamentals of Nursing A nurse may clarify a doctor’s explanation to a patient about an operation or a procedure but must refer questions about informed consent to the doctor. Fundamentals of Nursing The nurse shouldn’t use her thumb to take a patient’s pulse rate because the thumb has a pulse of its own and may be confused with the patient’s pulse. Fundamentals of Nursing An inspiration and an expiration count as one respiration. Fundamentals of Nursing Normal respirations are known as eupnea. Fundamentals of Nursing During a blood pressure measurement, the patient should rest the arm against a surface because using muscle strength to hold up the arm may raise the blood pressure. Fundamentals of Nursing Major unalterable risk factors for coronary artery disease include heredity, sex, race, and age. Fundamentals of Nursing Inspection is the most frequently used assessment technique. Fundamentals of Nursing Family members of an elderly person in a long-term care facility should transfer some personal items (such as photographs, a favorite chair, and knickknacks) to the person’s room to provide a homey atmosphere. Fundamentals of Nursing The upper respiratory tract warms and humidifies inspired air and plays a role in taste, smell, and mastication. Fundamentals of Nursing Signs of accessory muscle use include shoulder elevation, intercostal muscle retraction, and scalene and sternocleidosmastoid muscle use during respiration. Fundamentals of Nursing When patients use axillary crutches, their palms should bear the brunt of the weight. Fundamentals of Nursing Activities of daily living include eating, bathing, dressing, grooming, toileting, and interacting socially. Fundamentals of Nursing Normal gait has two phases: the stance phase, in which the patient’s foot rests on the ground, and the swing phase, in which that patient’s foot moves forward. Fundamentals of Nursing The phases of mitosis are prophase, metaphase, anaphase, and telophase. Fundamentals of Nursing The nurse should follow standard precautions in the routine care of all patients. Fundamentals of Nursing The nurse should use the bell of the stethoscope to listen for venous hums and cardiac murmurs. Fundamentals of Nursing The nurse can assess a patient’s general knowledge by asking questions such as “Who is the president for the United States?” Fundamentals of Nursing Cold packs are applied for the first 20 to 48 hours after an injury; then heat is applied. During cold application, the pack is applied for 20 minutes and then removed for 10 to 15 minutes to prevent reflex dilation (rebound phenomenon) and frostbite injury. Fundamentals of Nursing The pons is located above the medulla and consists of white matter (sensory and motor tracts) and gray matter (reflex centers). Fundamentals of Nursing The autonomic nervous system controls the smooth muscles. Fundamentals of Nursing A correctly written patient goal expresses the desired patient behavior, criteria for measurement, time frame for achievement, and conditions under which the behavior will occur. It is developed in collaboration with the patient. Fundamentals of Nursing The optic disk is yellowish pink and circular with a distinct border. Fundamentals of Nursing A primary disability results from a pathologic process; a secondary disability, from inactivity. Nurses usually are held liable for failing to keep an accurate count of sponges and other devices during surgery. Fundamentals of Nursing The best dietary sources of vitamin B6 are liver, kidney, pork, soybeans, corn, and whole-grain cereals. Fundamentals of Nursing Iron-rich foods, such as organ meats, nuts, legumes, dried fruit, leafy vegetables, eggs, and whole gains, generally have low water content. Fundamentals of Nursing Collaboration refers to joint communication and decision making between nurses and doctors designed to meet patients’ needs by integrating the care regimens of both professions in one comprehensive approach. Fundamentals of Nursing Bradycardia refers to a heart rate of fewer than 60 beats/minute. Fundamentals of Nursing A nursing diagnosis is a statement of a patient’s actual or potential health problems that can be resolved, diminished, or otherwise changed by nursing interventions. Fundamentals of Nursing During the assessment phase of the nursing process, the nurse collects and analyzes three types of data: health history, physical examination, and laboratory and diagnostic test data. Fundamentals of Nursing The patient’s health history consists primarily of subjective data, information supplied by patient. Fundamentals of Nursing The physical examination includes objective data obtained by inspection, palpation, percussion, and auscultation. Fundamentals of Nursing When documenting patient care, the nurse should write legibly, use only standard abbreviations, and sign every entry. The nurse should never destroy or attempt to obliterate documentation or leave vacant lines. Fundamentals of Nursing Factors that affect body temperature include time of day, age, physical activity, phase of menstrual cycle, and pregnancy. Fundamentals of Nursing The most accessible and commonly used artery for measuring a patient’s pulse rate is the radial artery, which is compressed against the radius to take the pulse rate. Fundamentals of Nursing The normal pulse rate of a resting adult is 60 to 100 beats/minute. The rate is slightly faster in women than in men and much faster in children than in adults. Fundamentals of Nursing Laboratory test results are an objective form of assessment data. Fundamentals of Nursing The measurement systems most often used in clinical practice are the metric system, apothecaries’ system, and household system. Fundamentals of Nursing Before signing an informed consent, a patient should know whether other treatment options are available and should understand what will occur during the preoperative, intraoperative, and postoperative phase; the risk involved; and the possible complications. The patient also should have a general idea of the time required from surgery to recovery and should have an opportunity to ask questions. Fundamentals of Nursing A patient must sign a separate informed consent form for each procedure. Fundamentals of Nursing During percussion, the nurse uses quick, sharp tapping of the fingers or hands against body surfaces to produce sounds (that helps determine the size, shape, position, and density of underlying organs and tissues), elicit tenderness, or assess reflexes. Fundamentals of Nursing Ballottement is a form of light palpation involving gentle, repetitive bouncing of tissues against the hand and feeling their rebound. Fundamentals of Nursing A foot cradle keeps bed linen off the patient’s feet, which prevent skin irritation and breakdown, especially in a patient with peripheral vascular disease or neuropathy. Fundamentals of Nursing If the patient is a married minor, permission to perform a procedure can be obtained form the patient’s spouse. Fundamentals of Nursing Gastric lavage is the flushing of the stomach and removal of ingested substances through a nasogastric tube. It can be used to treat poisoning or drug overdose. Fundamentals of Nursing During the evaluation step of the nursing process, the nurse assesses the patient’s response to therapy. Fundamentals of Nursing Bruits commonly indicate a life- or limb-threatening vascular disease. Fundamentals of Nursing O.U. means each eye; O.D., right eye; and O.S, left eye. Fundamentals of Nursing To remove a patient’s artificial eye, the nurse depresses the lower lid. Fundamentals of Nursing The nurse should use a warm saline solution to clean an artificial eye. Fundamentals of Nursing A thready pulse is very fine and scarcely perceptible. Fundamentals of Nursing Axillary temperature usually is 1º F lower than oral temperature. Fundamentals of Nursing After suctioning a tracheostomy tube, the nurse must document the color, amount, consistency, and odor of secretions. Fundamentals of Nursing On a medication prescription, the abbreviation p.c. means that the medication should be administered after meals. Fundamentals of Nursing After bladder irrigation, the nurse should document the amount, color, and clarity of the urine and the presence of clots or sediment. Fundamentals of Nursing Laws regarding patient self-determination vary from state to state. Therefore, the nurse must be familiar with the laws of the state in which she works. Fundamentals of Nursing Gauge refers to the inside diameter of a needle. The smaller the gauge, the larger the diameter. Fundamentals of Nursing An adult normally has 32 permanent teeth. Fundamentals of Nursing After turning a patient, the nurse should document the position used, time turned, and skin assessment findings. Fundamentals of Nursing PERRLA is an abbreviation for normal pupil assessment findings: pupils equal, round, and reactive to light with accommodation. Fundamentals of Nursing When purcussing a patient’s chest for postural drainage, the nurse’s hands should be cupped. Fundamentals of Nursing When measuring a patient’s pulse, the nurse should assess the rate, rhythms, quality, and strength. Fundamentals of Nursing Before transferring a patient from a bed to a wheelchair, the nurse should push the wheelchair’s footrests to the sides and lock its wheels. Fundamentals of Nursing When assessing respirations, the nurse should document the rate, rhythm, depth, and quality. Fundamentals of Nursing For a subcutaneous injection, the nurse should use a ⅝" 25G needle. Fundamentals of Nursing The notation “AA & O x 3” indicates that the patient is awake, alert, and oriented to person (knows who he is), place (knows where he is), and time (knows the date and time). Fundamentals of Nursing Fluid intake includes all fluids taken by mouth, including foods that are liquid at room temperature, such as gelatin, custard, and ice cream; I.V. fluids; and fluids administered in feeding tubes. Fluid output includes urine, vomitus, and drainage (such as from a nasogastric tube or from a wound) as well as blood loss, diarrhea or stool, and perspiration. Fundamentals of Nursing After administering an intradermal injection, the nurse shouldn’t massage the area because massage can irritate the site and interfere with results. Fundamentals of Nursing When administering an intradermal injection, the nurse should hold the syringe almost flat against the patient’s skin (at about a 15-degree angle) with the bevel up. Fundamentals of Nursing To obtain an accurate blood pressure, the nurse should inflate the manometer 20 to 30 mm Hg above the disappearance of the radial pulse before releasing the cuff pressure. Fundamentals of Nursing The nurse should count an irregular pulse for 1 full minute. Fundamentals of Nursing A patient who is vomiting while lying down should be placed in a lateral position to prevent aspiration of vomitus. Fundamentals of Nursing Prophylaxis is disease prevention. Fundamentals of Nursing Body alignment is achieved when the body parts are in proper relation to their natural position. Fundamentals of Nursing Trust is the foundation of a nurse-patient relationship. Fundamentals of Nursing Blood pressure in the force exerted by the circulating volume of blood on arterial walls. Fundamentals of Nursing Malpractice refers to the professional’s wrongful conduct, improper discharge of duties, or failure to meet standers of care, which causes harm to another. Fundamentals of Nursing As a general rule, nurses can’t refuse a patient care assignment; however, they may refuse to participate in abortions in most states. Fundamentals of Nursing A nurse can be found negligent if a patient is injured because the nurse failed to perform a duty that a reasonable and prudent person would perform or because the nurse performed an act that a reasonable and prudent person wouldn’t perform. Fundamentals of Nursing States have enacted Good Samaritan laws to encourage professionals to provide medical assistance at the scene of an accident without fear of a lawsuit arising from such assistance. These laws don’t apply to care provided in a health care facility. Fundamentals of Nursing A doctor should sign verbal and telephone orders within the time established by institutional policy, usually within 24 hours. Fundamentals of Nursing A competent adult has the right to refuse lifesaving medical treatment; however, the individual should be fully informed of the consequences of this refusal. Fundamentals of Nursing Although a patient’s health record or chart is the health care facility’s physical property, its contents belong to the patient. Fundamentals of Nursing Before a patient’s record can be released to a third party, the patient or patient’s legal guardian must give written consent. Fundamentals of Nursing Under the Controlled Substances Act, every dose of a controlled drug dispensed by the pharmacy must be counted for, whether the dose was administered to a particular patient or discarded accidentally. Fundamentals of Nursing A nurse can’t perform duties that violate a rule or regulation established by a state licensing board even if it is authorized by a health care facility or doctor. Fundamentals of Nursing The nurse should select a private room, preferably with a door that can be closed, to minimize interruptions during a patient interview. Fundamentals of Nursing In categorizing nursing diagnosis, the nurse should address actual life-threatening problems first, followed by potentially life-threatening concerns. Fundamentals of Nursing The major components of a nursing care plan are outcome criteria (patient goals) and nursing interventions. Fundamentals of Nursing Standing orders, or protocols, establish guidelines for treating a particular disease or set of symptoms. Fundamentals of Nursing In assessing a patient’s heart, the nurse normally finds the point of maximal impulse at the fifth intercostals space near the apex. Fundamentals of Nursing The S1 sound heard on auscultation is caused by closure of the mitral and tricuspid valves. Fundamentals of Nursing To maintain package sterility, the nurse should open the wrapper’s top flap away from the body, open side flap by touching only the outer part of the wrapper, and open the final flap by grasping the turned-down corner and pulling it toward the body. Fundamentals of Nursing The nurse shouldn’t use a cotton-tipped applicator to dry a patient’s ear canal or remove wax because it may force cerumen against the tympanic membrane. Fundamentals of Nursing A patient’s identification bracelet should remain in place until the patient has been discharged from the health care facility and has left the premises. Fundamentals of Nursing The Controlled Substances Act designated five categories, or schedules, that classify controlled drugs according to their abuse liability. Fundamentals of Nursing Schedule I drugs, such as heroin, have a high abuse potential and have no currently accepted medical use in the United States. Fundamentals of Nursing Schedule II drugs, such as morphine, opium, and meperidine (Demerol), have a high abuse potential but have currently accepted medical uses. Their use may lead to physical or psychological dependence. Fundamentals of Nursing Schedule III drugs, such as paregoric and butabarbital (Butisol), have a lower abuse potential than Schedule I or II drugs. Abuse of Schedule III drugs may lead to moderate or low physical or psychological dependence, or both. Fundamentals of Nursing Schedule IV drugs, such as chloral hydrate, have a low abuse potential compared with Schedule III drugs. Fundamentals of Nursing Schedule V drugs, such as cough syrups that contain codeine, have the lowest abuse potential of the controlled substances. Fundamentals of Nursing Activities of daily living are actions that the patient must perform every day to provide self-care and interact with society. Fundamentals of Nursing Testing of the six cardinal fields of gaze evaluates the function of all extraocular muscles and cranial nerves III, IV, and VI. Fundamentals of Nursing The six types of heart murmurs are graded from 1 to 6. A grade 6 heart murmur can be heard with stethoscope slightly raised from the chest. Fundamentals of Nursing The most important goal to include in a care plan is the patient’s goal. Fundamentals of Nursing Fruits are high in fiber and low in protein and should be omitted from a low-residue diet. Fundamentals of Nursing The nurse should use an objective scale to assess and quantify pain because postoperative pain varies greatly among individuals. Fundamentals of Nursing Postmortem care includes cleaning and preparing the deceased patient for family viewing, arranging transportation to the morgue or funeral home, and determining the disposition of belongings. Fundamentals of Nursing The nurse should provide honest answers to the patient’s questions. Fundamentals of Nursing Milk shouldn’t be included in a clear liquid diet. Fundamentals of Nursing Consistency in nursing personnel is paramount when caring for a child, and infant, or a confused patient. Fundamentals of Nursing The hypothalamus secretes vasopressin and oxytocin, which are stored in the pituitary gland. Fundamentals of Nursing The three membranes that enclose that brain and spinal cord are the dura mater, pia mater, and arachnoid. Fundamentals of Nursing A nasogastric tube is used to remove fluid and gas from the small intestine preoperatively or postoperatively. Fundamentals of Nursing Psychologists, physical therapists, and chiropractors aren’t authorized to write prescriptions for medication. Fundamentals of Nursing The area around a stoma should be cleaned with mild soap and water. Fundamentals of Nursing Vegetables have a high fiber content. Fundamentals of Nursing The nurse should use a tuberculin syringe to administer an S.C. injection of less than 1 ml. Fundamentals of Nursing For adults, S.C. injections require a 25G 1" needle; for infants, children, elderly, or very thin patients, they require a 25G to 27G ½" needle. Fundamentals of Nursing Before administering medication, the nurse should identify the patient by checking the identification band and asking the patient to state his name. Fundamentals of Nursing To clean the skin before an injection, the nurse should use a sterile alcohol swab and wipe from the center of the site outward in a circular motion. Fundamentals of Nursing The nurse always should inject heparin deep into S.C. tissue at a 90-degree angle (perpendicular to the skin) to prevent skin irritation. Fundamentals of Nursing If blood is aspirated into the syringe before an I.M. injection, the nurse should withdraw the needle, prepare another syringe, and repeat the procedure. Fundamentals of Nursing The nurse shouldn’t cut the patient’s hair without written consent from the patient or an appropriate relative. Fundamentals of Nursing If bleeding occurs after an injection, the nurse should apply pressure until the bleeding stops; if bruising occurs, the nurse should monitor the site for an enlarging hematoma. Fundamentals of Nursing When providing hair and scalp care, the nurse should begin combing at the end of the hair and work toward the head. Fundamentals of Nursing Frequency of patient hair care depends on the length and texture of the hair, duration of hospitalization, and patient’s condition. Fundamentals of Nursing Proper hearing aid function requires careful handling during insertion and removal, regular cleaning of the ear piece to prevent wax buildup, and prompt replacement of dead batteries. Fundamentals of Nursing The hearing aid marked with a blue dot is for the left ear; the one with the red dot is for the right ear. Fundamentals of Nursing A hearing aid shouldn’t be exposed to heat or humidity and shouldn’t be immersed in water. Fundamentals of Nursing The nurse should instruct a patient not to use hair spray while wearing a hearing aid. Fundamentals of Nursing The five branches of pharmacology are pharmacokinetics, pharmacodynamics, pharmacotherapeutics, toxicology, and pharmacognosy. Fundamentals of Nursing The nurse should remove heel protectors every 8 hours to inspect the foot for signs of skin breakdown. Fundamentals of Nursing The purpose of heat application is to promote vasodilation, which reduces pain caused by inflammation. Fundamentals of Nursing A sutured surgical incision is an example of healing by first intention (healing directly, without granulation). Fundamentals of Nursing Healing by secondary intention (healing by granulation) is closure of the wound by the granulation tissue filling the defect and allowing reepithelialization to occur, beginning at the wound edges and continuing to the center, until the entire wound is covered. Fundamentals of Nursing Keloid formation is an abnormality in healing characterized by overgrowth of scar tissue at the wound site. Fundamentals of Nursing The nurse should administer procaine penicillin by deep I.M. injection in the upper outer portion of the buttocks in the adult or in the midlateral thigh in the child. The nurse shouldn’t massage the injection site. Fundamentals of Nursing The ascending colostomy drains fluid feces; the descending colostomy drains solid fecal matter. Fundamentals of Nursing A folded towel (called a scrotal bridge) can provide scrotal support for the patient with scrotal edema caused by vasectomy, epididymitis, or orchitis. Fundamentals of Nursing When giving an injection to the patient with a bleeding disorder, the nurse should use a small-gauge needle and apply pressure to the site for 5 minutes after the injection. Fundamentals of Nursing Platelets are the smallest and most fragile formed element of the blood and are essential for coagulation. Fundamentals of Nursing To insert a nasogastric tube, the nurse should first instruct the patient to tilt the head back slightly and then insert the tube. When the tube is felt curving at the pharynx, the nurse should tell the patient to tilt the head forward to close the trachea and open the esophagus by swallowing. (Sips of water can facilitate this action.) Fundamentals of Nursing According to families whose loved ones are in intensive care units, their four most important needs are to have questions answered honestly, to be assured that the best possible car is being provided, to know the prognosis, and to feel there is hope. Fundamentals of Nursing A double-bind communication when the verbal message contradicts the nonverbal message and the receiver is unsure of which message to respond to. Fundamentals of Nursing A nonjudgmental attitude displayed by the nurse demonstrates that she neither approves nor disapproves of the patient. Fundamentals of Nursing Target symptoms are those that the patient and others find most distressing. Fundamentals of Nursing Advise the patient to take aspirin on an empty stomach with a full glass of water and to avoid foods with acid such as coffee, citrus fruits, and cola. Fundamentals of Nursing For every patient problem, there is a nursing diagnosis; for every nursing diagnosis, there is a goal; and for every goal, there are interventions designed to make the goal a reality. The keys to answering examination questions correctly are identifying the problem presented, formulating a goal for that specific problem, and then selecting the intervention from the choices provided that will enable the patient to reach that goal. Fundamentals of Nursing Fidelity means loyalty and can be shown as a commitment to the profession of nursing and to the patient. Fundamentals of Nursing Giving an I.M. injection against the patient’s will and without legal authority is battery. Fundamentals of Nursing An example of a third-party payor is an insurance company. Fundamentals of Nursing On-call medication should be given within 5 minutes of receipt of the call. Fundamentals of Nursing Generally, the best method to determine the cultural or spiritual needs of the patient is to ask him. Fundamentals of Nursing An incident report shouldn’t be made part of the patient’s record but is an in-house document for the purpose of correcting the problem. Fundamentals of Nursing Critical pathways are a multidisciplinary guideline for patient care. Fundamentals of Nursing When prioritizing nursing diagnoses, use this hierarchy: (1) problems associated with airway, (2) those concerning breathing, and (3) those related to circulation. Fundamentals of Nursing The two nursing diagnoses with the highest priority that the nurse can assign are Ineffective airway clearance and Ineffective breathing pattern. Fundamentals of Nursing A subjective sign that a sitz bath has been effective is that patient expresses a decrease in pain or discomfort. Fundamentals of Nursing For the nursing diagnosis Diversional activity deficit to be valid, the patient must make the statement that he’s “bored, there is nothing to do” or words to that effect. Fundamentals of Nursing The most appropriate nursing diagnosis for an individual who doesn’t speak English is Communication, impaired, related to inability to speak dominant language (English). Fundamentals of Nursing The family of the patient who has been diagnosed as hearing impaired should be instructed to face the individual when they speak to him. Fundamentals of Nursing Up to age 3, the pinna should be pulled down and back to straighten the eustachian tube before instilling medication. Fundamentals of Nursing When administering eyedrops, the nurse should waste the first drop and instill the medication in the lower conjunctival sac to prevent injury to the cornea. Fundamentals of Nursing When administering eye ointment, the nurse should waste the first bead of medication and then apply the medication from the inner to the outer canthus. Fundamentals of Nursing When removing gloves and mask, the gloves, which most likely contain pathogens and are soiled, should be removed first. Fundamentals of Nursing Crutches should placed 6" (15 cm) in front of the patient and 6" to the side to assume a tripod position. Fundamentals of Nursing Listening is the most effective communication technique. Fundamentals of Nursing Process recording is a method of evaluating one’s communication effectiveness. Fundamentals of Nursing When feeding the elderly, limit high-carbohydrate foods because of the risk of glucose intolerance. Fundamentals of Nursing Passive range of motion maintains joint mobility whereas resistive exercises increase muscle mass. Fundamentals of Nursing Isometric exercises are performed on an extremity in a cast. Fundamentals of Nursing A back rub is an example of the gate-control theory of pain. Fundamentals of Nursing Anything below the waist is considered unsterile, a sterile field becomes unsterile when it comes in contact with nay unsterile item, a sterile field must be continuously monitored, and the 1" (2.5 cm) border around a sterile field is considered unsterile. Fundamentals of Nursing A “shift to the left” is evident when there is an increase in immature cells (bands) in the blood to fight an infection. Fundamentals of Nursing A “shift to the right” is evident when there is an increase in mature cells in the blood as seen in advanced liver diseases and pernicious anemia. Fundamentals of Nursing Before administering preoperative medication, make sure that an informed consent form has been signed and attached to the patient’s record. Fundamentals of Nursing The nurse should spend no more than 30 minutes per 8-hour shift in providing care to the patient with a radiation implant. Fundamentals of Nursing The nurse should stand near the patient’s shoulders for cervical implants and at the foot of the bed for head and neck implants. Fundamentals of Nursing The nurse should never be assigned to care for more than one patient with radiation implants. Fundamentals of Nursing Long-handled forceps and a lead-lined container should be in the room of the patient who has a radiation implant. Fundamentals of Nursing Generally, patients who have the same infection and are in strict isolation can share the same room. Fundamentals of Nursing Diseases requiring strict isolation include chickenpox, diphtheria, and viral hemorrhagic fever such as Marburg virus disease. Fundamentals of Nursing For the patient abiding by Jewish custom, milk and meat shouldn’t be served in the same meal. Fundamentals of Nursing Whether the patient can perform a procedure (psychomotor domain of learning) is a better indicator of the effectiveness of patient teaching than whether the patient can simply state the steps of the procedure (cognitive domain of learning). Fundamentals of Nursing Developmental stages according to Erik Erikson are trust versus mistrust (birth to 18 months), autonomy versus shame and doubt (18 months to 3 years), initiative versus guilt (3 to 5 years), industry versus inferiority (5 to 12 years), identity versus identity diffusion (12 to 18 years), intimacy versus isolation (18 to 25 years), generativity versus stagnation (25 to 60 years), and ego integrity versus despair (older than 60 years). Fundamentals of Nursing Face the hearing impaired patient when communicating with him. Fundamentals of Nursing A proper nursing intervention for the spouse of the patient who has suffered a serious incapacitating disease is to assist him in mobilizing a support system. Fundamentals of Nursing Hyperpyrexia refers to extreme elevation in temperature above 106º F (41.1º C). Fundamentals of Nursing

Basic Drills Answers

1. According to Maslow, which of the following categories of needs represents the most basic?

A. Physiologic needs- Physiologic needs must be met before an individual is able to move toward psychological health and well-being.

B. Self-actualization-Self-actualization is the highest level of need

C. Safety and security needs-Safety and security needs, while lower level, are not essential to physiologic survival.

D. Belongingness- Belongingness and affection needs are not essential to physiologic survival.

2. Which of the following statements reflects the World Health Organization’s definition of health?

A. A state of complete physical, mental, and social well-being and not merely the absence of disease and infirmity.- Such a definition, however, does not allow for any variations in the degrees of wellness or illness.

B. A condition of homeostatis and adaptation.- The WHO definition addresses physical, mental, and social dimensions of being.

C. An individual’s location along a wellness--illness continuum.- The concept of a health--illness continuum allows for a greater range in describing a person’s health than the definition provided by the WHO.

D. A fluid, ever-changing balance reflected through physical, mental, and social behavior.- The WHO definition does not allow for any variations in the degrees of wellness and illness.

3. Which of the following statements defines culture?

A. The learned patterns of behavior, beliefs, and values that can be attributed to a particular group of people.- Included among characteristics that distinguish cultural groups are manner of dress, values, artifacts, and health beliefs and practices.

B. A group of people distinguished by genetically transmitted material.-A group of people distinguished by genetically transmitted material describes the term race.

C. The status of belonging to a particular region by origin, birth, or naturalization.-The status of belonging to a particular region by origin, birth, or naturalization describes the term nationality.

D. The classification of a group based upon certain distinctive characteristics.-The classification of a group based upon certain distinctive characteristics describes the term ethnicity.

4.The reason that case management has gained such prominence in health care can be traced to

A. decreased cost of care associated with inpatient stay.- The reasons case management has gained such prominence can be traced to the decreased cost of care associated with decreased length of hospital stay, coupled with rapid and frequent inter-unit transfers from specialty to standard care units.

B. increased length of hospital stay.- In general, length of hospital stay has decreased over the past 5 years.

C. discharge from specialty care units to home.- In general, patients are transferred from specialty care units to standard care units at least 24 hours prior to discharge.

D. limited availability for inter-unit hospital transfers.- In general, patients in acute care hospitals undergo frequent inter-unit transfers from specialty to standard care units.

5.A preferred provider organization is described as a

A. business arrangement between hospitals and physicians.- PPO’s usually contract to provide health care to subscribers, usually businesses, for a negotiated fee that often is discounted.

B. prepaid group health practice system.- A prepaid group health practice system is termed a health maintenance organization.

C. limited insurance program.- Insurance is a cost payment system of shared risk, not a health care delivery system.

D. health care savings account program.- A health care savings account program is an incentive program to consumers, not a health care delivery system.

6.Which of the following categories identifies the focus of community/public health nursing practice?

A. Promoting and maintaining the health of populations and preventing and minimizing the progress of disease
- lthough nursing interventions used by public health nurses might involve individuals, families, or small groups, the central focus remains promoting health and preventing disease in the entire community.

B. Rehabilitation and restorative services- Rehabilitation and restorative services are the focus of extended care facilities and home care nursing.

C. Adaptation of hospital care to the home environment.- Adaptation of hospital care to the home environment is the focus of home nursing.

D. Hospice care delivery. -Hospice care delivery refers to the delivery of services to the terminally ill.

7.A major goal for home care nurses is

A. restoring maximum health function.- Tertiary preventive nursing care, focusing on rehabilitation and restoring maximum health function, is a goal for home care nurses.

B. promoting the health of populations.-Promoting the health of populations is a focus of community/public health nursing.

C. minimizing the progress of disease.- Minimizing the progress of disease is a focus of community/public health nursing.

D. maintaining the health of populations.
Maintaining the health of populations is a focus of community/public health nursing.

8.In the United States, nurses performing invasive procedures need to be up-to-date with their immunizations, particularly

A. hepatitis B. -Hepatitis B is transmitted through contact with infected blood or plasma.

B. hepatitis E.

Hepatitis E is found mainly in underdeveloped countries with substandard sanitation and water quality.


C. hepatitis A. - Hepatitis A is transmitted through the oral route from the feces and saliva of an infected person.


D. hepatitis C.- At present, immunization against hepatitis C is not available.

9.At what time during a patient’s hospital stay does discharge planning begin?

A. Admission-To prepare for early discharge and the possible need for follow-up in

the home, discharge planning begins with the patient’s admission.


B. Twenty-four hours prior to discharge- Discharge planning requires identification of patient needs and anticipatory guidance and is not relegated to a specific time for beginning.

C. The shift prior to discharge- Discharge planning requires communication with an cooperation of the patient, family, and health care team and is not relegated to a specific time for beginning

D. By the third hospital day- Discharge planning may require involvement of personnel and agencies in the planning process and is not relegated to a specific day of hospital stay.

10.The leading health problems of elementary school children include

A. cancer.- The leading health problems of elementary school children are injuries, infections, malnutrition, dental disease, and cancer.


B. alcohol and drug abuse. - Alcohol and drug abuse are leading health problems for high school students.

C.. mental and emotional problems.- Mental and emotional problems are leading health problems for high school students.

D .homicide. - Homicide is a leading health problem for high school children.

11. Which skill needed by the nurse to think critically involves identification of patient problems indicated by data?

A. Analysis-
Analysis is used to identify patient problems indicated by data.

B. Interpretation- Interpretation is used to determine the significance of data that is gathered.

C. Inferencing- Inferences are used by the nurse to draw conclusions.

D. Explanation- Explanation is the justification of actions or interventions used to address patient problems and to help a patient move toward desired outcomes.

12.The ethics theory that focuses on ends or consequences of actions is the

A. utilitarian theory.- Utilitarian theory is based on the concept of the greatest good for the greatest number.

B. formalist theory.- Formalist theory argues that moral standards exist independently of the ends or consequences.

C. deontological theory.- Deontological theory argues that moral standards exist independently of the ends or consequences.

D. adaptation theory.- Adaptation theory is not an ethics theory.

13.Which of the following ethical principles refers to the duty to do good?

A. Beneficence-
Beneficence is the duty to do good and the active promotion of benevolent acts.

B. Fidelity- Fidelity refers to the duty to be faithful to one's commitments.

C. . Veracity- Veracity is the obligation to tell the truth.

d. Nonmaleficence- Nonmaleficence is the duty not to inflict, as well as to prevent and remove, harm; it is more binding than beneficence.

14.During which step of the nursing process does the nurse analyze data related to the patient's health status?

A. Assessment-
Analysis of data is included as part of the assessment.

B. Implementation. -Implementation is the actualization of the plan of care through nursing interventions.

C. Diagnosis.- Diagnosis is the identification of patient problems.

D. Evaluation. -Evaluation is the determination of the patient's responses to the nursing interventions and the extent to which the outcomes have been achieved.

15.The basic difference between nursing diagnoses and collaborative problems is that

A.. nurses manage collaborative problems using physician-prescribed interventions.- Collaborative problems are physiologic complications that nurses monitor to detect onset or changes and manage through the use of physician-prescribed and nursing-prescribed interventions to minimize the complications of events.

B.. collaborative problems can be managed by independent nursing interventions.- Collaborative problems require both nursing and physician-prescribed interventions.

C. nursing diagnoses incorporate physician-prescribed interventions.-Nursing diagnoses can be managed by independent nursing interventions.

D. nursing diagnoses incorporate physiologic complications that nurses monitor to detect change in status.- Nursing diagnoses refer to actual or potential health problems that can be managed by independent nursing interventions.

16.Health education of the patient by the nurse

A. is an independent function of nursing practice.- Health education is an independent function of nursing practice and is included in all state nurse practice acts.

B. requires a physician's order.- Teaching, as a function of nursing, is included in all state nurse practice acts.

C. must be approved by the physician.- Health education is a primary responsibility of the nursing profession.

D. must focus on wellness issues.- Health education by the nurse focuses on promoting, maintaining, and restoring health; preventing illness; and assisting people to adapt to the residual effects of illness.

17.Nonadherence to therapeutic regimens is a significant problem for which of the following age groups?

A. Adults 65 and over- Elderly people frequently have one or more chronic illnesses that are managed with numerous medications and complicated by periodic acute episodes, making adherence difficult.

B. Teenagers- Problems of teenagers, generally, are time limited and specific, and require promoting adherence to treatment to return to health.

C. Children- In general, the compliance of children depends on the compliance of their parents.

D. Middle-aged adults- Middle-aged adults, in general, have fewer health problems, thus promoting adherence.

18.Experiential readiness to learn refers to the patient's

A. past history with education and life experience.- Experiential readiness refers to past experiences that influence a person's ability to learn.

B. emotional status.-Emotional readiness refers to the patient's acceptance of an existing illness or the threat of an illness and its influence on the ability to learn.

C. acceptance of an existing illness.- Emotional readiness refers to the patient's acceptance of an existing illness or the threat of an illness and its influence on the ability to learn.

D. ability to focus attention.- Physical readiness refers to the patient's ability to cope with physical problems and focus attention upon learning.

19.Asking the patient questions to determine if the person understands the health teaching provided would be included during which step of the nursing process?

A. Evaluation-
Evaluation includes observing the person, asking questions, and comparing the patient's behavioral responses with the expected outcomes.

B. Assessment- Assessment includes determining the patient's readiness regarding learning.

C. Planning and goals- Planning includes identification of teaching strategies and writing the teaching plan.

D. Implementation- Implementation is the step during which the teaching plan is put into action.

20.Which of the following items is considered the single most important factor in assisting the health professional in arriving at a diagnosis or determining the person's needs?

A. History of present illness- The history of the present illness is the single most important factor in assisting the health professional in arriving at a diagnosis or determining the person's needs.

B. Physical examination- The physical examination is helpful but often only validates the information obtained from the history.

C. Diagnostic test results- Diagnostic test results can be helpful, but they often only verify rather than establish the diagnosis.

D. Biographical data- Biographical information puts the health history in context but does not focus the diagnosis.

21. Of the following areas for assessing the patient profile, which should be addressed after the others?

A. Body image- The patient is often less anxious when the interview progresses from information that is less personal to information that is more personal.

B. Education- Educational level is relatively impersonal and readily revealed by the patient.

C. Occupation- Occupation is relatively impersonal and readily revealed by the patient.

D. Environment- Housing, religion, and language are relatively impersonal and readily revealed by the patient.

22.Which of the following methods of physical examination refers to the translation of physical force into sound?

A. Percussion- Percussion translates the application of physical force into sound.

B. Palpation- Palpation refers to examination by non-forceful touching.

C. Auscultation- Auscultation refers to the skill of listening to sounds produced within the body created by movement of air or fluid.

D. Manipulation- Manipulation refers to the use of the hands to determine motion of a body part.

23.In which range of body mass index (BMI) are patients considered to have increased risk for problems associated with poor nutritional status?

A. Below 24- Additionally, higher mortality rates in hospitalized patients and community-dwelling elderly are associated with individuals who have low BMI.

B. 25-29- Those who have a BMI of 25 to 29 are considered overweight.

C. 30 to 34- Those who have BMI of 30-39 are considered obese.

D. Over 40- Those who have BMI over 40 are considered extremely obese.

24.To calculate the ideal body weight for a woman, the nurse allows

A. 100 pounds for 5 feet of height.- To calculate the ideal body weight of a woman, the nurse allows 100 pounds for 5 feet of height and adds 5 pounds for each additional inch over 5 feet

B. 106 pounds for 5 feet of height.- The nurse allows 106 pounds for 5 feet of height in calculating the ideal body weight for a man.

C. 6 pounds for each additional inch over 5 feet.- The nurse adds 6 pounds for each additional inch over 5 feet in calculating the ideal body weight for a man.

D. 80 pounds for 5 feet of height.- Eighty pounds for 5 feet of height is too little.

25.A steady state within the body is termed

A. homeostasis.- When a change occurs that causes a body function to deviate from its stable range, processes are initiated to restore and maintain the steady state or homeostasis.

B. constancy.- Constancy refers to the balanced internal state of the human body maintained by physiologic and biochemical processes.

C. adaptation.- Adaptation refers to a constant, ongoing process that requires change in structure, function, or behavior so that the person is better suited to the environment.

D. stress.- Stress refers to a state produced by a change in the environment that is perceived as challenging, threatening, or damaging to the person's dynamic balance or equilibrium.

26.Which of the following terms, according to Lazarus, refers to the process through which an event is evaluated with respect to what is at stake and what might and can be done?

A. Cognitive appraisal-
The outcome of cognitive appraisal is identification of the situation as either stressful or non-stressful.

B. Coping- Coping consists of both cognitive and behavioral efforts made to manage the specific external or internal demand that taxes a person's resources.

C. Hardiness- Hardiness is a personality characteristic that is composed of control, commitment, and challenge.

D. Adaptation- Lazarus believed adaptation was affected by emotion that subsumed stress and coping.

27.An increase in the number of new cells in an organ or tissue that is reversible when the stimulus for production of new cells is removed is termed

A. hyperplasia.-
Hyperplasia occurs as cells multiply and are subjected to increased stimulation resulting in tissue mass enlargement.

B. hypertrophy.- Hypertrophy is an increase in size and bulk of tissue that does not result from an increased number of cells.

C. atrophy.- Atrophy refers to reduction in size of a structure after having come to full maturity.

D. neoplasia.- With neoplasia, the increase in the number of new cells in an organ or tissue continues after the stimulus is removed.

28.Which of the following types of cells have a latent ability to regenerate?

A. Stable- Stable cells have a latent ability to regenerate if they are damaged or destroyed and are found in the kidney, liver, and pancreas, among other body organs.

B. Labile- Labile cells multiply constantly to replace cells worn out by normal physiologic processes.

C. Permanent- Permanent cells include neurons --- the nerve cell bodies, not their axons. Destruction of a neuron causes permanent loss, but axons may regenerate.

D. Epithelial- Epithelial cells are a type of labile cell that multiply constantly to replace cells worn out by normal physiologic processes.

29.The relaxation techniques of progressive muscle relaxation, relaxation with guided imagery, and the Benson Relaxation Response share which of the following elements?

A. A mental device (something on which to focus the attention)-Similar elements also include a quiet environment, a comfortable position, and a passive attitude.

B. Nutritional foundation- Relaxation techniques do not encompass specific nutritional guidelines.

C. Analgesic preparation- Relaxation techniques are used to reduce one's response to stress and do not require analgesia prior to practicing the techniques.

D. Physician's order- A physician's order is not required to assist an individual to learn techniques to reduce one's response to stress.

30.Which of the following terms has been defined by the American Psychiatric Association as a group of behavioral or psychological symptoms or a pattern that manifests itself in significant distress, impaired functioning, or accentuated risk of enduring severe suffering or possible death?

A. Mental disorder- The definition was adopted by the American Psychiatric Association in 1994.

B. Emotional disorder- There is no universally accepted definition of what constitutes an emotional disorder.

C. Anxiety- Anxiety is defined as fear of the unknown.

D. Schizophrenia- Schizophrenia is a specific disorder characterized by psychosis.

31.Establishing financial security has been identified as a developmental task of which of the following groups?

A. Middle adult- The middle adult's tasks also include launching children, and refocusing on one's marital relationship.

B. Older adult- The older adult's tasks include adapting to retirement and declining physical stamina.

C. Young adult- The young adult's tasks include establishing a lifestyle and independence.

D. Teenager- The teenager's primary developmental tasks include developing an identity and intimacy.


32.When up to a 6-month period elapses between the experience of trauma and the onset of symptoms of posttraumatic stress disorder (PTSD), the episode is termed

A.delayed.-
In the case of delayed PTSD, there may be up to a 6-month period of time that elapses between the trauma and the manifestation of symptoms.

B. acute.- Acute PTSD is defined as the experience of symptoms for less than a 3-month period.

C. chronic.- Chronic PTSD is defined as the experience of symptoms lasting longer than 3 months.

D. primary.-The concept of primary disease is not used in relation to PTSD.

33.Which of the following statements accurately describes a risk factor for depression?

A. History of physical or sexual abuse- History of physical or sexual abuse and current substance abuse are risk factors for depression.

B. Male gender- A risk factor for depression is female gender.

C. Age over 50 years- A risk factor for depression is onset before 40 years.

D. Negative family history of depression- Family history of depression is a risk factor.

34. Of the following stages of grieving as described by Kubler-Ross, which is the initial?

A. Denial- The stages include: denial, anger, bargaining, depression, and acceptance.

B. Anger- Anger is the second stage of the process.

C. Bargaining- Bargaining is the third stage of the process.

D. Depression- Depression is the fourth stage of the process.

35.Which of the following terms refers to Leininger's description of the learned and transmitted knowledge about values, beliefs, rules of behavior, and lifestyle practices that guide a designated group in their thinking and actions in patterned ways?

A. Culture- Leininger was the founder of the specialty called transcultural nursing and advocated culturally competent nursing care.

B. Minority- Minority refers to a group of people whose physical or cultural characteristics differ from the majority of people in a society.

C. Race- Race refers to a group of people distinguished by genetically transmitted characteristics.

D. Subculture- Subculture refers to a group that functions within a culture.

36.The inability of a person to recognize his or her own values, beliefs, and practices and those of others because of strong ethnocentric tendencies is termed

A. cultural blindness.- Cultural blindness results in bias and stereotyping.

B. acculturation.- Acculturation is the process by which members of a culture adapt or learn how to take on the behaviors of another group.

C. cultural imposition.- Cultural imposition is the tendency to impose one's cultural beliefs, values, and patterns of behavior on a person from a different culture.

D. cultural taboo.- Cultural taboos are those activities governed by rules of behavior that are avoided, forbidden, or prohibited by a particular cultural group.


37. Which of the following groups of individuals may stare at the floor during conversations as a sign of respect?

A. Native Americans- Some Native Americans stare at the floor during conversations, conveying respect and indicating that the listener is paying close attention to the speaker.

B. Indo-Chinese- The Indo-Chinese may consider direct eye contact impolite or aggressive.

C. Arabs- Arabs may consider direct eye contact impolite or aggressive.

D. Asians- Asians may consider direct eye contact impolite or aggressive.

38.For which of the following religious groups is all meat prohibited?

A. Hinduism- Hinduism prohibits consumption of all meats and animal shortening.

B. Seventh-Day Adventism- Seventh-Day Adventism prohibits consumption of pork.

C. Judaism- Judaism prohibits consumption of pork.

D. Islam- Islam prohibits the consumption of pork and animal shortening.

39.The paradigm that explains the cause of illness as an imbalance in the forces of nature is the

A. holistic perspective.- The naturalist or holistic perspective believes that health exists when all aspects of a persona are in perfect balance or harmony.

B. magico-religious view.- The magico-religious view holds that illness is caused by forces of evil.

C. biomedical view.-The biomedical view holds life events as cause and effect and incorporates the bacterial or viral explanation of communicable disease.

D. scientific view.- The scientific view holds life events as cause and effect and incorporates the bacterial or viral explanation of communicable disease.

40.The aim of genomic medicine is

A. improving predictions about individuals’ susceptibility to diseases-
Predictions regarding the time of their onset, their extent and eventual severity as well as which treatments or medications are likely to be most effective or harmful are the focus of genomic medicine.

B. reproduction- The focus of genomic medicine is broader than the reproduction of cells.

C. cure of disease- The focus of genomic medicine is broader than the cure of disease.

D. cloning- Genomic medicine is gene-based health care.

41.Nondisjunction of a chromosome results in which of the following diagnoses?

A. Down Syndrome-
When a pair of chromosomes fails to separate completely and creates a sperm or oocyte that contains two copies of a particular chromosome (nondisjunction) Down syndrome results from three number 21 chromosomes.

B. Huntingon Disease- Huntington disease is one example of a germ-line mutation.

C. Duchenne Muscular Dystrophy- Duchenne muscular dystrophy, an inherited form of muscular dystrophy, is an example of a genetic caused by structural gene mutations.

D. Marphan Syndrome- Marphan Syndrome is a genetic condition that may occur in a single family member as a result of spontaneous mutation.

42.Which type of Mendelian inherited condition results in both genders being affected equally in a vertical pattern?

A. Automosomal dominant inheritance- An individual who has an autosomal dominant inherited condition carries a gene mutation for that condition on one chromosome of a pair.

B. Automosomal recessive inheritance- The pattern of inheritance in autosomal recessive inherited conditions is different from that of autosomal dominant inherited conditions in that it is more horizontal than vertical, with relatives of a single generation tending to have the condition.

C. X-linked inheritance- X-linked conditions may be inherited in families in recessive or dominant patterns. In both, the gene mutation is located on the X-chromosome. All males inherit an X chromosome from their mother with no counterpart; hence, all males express the gene mutation.

D. Multifactorial genetic inheritance- Neural tube defects, such as spina bifida and anencephaly, are examples of multifactorial genetic conditions. The majority of neural tube defects are caused by both genetic and environmental influences that combine during early embryonic development leading to incomplete closure of the neural tube.


43.A specific BRCA1 cancer-predisposing gene mutation seems to occur more frequently among women of which descent?

A. Ashkanazi Jewish
- Expression of the BRCA1 gene is an example of inheritance in the development of breast cancer.

B. Mediterranean- Glucose-6-phosphate dehydrogenase deficiency (G6PD) is a common enzyme abnormality that affects millions of people throughout the world, especially those of Mediterranean, South East Asian, African, Middle Eastern, and Near Eastern origin.

C. African American- Sickle cell anemia is associated with the African-American population.

D. Chinese and Japanese- Individuals of Chinese and Japanese descent who are rapid metabolizers of the enzyme N-acetyltransferase and who are prescribed the drug isoniazid (as part of treatment for tuberculosis) are at significantly increased risk for developing isoniazid-induced hepatitis.

44.Which of the following statements describes accurate information related to chronic illness?

A. Most people with chronic conditions do not consider themselves sick or ill.
Although some people take on a sick role identity, most people with chronic conditions do not consider themselves sick or ill and try to live as normal a life as is possible.

B. Most people with chronic conditions take on a sick role identity.-Research has demonstrated that some people with chronic conditions may take on a sick role identity, but they are not the majority.

C. Chronic conditions do not result from injury. -Chronic conditions may be due to illness, genetic factors, or injury

D. Most chronic conditions are easily controlled.- Many chronic conditions require therapeutic regimens to keep them under control.

45.In which phase of the trajectory model of chronic illness are the symptoms under control and managed?

A. Stable-
The stable phase indicates that the symptoms and disability are under control or managed.

B. Acute- The acute phase is characterized by sudden onset of severe or unrelieved symptoms or complications that may necessitate hospitalization for their management.

C. Comeback- The comeback phase is the period in the trajectory marked by recovery after an acute period.

D. Downward- The downward phase occurs when symptoms worsen or the disability progresses despite attempts to control the course through proper management.

46.Which phase of the trajectory model of chronic illness is characterized by reactivation of the illness?

A. Unstable- The unstable phase is characterized by development of complications or reactivation of the illness.

B. Stable- The stable phase indicates that the symptoms and disability are under control or managed.

C. Acute- The acute phase is characterized by sudden onset of severe or unrelieved symptoms or complications that may necessitate hospitalization for their management.

D. Comeback- The comeback phase is the period in the trajectory marked by recovery after an acute period.

47.Which phase of the trajectory model of chronic illness is characterized by the gradual or rapid decline in the trajectory despite efforts to halt the disorder?

A. Dying- The dying phase is characterized by stoppage of life-maintaining functions.

B. Unstable- The unstable phase is characterized by development of complications or reactivation of the illness.

C. Acute- The acute phase is characterized by sudden onset of severe or unrelieved symptoms or complications that may necessitate hospitalization for their management.

D. Downward- The downward phase occurs when symptoms worsen or the disability progresses despite attempts to control the course through proper management.

48.In order to help prevent the development of an external rotation deformity of the hip in a patient who must remain in bed for any period of time, the most appropriate nursing action would be to use

A. a trochanter roll extending from the crest of the ilium to the midthigh.-A trochanter roll, properly placed, provides resistance to the external rotation of the hip.

B. pillows under the lower legs.- Pillows under the legs will not prevent the hips from rotating externally.

C. a hip-abductor pillow.- A hip-abductor pillow is used for the patient after total hip replacement surgery.

D. a footboard.- A footboard will not prevent the hips from rotating externally.

49.To prevent footdrop, the patient is positioned in:

A. Order to keep the feet at right angles to the leg- When the patient is supine in bed, padded splints or protective boots are used.

B. A semi-sitting position in bed- Semi-fowlers positioning is used to decrease the pressure of abdominal contents on the diaphragm.

C. A sitting position with legs hanging off the side of the bed- In order to prevent footdrop, the feet must be supported.

D. side-lying position- Side-lying positions do not provide support to prevent footdrop.

50.Through which of the following activities does the patient learn to consciously contract excretory sphincters and control voiding cues?

A. Biofeedback- Cognitively intact patients who have stress or urge incontinence may gain bladder control through biofeedback.

B. Kegel exercises- Kegel exercises are pelvic floor exercises that strengthen the pubococcygeus muscle.

C. Habit training- Habit training is used to try to keep the patient dry by strictly adhering to a toileting schedule and may be successful with stress, urge, or functional incontinence.

D. Bladder training- Habit training is a type of bladder training.

51.During which stage of pressure ulcer development does the ulcer extend into the subcutaneous tissue?

A. Stage III- Clinically, a deep crater with or without undermining of adjacent tissues is noted.

B. Stage IV- A stage IV pressure ulcer extends into the underlying structure, including the muscle and possibly the bone.

C. Stage II- A stage II ulcer exhibits a break in the skin through the epidermis or dermis.

D. Stage I- A stage I pressure ulcer is an area of nonblanchable erythema, tissue swelling, and congestion, and the patient complains of discomfort.

52.During which stage of pressure ulcer development does the ulcer extend into the underlying structures, including the muscle and possibly the bone?

A. Stage IV- A stage IV pressure ulcer extends into the underlying structure, including the muscle and possibly the bone.

B. Stage III- A stage III ulcer extends into the subcutaneous tissue.

C. Stage II- A stage II ulcer exhibits a break in the skin through the epidermis or dermis.

D. Stage I- A stage I pressure ulcer is an area of nonblanchable erythema, tissue swelling, and congestion, and the patient complains of discomfort.

53.Which type of incontinence is associated with weakened perineal muscles that permit leakage of urine when intra-abdominal pressure is increased?

A. Stress incontinence
- Stress incontinence may occur with coughing or sneezing.

B. Urge incontinence- Urge incontinence is involuntary elimination of urine associated with a strong perceived need to void.

C. Reflex (neurogenic) incontinence- Neurogenic incontinence is associated with a spinal cord lesion.

D. Functional incontinence- Functional incontinence refers to incontinence in patients with intact urinary physiology who experience mobility impairment, environmental barriers, or cognitive problems.

54.Ageism refers to

A. Bias against older people based solely on chronological age.-Individuals demonstrating ageism base their beliefs and attitudes about older people based upon chronological age without consideration of functional capacity.

B. fear of old age.- Fear of aging and the inability of many to confront their own aging process may trigger ageist beliefs.

C. loss of memory.- Age-related loss of memory occurs more with short-term and recent memory.

D. benign senescent forgetfulness.- Benign senescent forgetfulness refers to the age-related loss of memory in the absence of a pathologic process.

55. When assessing the older adult, the nurse anticipates increase in which of the follow components of respiratory status?

A. Residual lung volume- As a result, patient experience fatigue and breathlessness with sustained activity.

B. Vital capacity- The nurse anticipates decreased vital capacity.

C. Gas exchange and diffusing capacity- The nurse anticipates decreased gas exchange and diffusing capacity resulting in impaired healing of tissues due to decreased oxygenation.

D. Cough efficiency- The nurse anticipates difficulty coughing up secretions due to decreased cough efficiency.

56.According to the classification of hypertension diagnosed in the older adult, hypertension that can be attributed to an underlying cause is termed

A. secondary.- Secondary hypertension may be caused by a tumor of the adrenal gland (e.g., pheochromacytoma).

B. primary.- Primary hypertension has no known underlying cause.

C. essential.- Essential hypertension has no known underlying cause.

D. isolated systolic.- Isolated systolic hypertension is demonstrated by readings in which the systolic pressure exceeds 140 mm Hg and the diastolic measurement is normal or near normal (less than 90 mm Hg).

57. Which of the following terms refers to the decrease in lens flexibility that occurs with age, resulting in the near point of focus getting farther away?

A. Presbyopia
- Presbyopia usually begins in the fifth decade of life, when reading glasses are required to magnify objects.

B. Presbycusis- Presbycusis refers to age-related hearing loss.

C. Cataract- Cataract is the development of opacity of the lens of the eye.

D. Glaucoma- Glaucoma is a disease characterized by increased intraocular pressure.

58.Which of the following states is characterized by a decline in intellectual functioning?

A. Dementia- Dementia is an acquired syndrome in which progressive deterioration in global intellectual abilities is of such severity that it interferes with the person's customary occupational and social performance.

B. Depression- Depression is a mood disorder that disrupts quality of life.

C. Delirium- Delirium is often called acute confusional state.

D. Delusion- Delusion is a symptom of psychoses.

59.When a person who has been taking opioids becomes less sensitive to their analgesic properties, that person is said to have developed a (an)

A. tolerance.-
Tolerance is characterized by the need for increasing dose requirements to maintain the same level of pain relief.

B. addiction.- Addiction refers to a behavioral pattern of substance use characterized by a compulsion to take the drug primarily to experience its psychic effects.

C. dependence.- Dependence occurs when a patient who has been taking opioids experiences a withdrawal syndrome when the opioids are discontinued.

D. balanced analgesia.- Balanced analgesia occurs when the patient is using more than one form of analgesia concurrently to obtain more pain relief with fewer side effects.

60.Prostaglandins are chemical substances thought to

A. increase sensitivity of pain receptors.- Prostaglandins are believed to increase sensitivity to pain receptors by enhancing the pain-provoking effect of bradykinin.

B. reduce the perception of pain.- Endorphins and enkephalins reduce or inhibit transmission or perception of pain.

C. inhibit the transmission of pain.- Endorphins and enkephalins reduce or inhibit transmission or perception of pain.

D. inhibit the transmission of noxious stimuli.-Morphine and other opioid medications inhibit the transmission of noxious stimuli by mimicking enkephalin and endorphin.

61.Which of the following principles or guidelines accurately informs the nurse regarding placebos?

A. Placebos should never be used to test the person's truthfulness about pain.-Perception of pain is highly individualized.

B. A placebo effect is an indication that the person does not have pain.-A placebo effect is a true physiologic response.

C. A placebo should be used as the first line of treatment for the patient.-A placebo should never be used as a first line of treatment.

D. A positive response to a placebo indicates that the person's pain is not real.-
Reduction in pain as a response to placebo should never be interpreted as an indication that the person's pain is not real.

62.Regarding tolerance and addiction, the nurse understands that

A. although patients may need increasing levels of opioids, they are not addicted.
Physical tolerance usually occurs in the absence of addiction.

B. tolerance to opioids is uncommon.- Tolerance to opioids is common.

C. addiction to opioids commonly develops.- Addiction to opioids is rare.

D. the nurse must be primarily concerned about development of addiction by the patient in pain.-Addiction is rare and should never be the primary concern for a patient in pain.

63.The preferred route of administration of medication in the most acute care situations is which of the following routes?

A. Intravenous- The IV route is the preferred parenteral route in most acute care situations because it is much more comfortable for the patient, and peak serum levels and pain relief occur more rapidly and reliably.

B. Epidural- Epidural administration is used to control postoperative and chronic pain.

C. Subcutaneous- Subcutaneous administration results in slow absorption of medication.

D. Intramuscular- Intramuscular administration of medication is absorbed more slowly than intravenously administered medication.

64.Mu opioids have which of the following effects on respiratory rate:

A. Stimulation, then depression- Mu opioids also cause bradycardia, hypothermia, and constipation.

B. No change- Kappa opioids result in no change in respiratory rate.

C. Stimulation, only- Delta opioids result in stimulation of respiratory rate.

D. Depression, only- Neither mu, nor kappa, nor delta opoids depress respiratory rate as its only effect upon respiratory rate.

65.Which of the following electrolytes is a major cation in body fluid?

A. Potassium- Potassium is a major cation that affects cardiac muscle functioning.

B. Chloride- Chloride is an anion.

C. Bicarbonate- Bicarbonate is an anion.

D. Phosphate- Phosphate is an anion.

66.Which of the following electrolytes is a major anion in body fluid?

A. Chloride- Chloride is a major anion found in extracellular fluid.

B. Potassium-Potassium is a cation.

C. Sodium- Sodium is a cation.

D. Calcium- Calcium is a cation.

67. Oncotic pressure refers to

A. the osmotic pressure exerted by proteins. -Oncotic pressure is a pulling pressure exerted by proteins, such as albumin.

B. the number of dissolved particles contained in a unit of fluid.
Osmolality refers to the number of dissolved particles contained in a unit of fluid.

C. the excretion of substances such as glucose through increased urine output.
Osmotic diuresis occurs when the urine output increases due to excretion of substances such as glucose.

D. the amount of pressure needed to stop flow of water by osmosis.
Osmotic pressure is the amount of pressure needed to stop the flow of water by osmosis.

68.Which of the following solutions is hypotonic?

A. 0.45% NaCl.- Half-strength saline is hypotonic

B. Lactated Ringer's solution.- Lactated Ringer's is isotonic.

C. 0.9% NaCl.- Normal saline (0.9% NaCl) is isotonic.

D. 5% NaCl.- A solution that is 5% NaCl is hypertonic.

69.The normal serum value for potassium is

A. 3.5-5.5 mEq/L.- Serum potassium must be within normal limits to prevent cardiac dysrhythmias.

B. 135-145 mEq/L.-Normal serum sodium is 135-145 mEq/L.

C. 96-106 mEq/L.- Normal serum chloride is 96-106 mEq/L.

D. 8.5-10.5 mg/dL.- Normal total serum calcium is 8.5-10.5mg/dL.

70. In which type of shock does the patient experiences a mismatch of blood flow to the cells?

A. Distributive
- Distributive or vasogenic shock results from displacement of blood volume, creating a relative hypovolemia.

B. Cardiogenic- Cardiogenic shock results from the failure of a heart as a pump.

C. Hypovolemic- In hypovolemic shock, there is a decrease in the intravascular volume.

D. Septic- In septic shock, overwhelming infection results in a relative hypovolemia.

71.Which stage of shock is best described as that stage when the mechanisms that regulate blood pressure fail to sustain a systolic pressure above 90 mm Hg?

A. Progressive- In the progressive stage of shock, the mechanisms that regulate blood pressure can no longer compensate, and the mean arterial pressure falls below normal limits.

B. Refractory- The refractory or irreversible stage of shock represents the point at which organ damage is so severe that the patient does not respond to treatment and cannot survive.

C. Compensatory- In the compensatory state, the patient's blood pressure remains within normal limits due to vasoconstriction, increased heart rate, and increased contractility of the heart.

D. Irreversible- The refractory or irreversible stage of shock represents the point at which organ damage is so severe that the patient does not respond to treatment and cannot survive.

72. When the nurse observes that the patient's systolic blood pressure is less than 80--90 mm Hg, respirations are rapid and shallow, heart rate is over 150 beats per minute, and urine output is less than 30 cc per hour, the nurse recognizes that the patient is demonstrating which stage of shock?

A. Compensatory- In compensatory shock, the patient's blood pressure is normal, respirations are above 20, and heart rate is above 100 but below 150.

B. Progressive- In progressive shock, the patient's skin appears mottled and mentation demonstrates lethargy.

C. Refractory- In refractory or irreversible shock, the patient requires complete mechanical and pharmacologic support.

D. Irreversible- In refractory or irreversible shock, the patient requires complete mechanical and pharmacologic support.


73. Which of the following vasoactive drugs used in treating shock results in reduced preload and afterload, reducing oxygen demand of the heart?

A. Nitroprusside (Nipride)- A disadvantage of nitroprusside is that it causes hypotension.

B. Dopamine (Intropin)- Dopamine improves contractility, increases stroke volume, and increases cardiac output.

C. Epinephrine (adrenaline)- Epinephrine improves contractility, increases stroke volume, and increases cardiac output.

D. Methoxamine (Vasoxyl)- Methoxamine increases blood pressure by vasoconstriction.

74.The nurse anticipates that the immunosuppressed patient is at greatest risk for which type of shock?

A. Septic- Septic shock is associated with immunosuppression, extremes of age, malnourishment, chronic illness, and invasive procedures.

B. Neurogenic- Neurogenic shock is associated with spinal cord injury and anesthesia.

C. Cardiogenic- Cardiogenic shock is associated with disease of the heart.

D. Anaphylactic- Anaphylactic shock is associated with hypersensitivity reactions.

75.Which of the following colloids is expensive but rapidly expands plasma volume?

A.
Albumin- Albumin is a colloid that requires human donors, is limited in supply, and can cause congestive heart failure.

B. Dextran- Dextran is a colloid, synthetic plasma expander that interferes with platelet aggregation and is not recommended for hemorrhagic shock.

C. Lactated Ringers- Lactated ringers is a crystalloid, not a colloid.

D. Hypertonic Saline- Hypertonic saline is a crystalloid, not a colloid.

76. Which of the following terms refers to cells that lack normal cellular characteristics and differ in shape and organization with respect to their cells of origin?

A. Anaplasia-
Usually, anaplastic cells are malignant.

B. Neoplasia- Neoplasia refers to uncontrolled cell growth that follows no physiologic demand.

C. Dysplasia- Dysplasia refers to bizarre cell growth resulting in cells that differ in size, shape, or arrangement from other cells of the same type of tissue.

D. Hyperplasia- Hyperplasia refers to an increase in the number of cells of a tissue, most often associated with a period of rapid body growth.

77.Palliation refers to

A. relief of symptoms associated with cancer.-Palliation is the goal for care in terminal cancer patients.

B. hair loss.- Alopecia is the term that refers to hair loss.

C. the spread of cancer cells from the primary tumor to distant sites.
Metastasis is the term that refers to the spread of cancer cells from the primary tumor to distant sites.

D. the lowest point of white blood cell depression after therapy that has toxic effects on the bone marrow.-
Nadir is the term that refers to the lowest point of white blood cell depression after therapy that has toxic effects on the bone marrow.

78. During which step of cellular carcinogenesis do cellular changes exhibit increased malignant behavior?

A. Progression- During this third step, cells show a propensity to invade adjacent tissues and metastasize.

B. Promotion- During promotion, repeated exposure to promoting agents causes the expression of abnormal genetic information even after long latency periods.

C. Initiation- During this first step, initiators such as chemicals, physical factors, and biologic agents escape normal enzymatic mechanisms and alter the genetic structure of cellular DNA.

D. Prolongation- No stage of cellular carcinogenesis is termed prolongation.

79.The drug, Interleukin-2, is an example of which type of biologic response modifier?

A. Cytokine- Other cytokines include interferon alfa and filgrastim.

B. Monoclonal antibodies- Monoclonal antibodies include rituximab, trastuzumab, and gemtuzumab.

C. Retinoids- Retinoic acid is an example of a retinoid.

D. Antimetabolites- Antimetabolites are cell cycle-specific antineoplastic agents.

80.Of the following terms, which is used to refer to the period of time during which mourning a loss takes place?

A. Bereavement- Bereavement is the period of time during which mourning a loss takes place.

B. Grief- Grief is the personal feelings that accompany an anticipated or actual loss

C. Mourning- Mourning is the individual, family, group and cultural expressions of grief and associated behaviors

D. Hospice- Hospice is a coordinated program of interdisciplinary care and services provided primarily in the home to terminally ill patients and their families.

81. Which of the following "awareness contexts" is characterized by the patient, the family, and the health care professionals being aware that the patient is dying but all pretend otherwise?

A. Mutual pretense awareness- In mutual pretense awareness, the patient, the family and the health care professionals are aware that the patient is dying but all pretend otherwise.

B. Closed awareness- In closed awareness, the patient is unaware of his terminality in a context where others are aware.

C. Suspected awareness- In suspected awareness, the patient suspects what others know and attempts to find it out.

D. Open awareness- In open awareness, all are aware that the patient is dying and are able to openly acknowledge that reality.


82.For individuals known to be dying by virtue of age and/or diagnoses, which of the following signs indicate approaching death:

A. Increased restlessness- As the oxygen supply to the brain decreases, the patient may become restless.

B. Increased wakefulness
As the body weakens, the patient will sleep more and begin to detach from the environment.

C. Increased eating- For many patients, refusal of food is an indication that they are ready to die.

D. Increased urinary output
Based upon decreased intake, urinary output generally decreases in amount and frequency.

83. Which of the following terms best describes a living will?

A. Medical directive- The living will is a type of advance medical directive in which the individual of sound mind documents treatment preferences.

B. Proxy directive- A proxy directive is the appointment and authorization of another individual to make medical decisions on behalf of the person who created an advance directive when he/she is no loner able to speak for him/herself.

C. Health care power of attorney- Health care power of attorney is a legal document that enables the signer to designate another individual to make health care decisions on his/her behalf when he/she is unable to do so.

D. Durable power of attorney for health- A durable power of attorney for health care is a legal document that enables the signer to designate another individual to make health care decisions on his/her behalf when he/she is unable to do so.

84.A malignant tumor

A. gains access to the blood and lymphatic channels.- By this mechanism, the tumor metastasizes to other areas of the body.

B. demonstrates cells that are well-differentiated.- Cells of malignant tumors are undifferentiated.

C. is usually slow growing.- Malignant tumors demonstrate variable rate of growth; however, the more anaplastic the tumor, the faster its growth.

D. grows by expansion.- A malignant tumor grows at the periphery and sends out processes that infiltrate and destroy surrounding tissues.

85.Which of the following classes of antineoplastic agents is cell--cycle-specific?

A. Antimetabolites (5-FU)- Antimetabolites are cell--cycle-specific (S phase).

B. Antitumor antibiotics (bleomycin)- Antitumor antibiotics are cell-cycle nonspecific.

C. Alkylating agents (cisplatin)- Alkylating agents are cell-cycle nonspecific.

D. Nitrosureas (carmustine)- Nitrosureas are cell-cycle nonspecific.

86. Regarding the surgical patient, which of the following terms refers to the period of time that constitutes the surgical experience?

A. Perioperative phase- Perioperative period includes the preoperative, intraoperative, and postoperative phases.

B. Preoperative phase- Preoperative phase is the period of time from when the decision for surgical intervention is made to when the patient is transferred to the operating room table.

C. Intraoperative phase- Intraoperataive phase is the period of time from when the patient is transferred to the operating room table to when he or she is admitted to the postanesthesia care unit.

D. Postoperative phase- Postoperative phase is the period of time that begins with the admission of the patient to the postanesthesia care unit and ends after a follow-up evaluation in the clinical setting or home.

87.When the indication for surgery is without delay, the nurse recognizes that the surgery will be classified as

A. emergency.-Emergency surgery means that the patient requires immediate attention and the disorder may be life-threatening.

B. urgent.- Urgent surgery means that the patient requires prompt attention within 24-30 hours.

C. required.- Required surgery means that the patient needs to have surgery, and it should be planned within a few weeks or months.

D. elective.- Elective surgery means that there is an indication for surgery, but failure to have surgery will not be catastrophic.

88. When a person with a history of chronic alcoholism is admitted to the hospital for surgery, the nurse anticipates that the patient may show signs of alcohol withdrawal delirium during which time period?

A. Up to 72 hours after alcohol withdrawal- Alcohol withdrawal delirium is associated with a significant mortality rate when it occurs postoperatively.

B. Immediately upon admission- Onset of symptoms depends upon time of last consumption of alcohol.

C. Upon awakening in the post-anesthesia care unit- Onset of symptoms depends upon time of last consumption of alcohol.

D. Up to 24 hours after alcohol withdrawal-Twenty-four hours is too short a time frame to consider alcohol withdrawal delirium no longer a threat to a chronic alcoholic.


89. Which of the following categories of medications may result in seizure activity if withdrawn suddenly?

A. Tranquilizers- Abrupt withdrawal of tranquilizers may result in anxiety, tension, and even seizures if withdrawn suddenly.

B. Adrenal corticosteroids- Abrupt withdrawal of steroids may precipitate cardiovascular collapse.

C. Antidepressants- Monoamine oxidase inhibitors increase the hypotensive effects of anesthetics.

D. Diuretics- Thiazide diuretics may cause excessive respiratory depression during anesthesia due to an associated electrolyte imbalance.

90. When the patient is encouraged to concentrate on a pleasant experience or restful scene, the cognitive coping strategy being employed by the nurse is

A. imagery.- Imagery has proven effective for oncology patients.

B. optimistic self-recitation.- Optimistic self-recitation is practiced when the patient is encouraged to recite optimistic thoughts such as “I know all will go well.”

C. distraction.- Distraction is employed when the patient is encouraged to think of an enjoyable story or recite a favorite poem.

D. progressive muscular relaxation.- Progressive muscular relaxation requires contracting and relaxing muscle groups and is a physical coping strategy as opposed to cognitive.

91. According to the American Society of Anesthesiology Physical Status Classification System, a patient with severe systemic disease that is not incapacitating is noted to have physical status classification

A.
P3- Classification P3 patients are those who have compensated heart failure, cirrhosis, or poorly controlled diabetes, for example.

B. P4- Classification P4 patients have an incapacitating systemic disease that is a constant threat to life.

C. P1- Classification P1 refers to a normal healthy patient

D. P2- Classification P2 reflects a patient with mild systemic disease

92.Which stage of anesthesia is termed surgical anesthesia?

A. III- With proper administration of the anesthetic, this stage may be maintained for hours.

B. I- Stage I is beginning anesthesia, as the patient breathes in the anesthetic mixture and experiences warmth, dizziness, and a feeling of detachment.

C. II- Stage II is the excitement stage, which may be characterized by struggling, singing, laughing, or crying.

D. IV- Stage IV is a stage of medullary depression and is reached when too much anesthesia has been administered.

93.Fentanyl (Sublimaze) is categorized as which type of intravenous anesthetic agent?

A. Neuroleptanalgesic-
Fentanyl is 75-100 times more potent than morphine and has about 25% of the duration of morphine (IV).

B. Tranquilizer- Examples of tranquilizers include midazolam (Versed) and diazepam (Valium).

C. Opioid- Opioids include morphine and meperidine hydrochloride (Demerol).

D. Dissociative agent- Ketamine is a dissociative agent.

94. Which of the following manifestations is often the earliest sign of malignant hyperthermia?

A. Tachycardia (heart rate above 150 beats per minute)
-Tachycardia is often the earliest sign of malignant hyperthermia.

B. Hypotension- Hypotension is a later sign of malignant hyperthermia.

C. Elevated temperature- The rise in temperature is actually a late sign that develops rapidly.

D. Oliguria- Scant urinary output is a later sign of malignant hyperthermia.

95. Which of the following terms is used to refer to protrusion of abdominal organs through the surgical incision?

A. Evisceration-
Evisceration is a surgical emergency.

B. Hernia- A hernia is a weakness in the abdominal wall.

C. Dehiscence- Dehiscence refers to partial or complete separation of wound edges.

D. Erythema- Erythema refers to redness of tissue.

96. When the method of wound healing is one in which wound edges are not surgically approximated and integumentary continuity is restored by granulations, the wound healing is termed

A. second intention healing.- When wounds dehisce, they will be allowed to heal by secondary intention.

B. primary intention healing.- Primary or first intention healing is the method of healing in which wound edges are surgically approximated and integumentary continuity is restored without granulating.

C. first intention healing.- Primary or first intention healing is the method of healing in which wound edges are surgically approximated and integumentary continuity is restored without granulating.

D. third intention healing.- Third intention healing is a method of healing in which surgical approximation of wound edges is delayed and integumentary continuity is restored by bringing apposing granulations together.

97.The nurse recognizes which of the following signs as typical of the patient in shock?

A. Rapid, weak, thready pulse- Pulse increases as the body tries to compensate.

B. Flushed face- Pallor is an indicator of shock.

C. Warm, dry skin- Skin is generally cool and moist in shock.

D. Increased urine output- Usually, a low blood pressure and concentrated urine are observed in the patient in shock.

98. When the nurse observes that the postoperative patient demonstrates a constant low level of oxygen saturation, although the patient's breathing appears normal, the nurse identifies that the patient may be suffering which type of hypoxemia?

A. Subacute-
Supplemental oxygen may be indicated.

B. Hypoxic- Hypoxic hypoxemia results from inadequate breathing.


C. Episodic- Episodic hypoxemia develops suddenly, and the patient may be at risk for myocardial ischemia, cerebral dysfunction, and cardiac arrest.

D. Anemic- Anemic hypoxemia results from blood loss during surgery.

99. When the surgeon performs an appendectomy, the nurse recognizes that the surgical category will be identified as

A. clean contaminated.- Clean-contaminated cases are those with a potential, limited source for infection, the exposure to which, to a large extent, can be controlled.

B. clean.- Clean cases are those with no apparent source of potential infection.

C. contaminated.- Contaminated cases are those that contain an open and obvious source of potential infection.

D. dirty.- A traumatic wound with foreign bodies, fecal contamination, or purulent drainage would be considered a dirty case.

100. Which of the following terms is used to describe inability to breathe easily except in an upright position?

A. Orthopnea- Patients with orthopnea are placed in a high Fowler's position to facilitate breathing.

B. Dyspnea- Dyspnea refers to labored breathing or shortness of breath.

C. Hemoptysis- Hemoptysis refers to expectoration of blood from the respiratory tract.

D. Hypoxemia- Hypoxemia refers to low oxygen levels in the blood.

101.In relation to the structure of the larynx, the cricoid cartilage is

A. the only complete cartilaginous ring in the larynx.- The cricoid cartilage is located below the thyroid cartilage.

B. used in vocal cord movement with the thyroid cartilage.-The arytenoid cartilages are used in vocal cord movement with the thyroid cartilage.

C. the largest of the cartilage structures.- The thyroid cartilage is the largest of the cartilage structures; part of it forms the Adam's apple.

D. the valve flap of cartilage that covers the opening to the larynx during swallowing.
The epiglottis is the valve flap of cartilage that covers the opening to the larynx during swallowing.

102. Which respiratory volume is the maximum volume of air that can be inhaled after maximal expiration?

A. Inspiratory reserve volume- Inspiratory reserve volume is normally 3000 mL.

B. Tidal volume- Tidal volume is the volume of air inhaled and exhaled with each breath.

C. Expiratory reserve volume- Expiratory reserve volume is the maximum volume of air that can be exhaled forcibly after a normal exhalation.

D. Residual volume- Residual volume is the volume of air remaining in the lungs after a maximum exhalation.

103. The individual who demonstrates displacement of the sternum is described as having a

A. pigeon chest.- Pigeon chest may occur with rickets, Marfan's syndrome, or severe kyphoscoliosis.

B. barrel chest.- A barrel chest is seen in patients with emphysema as a result of over-inflation of the lungs.

C. funnel chest.- A funnel chest occurs when there is a depression in the lower portion of the sternum.

D. kyphoscoliosis.- Kyphoscoliosis is characterized by elevation of the scapula and a corresponding S-shaped spine.

104. When the nurse auscultates chest sounds that are harsh and cracking, sounding like two pieces of leather being rubbed together, she records her finding as

A. pleural friction rub.- A pleural friction rub is heard secondary to inflammation and loss of lubricating pleural fluid.

B. crackles.- Crackles are soft, high-pitched, discontinuous popping sounds that occur during inspiration.

C. sonorous wheezes.- Sonorous wheezes are deep, low-pitched rumbling sounds heard primarily during expiration.

D. sibilant wheezes.- Sibilant wheezes are continuous, musical, high-pitched, whistle-like sounds heard during inspiration and expiration.


105. Which of the following terms is used to describe hemorrhage from the nose?

A. Epistaxis- Epistaxis is due to rupture of tiny, distended vessels in the mucous membrane of any area of the nose.

B. Xerostomi- Xerostomia refers to dryness of the mouth.

C. Rhinorrhea- Rhinorrhea refers to drainage of a large amount of fluid from the nose.

D. Dysphasia- Dysphagia refers to difficulties in swallowing.

106. The herpes simplex virus (HSV-1), which produces a cold sore (fever blister), has an incubation period of

A. 2-12 days.- HSV-1 is transmitted primarily by direct contact with infected secretions.

B. 0-3 months.- The time period 0-3 months exceeds the incubation period.

C. 20-30 days.-The time period 20-30 days exceeds the incubation period.

D. 3-6 months.- The time period 3-6 months exceeds the incubation period.

107. Another term for clergyman's sore throat is

A. chronic granular pharyngitis.- In clergyman's sore throat, the pharynx is characterized by numerous swollen lymph follicles.

B. aphonia.- Aphonia refers to the inability to use one's voice.

C. atrophic pharyngitis.- Atrophic pharyngitis is characterized by a membrane that is thin, white, glistening, and at times wrinkled.

D. hypertrophic pharyngitis.- Hypertrophic pharyngitis is characterized by general thickening and congestion of the pharyngeal mucous membrane.

108. Which type of sleep apnea is characterized by lack of airflow due to pharyngeal occlusion?

A. Obstructive
- Obstructive sleep apnea occurs usually in men, especially those who are older and overweight.

B. Simple- Types of sleep apnea do not include a simple characterization.

C. Mixed- Mixed sleep apnea is a combination of central and obstructive apnea with one apneic episode.

D. Central- In central sleep apnea, the patient demonstrates simultaneous cessation of both airflow and respiratory movements.

109. When the patient who has undergone laryngectomy suffers wound breakdown, the nurse monitors him very carefully because he is identified as being at high risk for

A. carotid artery hemorrhage.-The carotid artery lies close to the stoma and may rupture from erosion if the wound does not heal properly.

B. pulmonary embolism.- Pulmonary embolism is associated with immobility.

C. dehydration.- Dehydration may lead to poor wound healing and breakdown.

D. pneumonia.- Pneumonia is a risk for any postoperative patient.

110. Which of the following terms refers to lung tissue that has become more solid in nature due to a collapse of alveoli or infectious process?

A. Consolidation- Consolidation occurs during an infectious process such as pneumonia.

B. Atelectasis- Atelectasis refers to collapse or airless condition of the alveoli caused by hypoventilation, obstruction to the airways, or compression.

C. Bronchiectasis- Bronchiectasis refers to chronic dilation of a bronchi or bronchi in which the dilated airway becomes saccular and a medium for chronic infection.

D. Empyema- Empyema refers to accumulation of purulent material in the pleural space.

111. Which of the following community-acquired pneumonias demonstrates the highest occurrence during summer and fall?

A. Legionnaires' disease- Legionnaires' disease accounts for 15% of community-acquired pneumonias.

B. Streptococcal (pneumococcal) pneumonia- Streptococcal pneumonia demonstrates the highest occurrence in winter months.

C. Mycoplasma pneumonia- Mycoplasma pneumonia demonstrates the highest occurrence in fall and early winter.

D. Viral pneumonia-Viral pneumonia demonstrates the greatest incidence during winter months.

112. When interpreting the results of a Mantoux test, the nurse explains to the patient that a reaction occurs when the intradermal injection site shows

A. redness and induration.-The site is inspected for redness and palpated for hardening.

B. drainage.- Drainage at the site does not indicate a reaction to the tubercle bacillus.

C. tissue sloughing.- Sloughing of tissue at the site of injection does not indicate a reaction to the tubercle bacillus.

D. bruising.- Bruising of tissue at the site may occur from the injection, but does not indicate a reaction to the tubercle bacillus.

113. Which of the following actions is most appropriate for the nurse to take when the patient demonstrates subcutaneous emphysema along the suture line or chest dressing 2 hours after chest surgery?

A. Record the observation.
-Subcutaneous emphysema occurs after chest surgery as the air that is located within the pleural cavity is expelled through the tissue opening created by the surgical procedure.

B. Apply a compression dressing to the area.-Subcutaneous emphysema is a typical post-operative finding in the patient after chest surgery.

C. Measure the patient's pulse oximetry.- Subcutaneous emphysema is absorbed by the body spontaneously after the underlying leak is treated or halted.

D. Report the finding to the physician immediately.- Subcutaneous emphysema results from air entering the tissue planes.

114. Which of the following types of lung cancer is characterized as fast growing and tending to arise peripherally?

A. Large cell carcinoma-
Large cell carcinoma is a fast-growing tumor that tends to arise peripherally.

B. Bronchioalveolar carcinoma- Bronchioalveolar cell cancer arises from the terminal bronchus and alveoli and is usually slow-growing.

C. Adenocarcinoma- Adenocarcinoma presents as peripheral masses or nodules and often metastasizes.

D. Squamous cell carcinoma- Squamous cell carcinoma arises from the bronchial epithelium and is more centrally located.

115. Which of the following methods is the best method for determining nasogastric tube placement in the stomach?

A. X-ray- Radiologic identification of tube placement in the stomach is most reliable.

B. Observation of gastric aspirate- Gastric fluid may be grassy green, brown, clear, or odorless while an aspirate from the lungs may be off-white or tan. Hence, checking aspirate is not the best method of determining nasogastric tube placement in the stomach.

C. Testing of pH of gastric aspirate- Gastric pH values are typically lower or more acidic than that of the intestinal or respiractory tract, but not always.

D. Placement of external end of tube under water- Placement of external end of tube under water and watching for air bubbles is not a reliable method for determining nasogastric tube placement in the stomach.

116. Which of the following types of lung cancer is the most prevalent carcinoma of the lung for both men and women?

A. Adenocarcinoma- Adenocarcinoma presents more peripherally as peripheral masses or nodules and often metastasizes.

B. Large cell carcinoma- Large cell carcinoma is a fast-growing tumor that tends to arise peripherally.

C. Squamous cell carcinoma- Squamous cell carcinoma is more centrally located and arises more commonly in the segmental and subsegmental bronchi in response to repetitive carcinogenic exposures.

D. Small cell carcinoma- Small cell carcinomas arise primarily as proximal lesions, but may arise in any part of the tracheobronchial tree.

117. Emphysema is described as:

A. A disease of the airways characterized by destruction of the walls of overdistended alveoli.- Emphysema is a category of COPD.

B. A disease that results in a common clinical outcome of reversible airflow obstruction.- Asthma is the disease described.

C. The presence of cough and sputum production for at least a combined total of two or three months in each of two consecutive years. - Bronchitis is the disease described.

D. Chronic dilatation of a bronchus or bronchi. -Bronchiectasis is the condition described.

118. Which of the following is the most important risk factor for development of Chronic Obstructive Pulmonary Disease?

A. Cigarette smoking- Pipe, cigar and other types of tobacco smoking are also risk factors.

B. Occupational exposure- While a risk factor, occupational exposure is not the most important risk factor for development of COPD.

C. Air pollution- Air pollution is a risk factor for development of COPD, but it is not the most important risk factor.

D. Genetic abnormalities- A deficiency of alpha-antitrypsin is a risk factor for development of COPD, but it is not the most important risk factor.


119. Which type of chest configuration is typical of the patient with COPD?

A. Barrel chest- "Barrel chest" results from fixation of the ribs in the inspiratory position.

B. Pigeon chest- Pigeon chest results from a displaced sternum.

C. Flail chest- Flail chest results when the ribs are fractured.

D. Funnel chest- Funnel chest occurs when there is a depression in the lower portion of the sternum and is associated with Mafan's syndrome or rickets.

120. In which stage of COPD is the forced expiratory volume (FEV1) < 30%?

A. III- Stage III patients demonstrate FEV1 < 30% with respiratory failure or clinical signs of right heart failure

B. II- Stage II patients demonstrate FEV1 between > 30% and 80%

C. I- Stage I is mild COPD with FEV1 < 70%.

D. O- Stage O is characterized by normal spirometry

121. Of the following oxygen administration devices, which has the advantage of providing high oxygen concentration?

A. Non-rebreather mask-
The non-rebreather mask provides high oxygen concentration but is usually poor fitting.

B. Venturi mask- The Venturi mask provides low levels of supplemental oxygen.

C. Catheter- The catheter is an inexpensive device that provides a variable fraction of inspired oxygen and may cause gastric distention.

D. Face tent- A face tent provides a fairly accurate fraction of inspired oxygen, but is bulky and uncomfortable. It would not be the device of choice to provide high oxygen concentration.

122. Which of the following ranges identifies the amount of pressure within the endotracheal tube cuff that is believed to prevent both injury and aspiration?

A. 20-25 mm Hg water pressure.
- Usually the pressure is maintained at less than 25 cm water pressure to prevent injury and at more than 20 cm water pressure to prevent aspiration.

B. 10-15 mm Hg water pressure.- A measure of 10--15 mm Hg water pressure would indicate that the cuff is underinflated.

C. 30-35 mm Hg water pressure.- A measure of 30--35 mm Hg water pressure would indicate that the cuff is overinflated.

D. 0-5 mm Hg water pressure- A measure of 0-5 mm Hg water pressure would indicate that the cuff is underinflated.

123. When performing endotracheal suctioning, the nurse applies suctioning while withdrawing and gently rotating the catheter 360 degrees for which of the following time periods?

A. 10-15 seconds- In general, the nurse should apply suction no longer than 10-15 seconds because hypoxia and dysrhythmias may develop, leading to cardiac arrest.

B. 30-35 seconds- Applying suction for 30-35 seconds is hazardous and may result in the patient's developing hypoxia, which can lead to dysrhythmias and, ultimately, cardiac arrest.

C. 20-25 seconds- Applying suction for 20-25 seconds is hazardous and may result in the patient's developing hypoxia, which can lead to dysrhythmias and, ultimately, cardiac arrest.

D. 0-5 seconds- Applying suction for 0-5 seconds would provide too little time for effective suctioning of secretions.

124. In general, chest drainage tubes are not used for the patient undergoing

A. Pneumonectomy- Usually, no drains are used for the pneumonectomy patient because the accumulation of fluid in the empty hemithorax prevents mediastinal shift.

B. Lobectomy - With lobectomy, two chest tubes are usually inserted for drainage, the upper for air and the lower for fluid

C. Wedge resection- With wedge resection, the pleural cavity usually is drained because of the possibility of an air or blood leak

D. Segmentectomy- With segmentectomy, drains are usually used because of the possibility of an air or blood leak.

125. Which term is used to describe the ability of the heart to initiate an electrical impulse?

A. Automaticity- Automaticity is the ability of specialized electrical cells of the cardiac conduction system to initiate an electrical impulse.

B. Contractility- Contractility refers to the ability of the specialized electrical cells of the cardiac conduction system to contract in response to an electrical impulse.

C. Conductivity- Conductivity refers to the ability of the specialized electrical cells of the cardiac conduction system to transmit an electrical impulse from one cell to another.

D. Excitability- Excitability refers to the ability of the specialized electrical cells of the cardiac conduction system to respond to an electrical impulse.


126. The nurse auscultates the apex beat at which of the following anatomical locations?

A. Fifth intercostal space, midclavicular line- The left ventricle is responsible for the apex beat or the point of maximum impulse, which is normally palpable in the left midclavicular line of the chest wall at the fifth intercostal space.

B. Mid-sternum- The right ventricle lies anteriorly, just beneath the sternum.

C. 2” to the left of the lower end of the sternum- Use of inches to identify the location of the apex beat is inappropriate based upon variations in human anatomy.

D. 1” to the left of the xiphoid process- Auscultation below and to the left of the xiphoid process will detect gastrointestinal sounds, but not the apex beat of the heart.

127. Which of the following terms describes the amount of blood ejected per heartbeat?

A. Stroke volume- Stroke volume is determined by preload, afterload, and contractility.

B. Cardiac output- Cardiac output is the amount of blood pumped by each ventricle during a given period and is computed by multiplying the stroke volume of the heart by the heart rate.

C. Ejection fraction- Ejection Fraction is the percentage of the end-diastolic volume that is ejected with each stroke, measured at 42--50% in the normal heart.

D. Afterload- Afterload is defined as the pressure that the ventricular myocardium must overcome to eject blood during systole and is one of the determinants of stroke volume.


128. When measuring the blood pressure in each of the patient's arms, the nurse recognizes that in the normal adult, the pressures

A. differ no more than 5 mm Hg between arm pressures.-Normally, in the absence of disease of the vasculature, there is a difference of no more than 5 mm Hg between arm pressures.

B. must be equal in both arms.- The pressures in each arm do not have to be equal in order to be considered normal.

C. may vary 10 mm Hg or more between arms.- Pressures that vary more than 10 mm Hg between arms indicate an abnormal finding.

D. may vary, with the higher pressure found in the left arm.- The left arm pressure is not anticipated to be higher than the right as a normal anatomical variant.

129. Central venous pressure is measured in which of the following heart chambers?

A. Right atrium-
The pressure in the right atrium is used to assess right ventricular function and venous blood return to the heart.

B. Left atrium- The left atrium receives oxygenated blood from the pulmonary circulation.

C. Left ventricle- The left ventricle receives oxygenated blood from the left atrium.

D. Right ventricle- The right ventricle is not the central collecting chamber of venous circulation.

130. Which of the following ECG characteristics is usually seen when a patient's serum potassium level is low?

A. U wave-
The U wave is an ECG waveform characteristic that may reflect Purkinje fiber repolarization. It is usually seen when a patient's serum potassium level is low.

B. T wave- The T wave is an ECG characteristic reflecting repolarization of the ventricles. It may become tall or "peaked" if a patient's serum potassium level is high.

C. P wave- The P wave is an ECG characteristic reflecting conduction of an electrical impulse through the atria.

D. QT interval- The QT interval is an ECG characteristic reflecting the time from ventricular depolarization to repolarization.

131. Which of the following ECG waveforms characterizes conduction of an electrical impulse through the left ventricle?

A. QRS complex- The QRS complex represents ventricular depolarization

B. P wave- The P wave is an ECG characteristic reflecting conduction of an electrical impulse through the atria.

C. PR interval- The PR interval is a component of an ECG tracing reflecting conduction of an electrical impulse through the AV node.

D. QT interval- The QT interval is an ECG characteristic reflecting the time from ventricular depolarization to repolarization.

132. When the nurse observes that the patient's heart rate increases during inspiration and decreases during expiration, the nurse reports that the patient is demonstrating

A. sinus dysrhythmia.- Sinus dysrhythmia occurs when the sinus node creates an impulse at an irregular rhythm.

B. normal sinus rhythm.- Normal sinus rhythm occurs when the electrical impulse starts at a regular rate and rhythm in the SA node and travels through the normal C. conduction pathway.

C. sinus bradycardia.- Sinus bradycardia occurs when the sinus node regularly creates an impulse at a slower-than-normal rate.

D. sinus tachycardia.- Sinus tachycardia occurs when the sinus node regularly creates an impulse at a faster-than-normal rate.

133. Which of the following terms is used to describe a tachycardia characterized by abrupt onset, abrupt cessation, and a QRS of normal duration?

A. Paroxysmal atrial tachycardia-
PAT is often caused by a conduction problem in the AV node and is now called AV nodal reentry tachycardia.

B. Sinus tachycardia- Sinus tachycardia occurs when the sinus node regularly creates an impulse at a faster-than-normal rate.

C. Atrial flutter- Atrial flutter occurs in the atrium and creates an atrial rate between 250-400 times per minute.

D. Atrial fibrillation- Atrial fibrillation causes a rapid, disorganized, and uncoordinated twitching of atrial musculature.

134. When the nurse observes an ECG tracing on a cardiac monitor with a pattern in lead II and observes a bizarre, abnormal shape to the QRS complex, the nurse has likely observed which of the following ventricular dysrhythmias?

A. Premature ventricular contraction-
A PVC is an impulse that starts in a ventricle before the next normal sinus impulse.

B. Ventricular bigeminy- Ventricular bigeminy is a rhythm in which every other complex is a PVC.

C. Ventricular tachycardia-Ventricular tachycardia is defined as three or more PVCs in a row, occurring at a rate exceeding 100 beats per minute.

D. Ventricular fibrillation- Ventricular fibrillation is a rapid but disorganized ventricular rhythm that causes ineffective quivering of the ventricles.

135. Premature ventricular contractions are considered precursors of ventricular tachycardia when they

A. occur at a rate of more than six per minute.-When PVCs occur at a rate of more than six per minute they indicate increasing ventricular irritability and are considered forerunners of ventricular tachycardia (VT).

B. occur during the QRS complex.- PVCs are dangerous when they occur on the T wave.

C. have the same shape.- PVCs are dangerous when they are multifocal (have different shapes).

D. are paired with a normal beat.- A PVC that is paired with a normal beat is termed bigeminy.

136. When no atrial impulse is conducted through the AV node into the ventricles, the patient is said to be experiencing which type of AV block?

A. Third degree-
In third degree heart block, two impulses stimulate the heart—one stimulates the ventricles and one stimulates the atria.

B. First degree- In first degree heart block, all the atrial impulses are conducted through the AV node into the ventricles at a rate slower than normal.

C. Second degree, type I- In second degree AV block, type I, all but one of the atrial impulses are conducted through the AV node into the ventricles.

D. Second degree, type II- In second degree AV block, type II, only some of the atrial impulses are conducted through the AV node into the ventricles.

137. Which of the following terms refers to chest pain brought on by physical or emotional stress and relieved by rest or medication?

A. angina pectoris- Angina pectoris is a symptom of myocardial ischemia.

B. atherosclerosis- Atherosclerosis is an abnormal accumulation of lipid deposits and fibrous tissue within arterial walls and lumens.

C. atheroma- Atheromas are fibrous caps composed of smooth muscle cells that form over lipid deposits within arterial vessels.

D. ischemia- Ischemia is insufficient tissue oxygenation and may occur in any part of the body.

138. Of the following risk factors, which is considered modifiable?

A. Diabetes mellitus- While diabetes mellitus cannot be cured, blood sugars and symptomatology can be managed through healthy heart living.

B. Gender- Gender is considered a non-modifiable risk factor.

C. Race- Race is considered a non-modifiable risk factor.

D. Increasing age- Increasing age is considered a non-modifiable risk factor.

139. When the patient with known angina pectoris complains that he is experiencing chest pain more frequently even at rest, the period of pain is longer, and it takes less stress for the pain to occur, the nurse recognizes that the patient is describing

A. unstable angina.- Unstable angina is also called crescendo or pre-infarction angina and indicates the need for a change in treatment.

B. intractable angina.- Intractable or refractory angina produces severe, incapacitating chest pain that does not respond to conventional treatment.

C. variant angina.- Variant angina is described as pain at rest with reversible ST-segment elevation and is thought to be caused by coronary artery vasospasm.

D. refractory angina.- Intractable or refractory angina produces severe, incapacitating chest pain that does not respond to conventional treatment.

140. Heparin therapy is usually considered therapeutic when the patient's activated partial thromboplasin time (aPTT) is how many times normal?

A. 1.5 to 2- The amount of heparin administered is based on aPTT results, which should be obtained in follow-up to any alteration of dosage.

B. .5 to 1- The patient's aPTT value would have to be greater than .5 to 1 times normal to be considered therapeutic.

C. 2.5 to 3- An aPTT value that is 2.5 to 3 times normal would be too high to be considered therapeutic.

D. .25 to .75- The patient's aPTT value would have to be greater than .25 to .75 times normal to be considered therapeutic.

141. When the post-cardiac surgery patient demonstrates restlessness, nausea, weakness, and peaked T waves, the nurse reviews the patient's serum electrolytes anticipating which abnormality?

A. Hyperkalemia-
Hyperkalemia is indicated by mental confusion, restlessness, nausea, weakness, and dysrhythmias (tall, peaked T waves).

B. Hypercalcemia- Hypercalcemia would likely be demonstrated by asystole.

C. Hypomagnesemia- Hypomagenesemia would likely be demonstrated by hypotension, lethargy, and vasodilation.

D. Hyponatremia- Hyponatremia would likely be indicated by weakness, fatigue, and confusion without change in T wave formation.

142. In order to be effective, Percutaneous Transluminal Coronary Angioplasty (PTCA) must be performed within what time frame, beginning with arrival at the emergency department after diagnosis of myocardial infarction?

A. 60 minutes- The sixty minute interval is known as "door to balloon time" for performance of PTCA on a diagnosed MI patient.

B. 30 minutes- The thirty minute interval is known as "door to needle" time for administration of thrombolytics post MI.

C. 9 days- The time frame of nine (9) days refers to the time for onset of vasculitis after administration of Streptokinase for thrombolysis in an acute MI patient.

D. 6-12 months- The six to twelve month time frame refers to the time period during which streptokinase will not be used again in the same patient for acute MI.

143. Which of the following statements reflect a goal of rehabilitation for the patient with an MI:

A. To improve the quality of life- Overall, cardiac rehabilitation is a complete program dedicated to extending and improving quality of life.

B. To limit the effects and progression of atherosclerosis- An immediate objective of rehabilitation of the MI patient is to limit the effects and progression of atherosclerosis.

C. To return the patient to work and a pre illness lifestyle- An immediate objective of rehabilitation of the MI patient is to return the patient to work and a pre illness lifestyle.

D. To prevent another cardiac event- An immediate objective of rehabilitation of the MI patient is to prevent another cardiac event.

144. Which of the following methods to induce hemostasis after sheath removal post Percutaneous Transluminal Coronary Angioplasty is the least effective?

A. Application of a sandbag to the area- Several nursing interventions frequently used as part of the standard of care, such as applying a sandbag to the sheath insertion site, have not been shown to be effective in reducing the incidence of bleeding.

B. Application of a vascular closure device, such as AngiosealTM, VasosealTM, DuettTM, Syvek patchTM- Application of a vascular closure device has been demonstrated to be very effective.

C. Direct manual pressure- Direct manual pressure to the sheath introduction site has been demonstrated to be effective and was the first method used to induce hemostasis post PTCA.

D. Application of a pneumatic compression device (e.g., Fem-StopTM)-Application of a pneumatic compression device post PTCA has been demonstrated to be effective.

145. A long-term effect of which of the following procedures post acute MI induces angioneogenesis?

A. Transmyocardial laser revascularization- TNR procedures usually involves making 20 to 40 channels in ventricular muscle. It is thought that some blood flows into the channels, decreasing the ischemia directly. Within the next few days to months, the channels close as a result of the body’s inflammatory process of healing a wound and new blood vessels form as a result of the inflammatory process.

B. Bracytherapy- Brachytherapy involves the delivery of gamma or beta radiation by placing a radioisotope close to the lesion and has been shown to be effective in reducing the recurrence of obstruction, preventing vessell restenosis by inhibiting smooth muscle cell proliferation.

C. Atherectomy- Atherectomy is an invasive interventional procedure that involves the removal of the atheroma, or plaque, from a coronary artery.

D. Stent placement- A stent is a woven stainless steel mesh that provides structural support to a vessel at risk of acute closure. Eventually, endothelium covers the stent and it is incorporated into the vessel wall. Because of the risk of thrombus formation in the stent, the patient receives antiplatelet medications (e.g., clopidigrel [Plavix]) therapy for 2 weeks and lifetime use of aspirin).

146. Which of the following medications are used to reverse the effects of heparin?

A. Protamine sulfate-
Protamine sulfate is known as the antagonist to heparin.

B. Streptokinase-Streptokinase is a thrombolytic agent.

C. Clopidigrel (Plavix)- Clopidigrel (Plavix) is an antiplatelet medication that is given to reduce the risk of thrombus formation post coronary stent placement.

D. Aspirin- The antiplatelet effect of aspirin does not reverse the effects of heparin.

147. Which of the following terms refers to leg pain that is brought on walking and caused by arterial insufficiency?

A. Intermittent claudication- Intermittent claudication is leg pain that is brought on by exercise and relieved by rest.

B. Dyspnea- Dyspnea is the patient's subjective statement of difficulty breathing.

C. Orthopnea- Orthopnea is the inability of the patient to breathe except in the upright (sitting) position.

D. Thromboangitis obliterans- Thomroangitis obliterans is a peripheral vascular disease also known as Burger's disease.

148. When the post-cardiac surgical patient demonstrates vasodilation, hypotension, hyporeflexia, slow gastrointestinal motility (hypoactive bowel sounds), lethargy, and respiratory depression, the nurse suspects which of the following electrolyte imbalances?

A. Hypermagnesemia- Untreated hypomagnesemia may result in coma, apnea, cardiac arrest.

B. Hypokalemia- Signs and symptoms of hypokalemia include signs of digitalis toxicity and dysrhythmias (U wave, AV block, flat or inverted T waves).

C. Hyperkalemia- Signs of hyperkalemia include: mental confusion, restlessness, nausea, weakness, paresthesias of extremities, dysrhythmias (tall, peaked T waves; increased amplitude, widening QRS complex; prolonged QT interval).

D. Hypomagnesemia- Signs and symptoms of hypomagnesemia include: paresthesias, carpopedal spasm, muscle cramps, tetany, irritability, tremors, hyperexcitability, hyperreflexia, cardiac dysrhythmias (prolonged PR and QT intervals, broad flat T waves), disorientation, depression, and hypotension.

149. When the nurse notes that the post cardiac surgery patient demonstrates low urine output (<> 1.025), the nurse suspects:

A. Inadequate fluid volume-
Urine output of less than 25 ml/hr may indicate a decrease in cardiac output. A high specific gravity indicates increased concentration of solutes in the urine which occurs with inadequate fluid volume.

B. Normal glomerular filtration- Indices of normal glomerular filtration are output of 25 ml or greater per hour and specific gravity between 1.010 and 1.025.

C. Overhydration- Overhydration is manifested by high urine output with low specific gravity.

D. Anuria- The anuric patient does not produce urine.

150. When the valve used in valve replacement surgery is made from the patient's own heart valve, which of the following terms is used?

A. Autograft- An example of autograft is found when the surgeon excises the pulmonic valve and uses it for an aortic valve replacement.

B. Allograft
Allograft refers to replacement using human tissue and is a synonym for homograft.

C. Homograft- Homograft refers to replacement using human tissue and is a synonym for allograft.

D. Xenograft- Xenograft refers to replacement of tissue from animal tissue.

151. Which of the following procedures most specifically describes splitting or separating fused cardiac valve leaflets?

A. Commisurotomy-
Commisurotomy is frequently used for mitral stenosis.

B. Annuloplasty- Annuloplasty refers to repair of a cardiac valve's outer ring.

C. Chordoplasty- Chordoplasty refers to repair of the chordae tendonae of atroventricular valve leaflets.

D. Valvuloplasty- Valvuloplasty is a general term that refers to repair of a stenosed or regurgitant cardiac valve by commisurotomy, annuloplasty, leaflet repair, and/or chordoplasty.

152. Which of the following mitral valve conditions generally produces no symptoms?

A. Prolapse- Mitral valve prolpase is a deformity that usually produces no symptoms and has been diagnosed more frequently in recent years, probably as a result of improved diagnostic methods.

B. Stenosis- Mitral valve stenosis usually causes progressive fatigue.

C. Regurgitation- Mitral valve regurgitation, in its acute stage, usually presents as severe heart failure.

D. Infection- Mitral valve infection, when acute, will produce symptoms typical of infective endocarditis.

153. In which type of cardiomyopathy does the heart muscle actually increase in size and mass weight, especially along the septum?

A. Hypertrophic- Because of the structural changes, hypertrophic cardiomyopathy had also been called idiopathic hypertrophic subaortic stenosis (IHSS) or asymmetric septal hypertrophy (ASH).

B. Dilated - Because of the structural changes, hypertrophic cardiomyopathy had also been called idiopathic hypertrophic subaortic stenosis (IHSS) or asymmetric septal hypertrophy (ASH).

C. Restrictive- Restrictive cardiomyopathy is characterized by diastolic dysfunction caused by rigid ventricular walls that impair ventricular stretch and diastolic filling

D. Arrhythmogenic right ventricular cardiomyopathy - Arrhythmogenic right ventricular cardiomyopathy (ARVC) occurs when the myocardium of the right ventricle is progressively infiltrated and replaced by fibrous scar and adipose tissue

154. Which of the following patient behaviors, if observed by the nurse, would indicate that the cardiac patient's level of anxiety has decreased?

A. Answers questions regarding status with no problem.
-Generally, when anxiety begins to increase, the patient will be less likely to want to discuss prognosis.

B. Discusses prognosis freely.- Open discussion generally indicates some degree of comfort with prognosis.

C. Verbalizes fears and concerns.- Verbalization of fears and concerns indicates some degree of comfort with prognosis.

D. Participates in support groups.- Participation in support groups indicates some degree of comfort with prognosis.

155. The patient with which of the follow characteristics is considered high risk for the development of infective endocarditis?

A. The patient who has complex cyanotic congenital malformations is at high risk for the development of infective endocarditis.

B. Mitral valve prolapse with valvular regurgitation .-The patient with mitral valve prolapse with valvular regurgitation is at moderate risk for the development of infective endocarditis.

C. Hypertrophic cardiomyopathy. The patient with hypertrophic cardiomyopathy is at moderate risk for the development of infective endocarditis.

D. Acquired valvular dysfunction. The patient with acquired valvular dysfunction is at moderate risk for the development of infective endocarditis.


156. Which of the following terms indicates the amount of blood pumped out of the ventricle with each contraction of the heart?

A. Stroke volume- Stroke volume is the amount of blood pumped out (ejected) with each contraction of the heart (heart beat). Stroke volume times heart rate equals cardiac output.

B. Afterload- Afterload is the amount of resistance to ejection of blood from a ventricle.

C. Cardiac output- Cardiac output is the amount of blood pumped out of the heart in one minute.

D. Preload- Preload is the pressure created by a volume of blood within a ventricle before contraction.

157. When the balloon on the distal tip of a pulmonary artery catheter is inflated and a pressure is measured, the measurement obtained is referred to as the

A. pulmonary artery wedge pressure.
- When the balloon is inflated, the tip of the catheter floats into smaller branches of the pulmonary arty until it can no longer be passed and the pressure is recorded, reflecting left atrial pressure and left ventricular end-diastolic pressure.

B. central venous pressure.- Central venous pressure is measured in the right atrium.

C. pulmonary artery pressure.- Pulmonary artery pressure is measured when the balloon tip is not inflated.

D. cardiac output.- Cardiac output is determined through thermodilution involving injection of fluid into the pulmonary artery catheter.

158. Which of the following medications is categorized as a loop diuretic?

A. Furosemide (Lasix)- Lasix is commonly used in the treatment of cardiac failure.

B. Chlorothiazide (Diuril)- Chlorothiazide is categorized as a thiazide diuretic.

C. Chlorthalidone (Hygroton)- Chlorothalidone is categorized as a thiazide diuretic.

D. Spironolactone (Aldactone)- Spironolactone is categorized as a potassium-sparing diuretic.

159. When the nurse observes that the patient always has difficulty breathing when lying flat, the nurse records that the patient is demonstrating

A. Orthopnea
- Patients with orthopnea prefer not to lie flat and will need to maintain their beds in a semi- to high Fowler's position

B. Dyspnea on exertion.- Dyspnea on exertion refers to difficulty breathing with activity.

C. Hyperpnea.- Hyperpnea refers to increased rate and depth of respiration.

D. Paroxysmal nocturnal dyspnea.- Paroxysmal nocturnal dyspnea refers to orthopnea that occurs only at night.

160. The patient with cardiac failure is taught to report which of the following symptoms to the physician or clinic immediately?

A. Persistent cough
- Persistent cough may indicate an onset of left-heart failure.

B. Increased appetite- Loss of appetite should be reported immediately.

C. Weight loss- Weight gain should be reported immediately.

D. Ability to sleep through the night- Frequent urination, causing interruption of sleep, should be reported immediately.

161. A classic sign of cardiogenic shock is

A. Tissue hypoperfusion -Tissue hypoperfusion is manifested as cerebral hypoxia (restlessness, confusion, agitation).

B. High blood pressure- Low blood pressure is a classic sign of cardiogenic shock.

C. Hyperactive bowel sounds- Hypoactive bowel sounds are classic signs of cardiogenic shock.

D. Increased urinary output- Decreased urinary output is a classic sign of cardiogenic shock.

162. Vasoactive drugs which cause the arteries and veins to dilate, thereby shunting much of the intravascular volume to the periphery and causing a reduction in preload and afterload include agents such as

A. Sodium nitroprusside (Nipride)- Sodium nitroprusside is used in the treatment of cardiogenic shock.

B. Norepinephrine (Levophed)-Norepinephrine (Levophed) is a vasopressor that is used to promote perfusion to the heart and brain.

C. Dopamine (Inotropin)- Dopamine (Inotropin) tends to increase the workload of the heart by increasing oxygen demand; thus, it is not administered early in the treatment of cardiogenic shock.

D. Furosemide (Lasix)-Furosemide (Lasix) is a loop diuretic that reduces intravascular fluid volume.



163. Which of the following terms refers to a muscular, cramp-like pain in the extremities consistently reproduced with the same degree of exercise and relieved by rest?

A. Intermittent claudication- Intermittent claudication is a sign of peripheral arterial insufficiency.

B. Aneurysm- An aneurysm is a localized sac of an artery wall formed at a weak point in the vessel.

C. Bruit- A bruit is the sound produced by turbulent blood flow through an irregular, tortuous, stenotic, or dilated vessel.

D. Ischemia- Ischemia is a term used to denote deficient blood supply.

164. Which of the following observations regarding ulcer formation on the patient's lower extremity indicate to the nurse that the ulcer is a result of venous insufficiency?

A. The border of the ulcer is irregular.-
The border of an ulcer caused by arterial insufficiency is circular.

B. The ulcer is very painful to the patient, even though superficial.-Superficial venous insufficiency ulcers cause minimal pain.

C. The ulcer base is pale to black.- The base of a venous insufficiency ulcer shows beefy red to yellow fibrinous color.

D. The ulcer is deep, involving the joint space.- Venous insufficiency ulcers are usually superficial.

165. A diagnostic test that involves injection of a contrast media into the venous system through a dorsal vein in the foot is termed

A. contrast phlebography.-
When a thrombus exists, an x-ray image will disclose an unfilled segment of a vein.

B. air plethysmography- Air plethysmography quantifies venous reflux and calf muscle pump ejection.

C. lymphangiography.- In lymphangiography, contrast media are injected into the lymph system.

D. lymphoscintigraphy.- In lymphoscintigraphy, a radioactive-labeled colloid is injected into the lymph system.

166. The nurse teaches the patient with peripheral vascular disease to refrain from smoking because nicotine causes

A. vasospasm.-
Nicotine causes vasospasm and can thereby dramatically reduce circulation to the extremities.

B. slowed heart rate.- Nicotine has stimulant effects.

C. depression of the cough reflex.- Nicotine does not suppress cough. Smoking irritates the bronchial tree, causing coughing.

D. diuresis.- Nicotine does not cause diuresis.

167. Which of the following types of aneurysms results in bleeding into the layers of the arterial wall?

A. Dissecting- Dissection results from a rupture in the intimal layer, resulting in bleeding between the intimal and medial layers of the arterial wall.

B. Saccular- Saccular aneurysms collect blood in the weakened outpouching.

C. False- In a false aneurysm, the mass is actually a pulsating hematoma.

D. Anastomotic- An anastomotic aneurysm occurs as a result of infection at arterial suture or graft sites.

168. Which of the following terms refers to enlarged, red, and tender lymph nodes?

A. Lymphadenitis- Acute lymphadenitis is demonstrated by enlarged, red and tender lymph nodes.

B. Lymphangitis- Lymphangitis is an acute inflammation of the lymphatic channels.

C. Lymphedema- Lymphedema is demonstrated by swelling of tissues in the extremities because of an increased quantity of lymph that results from an obstruction of lymphatic vessels.

D. Elephantiasis- Elephantiasis refers to a condition in which chronic swelling of the extremity recedes only slightly with elevation.

169. Which of the following terms is given to hypertension in which the blood pressure, which is controlled with therapy, becomes uncontrolled (abnormally high) with the discontinuation of therapy?

A. Rebound-
Rebound hypertension may precipitate a hypertensive crisis.

B. Essential- Essential or primary hypertension denotes high blood pressure from an unidentified source.

C. Primary- Essential or primary hypertension denotes high blood pressure from an unidentified source.

D. Secondary- Secondary hypertension denotes high blood pressure from an identified cause, such as renal disease.

170. Officially, hypertension is diagnosed when the patient demonstrates a systolic blood pressure greater than ______ mm Hg and a diastolic blood pressure greater than _____ mm Hg over a sustained period.

A. 140, 90- According to the categories of blood pressure levels established by the JNC VI, stage 1 hypertension is demonstrated by a systolic pressure of 140--159 or a diastolic pressure of 90--99.

B. 130, 80- Pressure of 130 systolic and 80 diastolic falls within the normal range for an adult.

C. 110, 60- Pressure of 110 systolic and 60 diastolic falls within the normal range for an adult.

D. 120, 70- Pressure of 120 systolic and 70 diastolic falls within the normal range for an adult.

171. The nurse teaches the patient which of the following guidelines regarding lifestyle modifications for hypertension?

A. Maintain adequate dietary intake of potassium- In general, one serving of a potassium-rich food such as banana, kale, broccoli, or orange juice will meet the daily need for potassium.

B. Reduce smoking to no more than four cigarettes per day-The patient should be guided to stop smoking.

C. Limit aerobic physical activity to 15 minutes, three times per week
The general guideline is to advise the patient to increase aerobic activity to 30 to 45 minutes most days of the week.

D. Stop alcohol intake- In general, alcohol intake should be limited to no more than 1 oz of ethanol per day.

172. Of the following diuretic medications, which conserves potassium?

A. Spironolactone (Aldactone)- Aldactone is known as a potassium-sparing diuretic.

B. Furosemide (Lasix)- Lasix causes loss of potassium from the body.

C. Chlorothiazide (Diuril)- Diuril causes mild hypokalemia.

D. Chlorthalidone (Hygroton)- Hygroton causes mild hypokalemia.

173. Which of the following adrenergic inhibitors acts directly on the blood vessels, producing vasodilation?

A. Prazosin hydrochloride (Minipress)- Minipress is a peripheral vasodilator acting directly on the blood vessel. It is not used in angina and coronary artery disease, however, because it induces tachycardia if not preceded by administration of propranolol and a diuretic.

B. Reserpine (Serpasil)- Serpasil impairs synthesis and reuptake of norepinephrine.

C. Propranolol (Inderal)- Inderal blocks the beta-adrenergic receptors of the sympathetic nervous system, especially the sympathetics to the heart, producing a slower heart reate and lowered blood pressure.

D. Clonidine hydrochloride (Catapres)- Catapres acts through the central nervous system, apparently through centrally mediated alpha-adrenergic stimulation in the brain, producing blood pressure reduction.

174. Which of the following terms refers to an abnormal decrease in white blood cells, red blood cells, and platelets?

A. Pancytopenia- Pancytopenia may be congenital or acquired.

B. Anemia- Anemia refers to decreased red cell mass.

C. Leukopenia- Leukopenia refers to a less-than-normal amount of WBCs in circulation.

D. Thrombocytopenia- Thrombocytopenia refers to a lower-than-normal platelet count.

175. Which of the following terms refers to a form of white blood cell involved in immune response?

A. Lymphocyte-
Both B and T lymphocytes respond to exposure to antigens.

B. Granulocyte- Granulocytes include basophils, neutrophils, and eosinophils.

C. Spherocyte- A spherocyte is a red blood cell without central pallor, seen with hemolysis.

D. Thrombocyte- A thombocyte is a platelet.

176. The term that is used to refer to a primitive cell, capable of self-replication and differentiation, is

A. stem cell.- Stem cells may differentiate into myeloid or lymphoid stem cells.

B. band cell.- A band cell is a slightly immature neutrophil.

C. spherocyte.- A spherocyte is a red blood cell without central pallor.

D. reticulocyte.- A reticulocyte is a slightly immature red blood cell.

177. Of the following hemolytic anemias, which is categorized as inherited?

A. Sickle cell anemia- Glucose 6-phosphate dehydrogenase deficiency is an inherited abnormality resulting in hemolytic anemia.

B. Autoimmune hemolytic anemia- Autoimmune hemolytic anemia is an acquired anemia.

C. Cold agglutinin disease- Cold agglutinin disease is an acquired anemia.

D. Hypersplenism- Hypersplenism results in an acquired hemolytic anemia.

178.The antidote to heparin is

A. protamine sulfate.- Protamine sulfate, in the appropriate dosage, acts quickly to reverse the effects of heparin.

B. vitamin K.- Vitamin K is the antidote to warfarin (Coumadin).

C. Narcan.- Narcan is the drug used to reverse signs and symptoms of medication-induced narcosis.

D. Ipecac.- Ipecac is an emetic used to treat some poisonings.

179. Which of the following terms describes a gastric secretion that combines with vitamin B-12 so that it can be absorbed?

A. Intrinsic factor-
Lack of intrinsic factor is a common finding in the aged patient.

B. Amylase- Amylase is an enzyme that aids in the digestion of starch.

C. Pepsin- Pepsin is a gastric enzyme important in protein digestion.

D. Trypsin- Trypsin is an enzyme that aids in the digestion of protein.

180. When bowel sounds are heard about every 15 seconds, the nurse would record that the bowel sounds are

A. normal.- Normal bowel sounds are heard every 5-20 seconds.

B. hypoactive.- Hypoactive bowel sounds is the description given to auscultation of one to two bowel sounds in 2 minutes.

C. sluggish.- Sluggish is not a term a nurse would use to accurately describe bowel sounds.

D. absent.- The nurse records that bowel sounds are absent when no sound is heard in 3-5 minutes.

181. When gastric analysis testing reveals excess secretion of gastric acid, which of the following diagnoses is supported?

A. Duodenal ulcer-
Patients with duodenal ulcers usually secrete an excess amount of hydrochloric acid.

B. Chronic atrophic gastritis- Patients with chronic atrophic gastritis secrete little or no acid.

C. Gastric cancer- Patients with gastric cancer secrete little or no acid.

D. Pernicious anemia- Patients with pernicious anemia secrete no acid under basal conditions or after stimulation.

182. Which of the following terms is used to describe stone formation in a salivary gland, usually the submandibular gland?

A. Sialolithiasis- Salivary stones are formed mainly from calcium phosphate.

B. Parotitis- Parotitis refers to inflammation of the parotid gland.

C. Sialadenitis- Sialadenitis refers to inflammation of the salivary glands.

D. Stomatitis- Stomatitis refers to inflammation of the oral mucosa.

183. Irritation of the lips associated with scaling, crust formation, and fissures is termed

A. leukoplakia.-
Leukoplakia is characterized by white patches, usually on the buccal mucosa.

B. lichen planus.- Lichen planus refers to white papules at the intersection of a network of interlacing lesions.

C. actinic cheilitis.- Actinic cheilitis is the result of cumulative exposure to sun.

D. chancre.- A chancre is demonstrated as a reddened circumscribed lesion that ulcerates and becomes crusted and is a primary lesion of syphilis.

184. Regarding oral cancer, the nurse provides health teaching to inform the patient that

A. many oral cancers produce no symptoms in the early stages.- As the cancer progresses, the patient may complain of tenderness or difficulty in chewing, swallowing, or speaking.

B. most oral cancers are painful at the outset.- The most frequent symptom of oral cancer is a painless sore that will not heal.

C. Blood testing is used to diagnose oral cancer.- Biopsy is used to diagnose oral cancer.

D. a typical lesion is soft and crater-like.- A typical lesion in oral cancer is a painless hardened ulcer with raised edges.

185. The most common symptom of esophageal disease is

A. dysphagia.- This symptom may vary from an uncomfortable feeling that a bolus of food is caught in the upper esophagus to acute pain on swallowing.

B. nausea.- Nausea is the most common symptom of gastrointestinal problems in general.

C. vomiting.- Vomiting is a nonspecific symptom that may have a variety of causes.

D. odynophagia.- Odynophagia refers specifically to acute pain on swallowing.

186. Halitosis and a sour taste in the mouth are signs and symptoms associated most directly with

A. esophageal diverticula.- Because the diverticula may retain decomposed food, halitosis and a sour taste in the mouth are frequent complaints.

B. achalasia.- Achalasia presents as difficulty in swallowing both liquids and solids.

C. gastroesophageal reflux.- Gastroesophageal reflux presents as burning in the esophagus, indigestion, and difficulty in or pain upon swallowing.

D. hiatal hernia.- Hiatal hernia presents as heartburn, regurgitation, and dysphagia in many patients while at least 50% are asymptomatic.


187. Which of the following terms refers to the symptom of gastroesophageal reflux disease (GERD) which is characterized by a burning sensation in the esophagus?

A. Pyrosis-
Pyrosis refers to a burning sensation in the esophagus and indicates GERD.

B. Dyspepsia- Indigestion is termed dyspepsia.

C. Dysphasia- Difficulty swallowing is termed dysphagia.

D. Odynophagia- Pain on swallowing is termed odynophagia.

188.The nurse teaches the patient with gastroesophageal reflux disease (GERD) which of the following measures to manage his disease?

A. Avoid eating or drinking 2 hours before bedtime.-
The patient should not recline with a full stomach.

B. Minimize intake of caffeine, beer, milk, and foods containing peppermint and spearmint.-The patient should be instructed to avoid the listed foods and food components.

C. Elevate the foot of the bed on 6- to 8-inch blocks. -The patient should be instructed to elevate the head of the bed on 6- to 8-inch blocks.

D. Eat a low carbohydrate diet- The patient is instructed to eat a low-fat diet

189. Which of the following statements accurately describes cancer of the esophagus?

A. Chronic irritation of the esophagus is a known risk factor.-
In the United States, cancer of the esophagus has been associated with the ingestion of alcohol and the use of tobacco.

B. It is three times more common in women in the U.S. than men.
In the United States, carcinoma of the esophagus occurs more than three times as often in men as in women.

C. It is seen more frequently in Caucasians than in African Americans.-It is seen more frequently in African Americans than in Caucasians.

D. It usually occurs in the fourth decade of life. - It usually occurs in the fifth decade of life.

190. Which of the following venous access devices can be used for no more than 30 days in patients requiring parenteral nutrition?

A. Non-tunneled catheter.The subclavian vein is the most common vessel used because the subclavian area provides a stable insertion site to which the catheter can be anchored, allows the patient freedom of movement, and provides easy access to the dressing site.

B. Peripherally-inserted central catheter (PICC)- PICC lines may be used for intermediate terms (3-12 months).

C. Tunneled catheters- Tunneled central catheters are for long-term use and may remain in place for many years.

D. Implanted ports- Implanted ports are devices also used for long term home IV therapy (eg, Port A Cath, Mediport, Hickman Port, P.A.S. Port).

191.To ensure patency of central venous line ports, dilute heparin flushes are used in which of the following situations?

A. Daily when not in use- Daily instillation of dilute heparin flush when a port is not in use will maintain the port.

B. With continuous infusions- Continuous infusion maintains the patency of each port. Heparin flushes are used after each intermittent infusion.

C. Before blood drawing- Heparin flushes are used after blood drawing in order to prevent clotting of blood within the port.

D. When the line is discontinued- Heparin flush of ports is not necessary if a line is to be discontinued.

192. For which of the following medications must the nurse contact the pharmacist in consultation when the patient receives all oral medications by feeding tube?

A. Enteric-coated tablets-
Enteric-coated tablets are meant to be digested in the intestinal tract and may be destroyed by stomach acids. A change of form of medication is required by patients with tube feedings.

B. Simple compressed tablets- Simple compressed tablets may be crushed and dissolved in water for patient receiving oral medications by feeding tube.

C. Buccal or sublingual tablets- Buccal or sublingual tablets are absorbed by mucous membranes and may be given as intended to the patient undergoing tube feedings.

D. Soft gelatin capsules filled with liquid- The nurse may make an opening in the capsule and squeeze out contents for administration by feeding tube.

193. Medium -length nasoenteric tubes are used for:

A. Feeding- Placement of the tube must be verified prior to any feeding.

B. Decompression- A gastric sump and nasoenteric tube are used for gastrointestinal decompression.

C. Aspiration- Nasoenteric tubes are used for gastrointestinal aspiration.

D. Emptying- Gastric sump tubes are used to decompress the stomach and keep it empty.

194. Mercury is typically used in the placement of which of the following tubes?

A. Miller-Abbott- Most nasoenteric tubes use mercury to carry the tube by gravity to its desired location.

B. Gastric sump- A gastric sump is used to decompress and empty the stomach.

C. Dobbhoff- Dobbhoff tubes are used for enteric feeding.

D. EnterafloW- Enteraflow tubes are used for enteric feeding

195. The most significant nursing problem related to continuous tube feedings is

A. potential for aspiration- Because the normal swallowing mechanism is bypassed, consideration of the danger of aspiration must be foremost in the mind of the nurse caring for the patient receiving continuous tube feedings.

B. interruption of GI integrity- Tube feedings preserve GI integrity by intraluminal delivery of nutrients.

C. disturbance in the sequence of intestinal and hepatic metabolism-Tube feedings preserve the normal sequence of intestinal and hepatic metabolism.

D. interruption in fat metabolism and lipoprotein synthesis-Tube feedings maintain fat metabolism and lipoprotein synthesis.

196. When the nurse prepares to give a bolus tube feeding to the patient and determines that the residual gastric content is 150 cc, her best action is to

A. reassess the residual gastric content in 1 hour.-
If the gastric residual exceeds 100 cc 2 hours in a row, the physician should be notified.

B. notify the physician. -One observation of a residual gastric content over 100 cc does not have to be reported to the physician. If the observation occurs two times in succession, the physician should be notified.

C. give the tube feeding.- If the amount of gastric residual exceeds 100 cc, the tube feeding should be withheld at that time.

D. withhold the tube feeding indefinitely.- If the amount of gastric residual exceeds 100 cc, the tube feeding should be withheld at that time, but not indefinitely.

197. If tube feeding is continuous, the placement of the feeding tube should be checked

A. every shift.-
Each nurse caring for the patient is responsible for verifying that the tube is located in the proper area for continuous feeding.

B. every hour.- Checking for placement each hour is unnecessary unless the patient is extremely restless or there is basis for rechecking the tube based on other patient activities.

C. every 24 hours.- Checking for placement every 24 hours does not meet the standard of care due the patient receiving continuous tube feedings.

D. when a tube feeding is continuous, it is unnecessary to check placement.
Even though the feedings are continuous, the placement must be assessed.

198. Decrease in absorption of which of the following vitamins in the geriatric patient results in pernicious anemia?

A. B12- Vitamin B12 requires the intrinsic factor secreted by the gastric mucosa for absorption.

B. C- Lack of vitamin C may cause development of signs and symptoms of scurvy.

C. D-Vitamin D deficiency results in an inability to absorb calcium.

D. B6- Vitamin B6 affects neuromuscular function.

199. Which of the following terms refers to tarry, black stools?

A. Melena- Melena indicates blood in the stool.

B. Hemarthrosis- Hemarthrosis is bleeding into a joint.

C. Hematemesis- Hematemesis is vomiting blood.

D. Pyrosis- Pyrosis refers to heartburn.

200. Which of the following statements accurately reflects a rule of thumb upon which the nurse may rely in assessing the patient's fluid balance?

A. Minimal intake of 1.5 liters per day
- If food and fluids are withheld, IV fluids (3L/day) are usually prescribed.

B. Minimal urine output of 50 milliliters per hour- Minimal urine output may be less than 50 mL/hr.

C. Minimal urine output of 10 milliliters per hour- Minimal urine output must exceed 10 mL/hr.

D. Minimal intake of 2 liters per day- Minimal intake, as a rule of thumb, is less than 2 liters per day.

201.The nurse recognizes that the patient with a duodenal ulcer will likely experience

A. pain 2-3 hours after a meal.-The patient with a gastric ulcer often awakens between 1-2 A.M. with pain and ingestion of food brings relief.

B. vomiting.- Vomiting is uncommon in the patient with duodenal ulcer.

C. hemorrhage.- Hemorrhage is less likely in the patient with duodenal ulcer than the patient with gastric ulcer.

D. weight loss.- The patient with a duodenal ulcer may experience weight gain.

202. Of the following categories of medications, which is used in combination with bismuth salts to eradicate Helicobacter pylori?

A. Antibiotics
- Antibiotics and bismuth salts are given to eradicate H. pylori.

B. Antacids- Antacids are given to manage gastric acidity.

C. Proton pump inhibitors- Proton pump inhibitors are given to decrease acid secretion.

D. Histamine-2 receptor antagonists- Histamine-2 receptor antagonists are given to decrease the acid secretion in the stomach.

203. Which of the following medications represents the category proton (gastric acid) pump inhibitors?

A. Omeprazole (Prilosec)-
Omeprazole decreases gastric acid by slowing the hydrogen-potassium-adenosine-triphosphatase pump on the surface of the parietal cells.

B. Sucralfate (Carafate)- Sucralfate is a cytoprotective drug.

C. Famotidine (Pepcid)- Famotidine is a histamine-2 receptor antagonist.

D. Metronidazole (Flagyl)- Metronidazole is an antibiotic, specifically an amebicide.

204. Which of the following medications used for the treatment of obesity prevents the absorption of triglycerides?

A. Orlistat (Xenical)- Orlistat (Xenical) prevents the absorption of triglycerides. Side effects of Xenical may include increased bowel movements, gas with oily discharge, decreased food absorption, decreased bile flow, and decreased absorption of some vitamins.

B. bupropion hydrochloride (Wellbutrin)- buproprion hydrochloride (Wellbutrin) is an antidepressant medication.

C. Sibutramine hydrochoride (Meridia)- Sibutramine hydrochloride (Meridia) inhibits the reuptake of serotonin and norepinephine. Meridia decreases appetite.

D. Fluoxetine hydrochloride (Prozac)- Fluoxetine hydrochloride (Prozac) has not been approved by the FDA for use in the treatment of obesity.

205. Of the following bariatric surgical procedures, which is the best procedure for long-term weight loss?

A. Roux-en-Y- The Roux-en-Y gastric bypass is the recommended procedure for long-term weight loss. In this procedure, a horizontal row of staples creates a stomach pouch with a 1-cm stoma that is anastomosed with a portion of distal jejunum, creating a gastroenterostomy.

B. Vertical banded gastroplasty- In vertical banded gastroplasty, a double row of staples is applied vertically along the lesser curvature of the stomach, beginning at the angle of His.- Over time, the gastric restriction may fail.

C. Gastric ring application- Application of a silicone ring to the fundus of the stomach may fail.

D. Jejuno-ileal bypass- The first surgical procedure to treat morbid obesity was the jejuno-ileal bypass. This procedure, which resulted in significant complications, has been largely replaced by gastric restriction procedures

206. Which of the following statements regarding gastric cancer is accurate?

A. The incidence of cancer of the stomach continues to decrease in the United States.- While the incidence continues to decrease, gastric cancer still accounts for 12,800 deaths annually.

B. Most gastric cancer deaths occur in people younger than 40 years.- While gastric cancer deaths occasionally occur in younger people, most occur in people older than 40 years.

C. Females have a higher incidence of gastric cancers than males.- Males have a higher incidence of gastric cancers than females.

D. A diet high in smoked foods and low in fruits and vegetables may decrease the risk of gastric cancer. - More accurately, a diet high in smoked foods and low in fruits and vegetables may increase the risk of gastric cancer.


207. Which of the following categories of laxatives draw water into the intestines by osmosis?

A. Saline agents (milk of magnesia)-Saline agents use osmosis to stimulate peristalsis and act within 2 hours of consumption.

B. Bulk-forming agents (Metamucil)- Bulk-forming agents mix with intestinal fluids, swell, and stimulate peristalsis.

C. Stimulants (Dulcolax)- Stimulants irritate the colon epithelium.

D. Fecal softeners (Colace)- Fecal softeners hydrate the stool by surfactant action on the colonic epithelium, resulting in mixing of aqueous and fatty substances.

208. Crohn's disease is a condition of malabsorption caused by

A. inflammation of all layers of intestinal mucosa. -Crohn's disease is also known as regional enteritis and can occur anywhere along the GI tract, but most commonly at the distal ileum and in the colon.

B. infectious disease.- Infectious disease causes problems such as small bowel bacterial overgrowth leading to malabsorption.

C. disaccharidase deficiency.-Disaccharidase deficiency leads to lactose intolerance.

D. gastric resection.- Postoperative malabsorption occurs after gastric or intestinal resection.

209. The nurse teaches the patient whose surgery will result in a sigmoid colostomy that the feces expelled through the colostomy will be

A. solid.
- With a sigmoid colostomy, the feces are solid.

B. semi-mushy.- With a descending colostomy, the feces are semi-mushy.

C. mushy.- With a transverse colostomy, the feces are mushy.

D. fluid.- With an ascending colostomy, the feces are fluid.

210. When irrigating a colostomy, the nurse lubricates the catheter and gently inserts it into the stoma no more than _______ inches

A. 3”
- The nurse should insert the catheter no more than 3 inches.

B. . 2”- Insertion of the catheter 2 inches is inadequate.

C. 4”-NInsertion of the catheter 4 inches is excessive and not recommended.

D. 5”- Insertion of the catheter 5 inches is excessive and not recommended.

211. A longitudinal tear or ulceration in the lining of the anal canal is termed a (an)

A. anal fissure.- Fissures are usually caused by the trauma of passing a large, firm stool or from persistent tightening of the anal canal secondary to stress or anxiety (leading to constipation).

B. anorectal abscess.- An anorectal abscess is an infection in the pararectal spaces.

C. anal fistula.- An anal fistula is a tiny, tubular, fibrous tract that extends into the anal canal from an opening located beside the anus.

D. hemorrhoid.- A hemorrhoid is a dilated portion of vein in the anal canal.

212. Which type of diarrhea is caused by increased production and secretion of water and electrolyes by the intestinal mucosa into the intestinal lumen?

A. Secretory diarrhea-
Secretory diarrhea is usually high volume diarrhea and is caused by increased production and secretion of water and electrolytes by the intestinal mucosa into the intestinal lumen.

B. Osmotic diarrhea- Osmotic diarrhea occurs when water is pulled into the intestines by the osmotic pressure of nonabsorbed particles, slowing the reabsorption of water.

C. Mixed diarrhea- Mixed diarrhea is caused by increased peristalsis (usually from inflammatory bowel disease) and a combination of increased secretion or decreased absorption in the bowel.

D. Diarrheal disease- The most common cause of diarrheal disease is contaminated food.

213. Which of the following terms is used to refer to intestinal rumbling?

A. Borborygmus- Borborygmus is the term used to refer to intestinal rumbling which accompanies diarrhea.

B. Tenesmus- Tenesmus is the term used to refer to ineffectual straining at stool.

C. Azotorrhea- Azotorrhea is the term used to refer to excess of nitrogenous matter in the feces or urine.

D. Diverticulitis- Diverticulitis is the term used to refer to inflammation of a diverticulum from obstruction (by fecal matter) resulting in abscess formation.

214. The presence of mucus and pus in the stools suggests

A. Inflammatory colitis- The presence of mucus and pus in the stools suggests inflammatory colitis or enteritis.

B. Small bowel disease- Watery stools are characteristic of small bowel disease.

C. Disorders of the colon- Loose, semisolid stools are associated more often with disorders of the colon.

D. Intestinal malabsorption- Voluminous, greasy stools suggest intestinal malabsorption.


215. Celiac sprue is an example of which category of malabsorption?

A. Mucosal disorders causing generalized malabsorption.- In addition to celiac sprue, regional enteritis and radiation enteritis are examples of mucosal disorders.

B. Infectious diseases causing generalized malabsorption .-Examples of infectious diseases causing generalized malabsorption include small bowel bacterial overgrowth, tropical sprue, and Whipple's disease.

C. Luminal problems causing malabsorption .- Examples of luminal problems causing malabsorption include bile acid deficiency, Zollinger Ellison syndrome, and pancreatic insufficiency.

D. Postoperative malabsorption - Postoperative gastric or intestinal resection can result in development of malabsorption syndromes.

216. Typical signs and symptoms of appendicitis include:

A. Nausea- Nausea is typically associated with appendicitis with or without vomiting.

B. Left lower quadrant pain- Pain is generally felt in the right lower quadrant.

C. Pain when pressure is applied to the right lower quadrant of the abdomen.
Rebound tenderness, or pain felt with release of pressure applied to the abdomen, may be present with appendicitis.

D. High fever- Low-grade fever is associated with appendicitis.

217. Regional enteritis is characterized by:

A. Transmural thickening- Transmural thickeneing is an early pathologic change of Crohn's disease. Later pathology results in deep, penetrating granulomas.

B. Diffuse involvement- Regional enteritis is characterized by regional discontinuous lesions.

C. Severe diarrhea- Severe diarrhea is characteristic of ulcerative colitis while diarrhea in regional enteritis is less severe.

D. Exacerbations and remissions- Regional enteritis is characterized by a prolonged and variable course while ulcerative colitis is characterized by exacerbations and remissions.

218. What is the most common cause of small bowel obstruction?

A. Adhesions- Adhesions are scar tissue that forms as a result of inflammation and infection.

B. Hernias- Hernias are one of the second most common causes of small bowel obstruction.

C. Neoplasms- Neoplasms are one the second most common causes of small bowel obstruction.

D. Volvulus- Volvulus (twisting of the bowel) is a less common cause of small bowel obstruction.

219. Which of the follow statements provides accurate information regarding cancer of the colon and rectum?

A. Cancer of the colon and rectum is the second most common type of internal cancer in the United States. - Cancer of the colon and rectum is the second most common type of internal cancer in the United States.

B. Rectal cancer affects more than twice as many people as colon cancer. -Colon cancer affects more than twice as many people as does rectal cancer (94,700 for colon, 34,700 for rectum).

C. The incidence of colon and rectal cancer decreases with age.- The incidence increases with age (the incidence is highest in people older than 85).

D. There is no hereditary component to colon cancer.- Colon cancer occurrence is higher in people with a family history of colon cancer.

220. Which of the following characteristics are risk factors for colorectal cancer?

A. Familial polyposis- Family history of colon cancer or familial polyposis is a risk factor for colorectal cancer.

B. Age younger than 40- Being older than age 40 is a risk factor for colorectal cancer.

C. Low fat, low protein, high fiber diet- A high-fat, high-protein, low-fiber diet is a risk factor for colorectal cancer.

D. History of skin cancer- History of skin cancer is not a recognized risk factor for colorectal cancer.

221. Which type of jaundice in adults is the result of increased destruction of red blood cells?

A. Hemolytic- Hemolytic jaundice results because, although the liver is functioning normally, it cannot excrete the bilirubin as quickly as it is formed.

B. Hepatocellular

- Hepatocellular jaundice is the result of liver disease.

C. Obstructive- Obstructive jaundice is the result of liver disease.

D. Non-obstructive- Non-obstructive jaundice occurs with hepatitis.


222. The nurse places the patient after liver biopsy in which of the following positions?

A. On the right side-
In this position, the liver capsule at the site of penetration is compressed against the chest wall, and the escape of blood or bile through the perforation made for the biopsy is impeded.

B. On the left side- Positioning the patient on his left side is not indicated.

C. Trendelenburg- Positioning the patient in the Trendelenburg position may be indicated if the patient is in shock, but is not the position designed for the patient after liver biopsy.

D. High Fowler's- High Fowler's position is not indicated for the patient after liver biopsy.

223. Which of the following terms is used to describe a chronic liver disease in which scar tissue surrounds the portal areas?

A. Alcoholic cirrhosis- This type of cirrhosis is due to chronic alcoholism and is the most common type of cirrhosis.

B. Postnecrotic cirrhosis- In postnecrotic cirrhosis, there are broad bands of scar tissue, which are a late result of a previous acute viral hepatitis.

C. Biliary cirrhosis- In biliary cirrhosis, scarring occurs in the liver around the bile ducts.

D. Compensated cirrhosis- Compensated cirrhosis is a general term given to the state of liver disease in which the liver continues to be able to function effectively.

224. Which of the following terms describes the passage of a hollow instrument into a cavity for the withdrawal of fluid?

A. Paracentesis-
Paracentesis may be used to withdraw ascitic fluid if the fluid accumulation is causing cardiorespiratory compromise.

B. Astrerixis- Asterixis refers to involuntary flapping movements of the hands associated with metabolic liver dysfunction.

C. Ascites- Ascites refers to accumulation of serous fluid within the peritoneal cavity.

D. Dialysis- Dialysis refers to a form of filtration to separate crystalloid from colloid substances.

225. Which of the following terms most precisely refers to the incision of the common bile duct for removal of stones?

A. Choledocholithotomy-
Choledocholithotomy refers to incision of the common bile duct for the removal of stones (liths).

B. Cholecystostomy- Cholecystostomy refers to opening and drainage of the gallbladder.

C. Choledochotomy- Choledochotomy refers to opening into the common duct.

D. Choledochoduodenostomy- Choledochoduodenostomy refers to anastomosis of the common duct to the duodenum.

226. Which of the following clinical characteristics is associated with Type 1 diabetes (previously referred to as insulin-dependent diabetes mellitus [IDDM])?

A. Presence of islet cell antibodies. Individuals with Type 1 diabetes often have islet cell antibodies.

B. Obesity- Individuals with Type 1 diabetes are usually thin or demonstrate recent weight loss at the time of diagnosis

C. Rare ketosis- Individuals with Type 1 diabetes are ketosis-prone when insulin is absent.

D. Requirement for oral hypoglycemic agents.- Individuals with Type 1 diabetes need insulin to preserve life.

227. Which of the following clinical characteristics is associated with Type 2 diabetes (previously referred to as non-insulin-dependent diabetes mellitus [NIDDM])?

A. Can control blood glucose through diet and exercise
. Oral hypoglycemic agents may improve blood glucose levels if dietary modification and exercise are unsuccessful.

B. Usually thin at diagnosis- Individuals with Type 2 diabetes are usually obese at diagnosis.

C. Ketosis-prone- Individuals with Type 2 diabetes rarely demonstrate ketosis, except in stress or infection.

D. Demonstrate islet cell antibodies- Individuals with Type 2 diabetes do not demonstrate islet cell antibodies.

228. Of the following types of insulin, which is the most rapid acting?

A. Humalog- The onset of action of rapid-acting Humalog is within 10-15 minutes.

B. Regular- The onset of action of short-acting regular insulin is 30 minutes-1 hour.

C. NPH- The onset of action of intermediate acting NPH is 3-4 hours.

D. Ultralente- The onset of action of long-acting Ultralente is 6-8 hours.

229. Of the following categories of oral antidiabetic agents, which exert their primary action by directly stimulating the pancreas to secrete insulin?

A. Sulfonylureas-
Therefore, a functioning pancreas is necessary for sulfonylureas to be effective.

B. Thiazolidinediones- Thiazolidinediones enhance insulin action at the receptor site without increasing insulin secretion from the beta cells of the pancreas

C. Biguanides- Biguanides facilitate insulin's action on peripheral receptor sites.

D. Alpha glucosidase inhibitors- Alpha glucosidase inhibitors delay the absorption of glucose in the intestinal system, resulting in a lower postprandial blood glucose level.

230. The nurse teaches the patient about diabetes including which of the following statements?

A. Elevated blood glucose levels contribute to complications of diabetes, such as diminished vision.- When blood glucose levels are well controlled, the potential for complications of diabetes is reduced.

B. Sugar is found only in dessert foods.- Several types of foods contain sugar, including cereals, sauces, salad dressing, fruit, and fruit juices.

C. The only diet change needed in the treatment of diabetes is to stop eating sugar.
It is not feasible, nor is it advisable, to remove all sources of sugar from the diet.

D. Once insulin injections are started in the treatment of Type 2 diabetes, they can never be discontinued.- If the diabetes had been well controlled without insulin prior to the period of acute stress causing the need for insulin, the patient may be able to resume previous methods for control of diabetes when the stress is resolved.

231. The nurse teaches the patient about glargine (Lantus), a "peakless" basal insulin including which of the following statements?

A. Do not mix the drug with other insulins-
Because glargine is in a suspension with a pH of 4, it cannot be mixed with other insulins because this would cause precipitation. When administering glargine (Lantus) insulin it is very important to read the label carefully and to avoid mistaking Lantus insulin for Lente insulin and vice versa.

B. Administer the total daily dosage in two doses. - Glargine is absorbed very slowly over a 24-hour period and can be given once a day.

C. Draw up the drug first, then add regular insulin. - Because glargine is in a suspension with a pH of 4, it cannot be mixed with other insulins because this would cause precipitation.

D. The drug is rapidly absorbed and has a fast onset of action.- Glargine is a "peakless" basal insulin that is absorbed very slowly over a 24-hour period.

232. Which of the following disorders is characterized by a group of symptoms produced by an excess of free circulating cortisol from the adrenal cortex?

A. Cushing's syndrome-
The patient with Cushing's syndrome demonstrates truncal obesity, moon face, acne, abdominal striae, and hypertension.

B. Addison's disease- In Addison's disease, the patient experiences chronic adrenocortical insufficiency.

C. Graves' disease- In Graves' disease, the patient experiences hyperthyroidism.

D. Hashimoto's disease- The individual with Hashimoto's disease demonstrates inflammation of the thyroid gland, resulting in hypothyroidism.


233. Of the following disorders, which results from excessive secretion of somatotropin?

A. Acromegaly-
The patient with acromegaly demonstrates progressive enlargement of peripheral body parts, most commonly the face, head, hands, and feet.

B. Cretinism- Cretinism occurs as a result of congenital hypothyroidism.

C. Dwarfism- Dwarfism is caused by insufficient secretion of growth hormone during childhood.

D. Adrenogenital syndrome- Adrenogenital syndrome is the result of abnormal secretion of adrenocortical hormones, especially androgen.

234. Which of the following hormones is secreted by the posterior pituitary?

A. Vasopressin- Vasopressin causes contraction of smooth muscle, particularly blood vessels.

B. Calcitonin- Calcitonin is secreted by the parafollicular cells of the thyroid gland.

C. Corticosteroids- Corticosteroids are secreted by the adrenal cortex.

D. Somatostatin- Somatostatin is released by the anterior lobe of the pituitary.

235. Trousseau's sign is positive when

A. carpopedal spasm is induced by occluding the blood flow to the arm for 3 minutes with the use of a blood pressure cuff.- A positive Trousseau's sign is suggestive of latent tetany.

B. a sharp tapping over the facial nerve just in front of the parotid gland and anterior to the ear causes spasm or twitching of the mouth, nose, and eye.- A positive Chvostek's sign is demonstrated when a sharp tapping over the facial nerve just in front of the parotid gland and anterior to the ear causes spasm or twitching of the mouth, nose, and eye.

C. after making a clenched fist, the palm remains blanched when pressure is placed over the radial artery.- A positive Allen's test is demonstrated by the palm remaining blanched with the radial artery occluded. The radial artery should not be used for an arterial puncture.

D. The patient complains of pain in the calf when his foot is dorsiflexed.- A positive Homans' sign is demonstrated when the patient complains of pain in the calf when his foot is dorsiflexed.

236.The digestion of carbohydrates is aided by

A. amylase.- Amylase is secreted by the exocrine pancreas.

B. lipase.- Lipase aids in the digestion of fats.

C. trypsin.- Trypsin aids in the digestion of proteins.

D. secretin.- Secretin is the major stimulus for increased bicarbonate secretion from the pancreas.

237. The term used to describe total urine output of less than 400 mL in 24 hours is

oliguria.
Oliguria is associated with acute and chronic renal failure.

A. anuria.- Anuria is used to describe total urine output of less than 50 mL in 24 hours.

B. nocturia.- Nocturia refers to awakening at night to urinate.

C. dysuria.- Dysuria refers to painful or difficult urination.


238. When fluid intake is normal, the specific gravity of urine should be

A. 1.010-1.025.- Urine specific gravity is a measurement of the kidney's ability to concentrate urine.

B. 1.000.- The specific gravity of water is 1.000.

C. less than 1.010.- A urine specific gravity of less than 1.010 may indicate inadequate fluid intake.

D. greater than 1.025.- A urine specific gravity greater than 1.025 may indicate overhydration.

239. Of the following terms, which refers to casts in the urine?

A. Cylindruria- Casts may be identified through microscopic examination of the urine sediment after centrifuging.

B. Crystalluria- Crystalluria is the term used to refer to crystals in the urine.

C. Pyuria- Pyuria is the term used to refer to pus in the urine.

D. Bacteriuria- Bacteriuria refers to a bacterial count higher than 100,000 colonies/mL in the urine.

240. When the nurse observes the patient's urine to be orange, she further assesses the patient for

A. intake of medication such as phenytoin (Dilantin).-Urine that is orange may be caused by intake of Dilantin or other medications. Orange to amber colored urine may also indicate concentrated urine due to dehydration or fever.

B. bleeding.- Urine that is pink to red may indicate lower urinary tract bleeding.

C. intake of multiple vitamin preparations.- Urine that is bright yellow is an anticipated abnormal finding in the patient taking a multiple vitamin preparation.

D. infection.- Yellow to milky white urine may indicate infection, pyruria, or in the female patient, the use of vaginal creams.

241. To assess circulating oxygen levels the 2001 Kidney Disease Outcomes Quality Initiative: Management of Anemia Guidelines recommends the use of which of the following diagnostic tests?

A. Hemoglobin- Although hematocrit has always been the blood test of choice to assess for anemia, the 2001 Kidney Disease Outcomes Quality Initiative: Management of Anemia Guidelines, recommend that anemia be quantified using hemoglobin rather than hematocrit measurements, as it is more accurate in assessment of circulating oxygen.

B. Hematocrit- Hemoglobin is recommended as it is more accurate in the assessment of circulating oxygen than hematocrit.

C. Serum iron levels- Serum iron levels measure iron storage in the body.

D. Arterial blood gases- Arterial blood gases assess the adequacy of oxygenation, ventilation, and acid-base status.

242. Which of the following types of incontinence refers to involuntary loss of urine through an intact urethra as a result of a sudden increase in intra-abdominal pressure?

A. Stress-
Stress incontinence may occur with sneezing and coughing.

B. Overflow- Overflow incontinence refers to the involuntary loss of urine associated with overdistention of the bladder.

C. Urge- Urge incontinence refers to involuntary loss of urine associated with urgency.

D. Reflex- Reflex incontinence refers to the involuntary loss of urine due to involuntary urethral relaxation in the absence of normal sensations.

243. To facilitate entry of a catheter into the male urethra, the penis should be positioned at which of the following degree angles (in relation to the body)?

A. 90 degrees- A right angle straightens the urethra and makes it easier to insert the catheter.

B. 45 degrees- A 45-degree angle will not straighten the urethra.

C. 180 degrees- A 180-degree angle will result in the penis being parallel to the body and inappropriately positioned for catheterization.

D. 270 degrees- A 270-degree angle is a physical impossibility.

244. In assessing the appropriateness of removing a suprapubic catheter, the nurse recognizes that the patient's residual urine must be less than which of the following amounts on two separate occasions (morning and evening)?

A. 100 cc- If the patient complains of discomfort or pain, however, the suprapubic catheter is usually left in place until the patient can void successfully.

B. 30 cc- Residual urine may be greater than 30 cc and still allow discontinuance of a suprapubic catheter.

C. 50 cc- Residual urine may be greater than 50 cc and still allow discontinuance of a suprapubic catheter.

D. 400 cc- Residual urine that is greater than 100 cc indicates that the suprapubic catheter cannot be discontinued.

245. When providing care to the patient with bilateral nephrostomy tubes, the nurse never does which of the following?

A. Clamps each nephrostomy tube when the patient is moved.-The nurse must never clamp a nephrostomy tube because it could cause obstruction and resultant pyelonephritis.

B. Reports a dislodged nephrostomy tube immediately
A dislodged nephrostomy tube must be reported immediately to allow the surgeon to replace the tube immediately to prevent the opening from contracting.

C. Measures urine output from each tube separately
The output from each tube is assessed, indicating the functioning of the tube.

D. Irrigates each nephrostomy tube with 30 cc of normal saline q8h as ordered
The nurse may irrigate a nephrostomy tube with specific orders to do so.

246. Which type of medication may be used in the treatment of a patient with incontinence to inhibit contraction of the bladder?

A. Anticholinergic agent
- Anticholinergic agents are considered first-line medications for urge incontinence.

B. Estrogen hormone- Estrogen decreases obstruction to urine flow by restoring the mucosal, vascular, and muscular integrity of the urethra.

C. Tricyclic antidepressants- Tricyclic antidepressants decrease bladder contractions as well as increase bladder neck resistance.

D. Over-the-counter decongestant- Stress incontinence may be treated using pseudoephedrine and phenylpropanolamine, ingredients found in over-the-counter decongestants.

247. Which of the following is a reversible cause of urinary incontinence in the older adult?

A. Constipation.
- Constipation is a reversible cause of urinary incontinence in the older adult. Other reversible causes include acute urinary tract infection, infection elsewhere in the body, decreased fluid intake, a change in a chronic disease pattern, and decreased estrogen levels in the menopausal women.

B. Increased fluid intake- A decreased fluid intake, rather than increased fluid intake, is a reversible cause of urinary incontinence in the older adult.

C. Age- Age is a risk factor for urinary incontinence, not a reversible cause.

D. Decreased progesterone level in the menopausal woman. - A decreased estrogen, not progresterone, level in the menopausal woman is a reversible cause of urinary incontinence in the older woman.

248. Bladder retraining following removal of an indwelling catheter begins with
instructing the patient to follow a 2-3 hour timed voiding schedule.
Immediately after the removal of the indwelling catheter, the patient is placed on a timed voiding schedule, usually two to three hours. At the given time interval, the patient is instructed to void.

A. encouraging the patient to void immediately.-
Immediate voiding is not usually encouraged. The patient is commonly placed on a timed voiding schedule, usually within two to three hours.

B. advising the patient to avoid urinating for at least 6 hours.- Immediately after the removal of the indwelling catheter, the patient is placed on a timed voiding schedule, usually two to three hours, not six.

C. performing straight catherization after 4 hours.- If bladder ultrasound scanning shows 100 mL or more of urine remaining in the bladder after voiding, straight catheterization may be performed for complete bladder emptying.

249. Which of the following terms is used to refer to inflammation of the renal pelvis?

A. Pyelonephritis- Pyelonephritis is an upper urinary tract inflammation, which may be acute or chronic.

B. Cystitis- Cystitis is inflammation of the urinary bladder.

C. Urethritis- Urethritis is inflammation of the urethra.

D. Interstitial nephritis- Interstitial nephritis is inflammation of the kidney.

250. If an indwelling catheter is necessary, nursing interventions that should be implemented to prevent infection include

A. performing meticulous perineal care daily with soap and water.- Cleanliness of the area will reduce potential for infection.

B. using clean technique during insertion.- Strict aseptic technique must be used during insertion of a urinary bladder catheter.

C. using sterile technique to disconnect the catheter from tubing to obtain urine specimens.- The nurse must maintain a closed system and use the catheter's port to obtain specimens.

D. placing the catheter bag on the patient's abdomen when moving the patient.- The catheter bag must never be placed on the patient's abdomen unless it is clamped because it may cause backflow of urine from the tubing into the bladder.

251. The nurse who provides teaching to the female patient regarding prevention of recurrent urinary tract infections includes which of the following statements?

A. Void immediately after sexual intercourse.-
Voiding will serve to flush the urethra, expelling contaminants.

B. Take tub baths instead of showers.- Showers are encouraged rather than tub baths because bacteria in the bath water may enter the urethra.

C. Increase intake of coffee, tea, and colas.- Coffee, tea, colas, alcohol, and other fluids that are urinary tract irritants should be avoided.

D. Void every 5 hours during the day.- The patient should be encouraged to void every 2-3 hours during the day and completely empty the bladder.

252. A history of infection specifically caused by group A beta-hemolytic streptococci is associated with which of the following disorders?

A. Acute glomerulonephritis
- Acute glomerulonephritis is also associated with varicella zoster virus, hepatitis B, and Epstein-Barr virus.

B. Acute renal failure- Acute renal failure is associated with hypoperfusion to the kidney, parenchymal damage to the glomeruli or tubules, and obstruction at a point distal to the kidney.

C. Chronic renal failure- Chronic renal failure may be caused by systemic disease, hereditary lesions, medications, toxic agents, infections, and medications.

D. Nephrotic syndrome- Nephrotic syndrome is caused by disorders such as chronic glomerulonephritis, systemic lupus erythematosus, multiple myeloma, and renal vein thrombosis.

253. Rejection of a transplanted kidney within 24 hours after transplant is termed

A. hyperacute rejection.- Hyperacute rejection may require removal of the transplanted kidney.

B. acute rejection.- Acute rejection occurs within 3-14 days of transplantation.

C. chronic rejection.- Chronic rejection occurs after many years.

D. simple rejection.- The term simple is not used in the categorization of types of rejection of kidney transplants.

254. When caring for a patient with an uncomplicated, mild urinary tract infection (UTI), the nurse knows that recent studies have shown which of the following drugs to be a good choice for short-course (e.g. 3-day) therapy?

A. Levofloxacin (Levaquin)- Levofloxacin, a floroquinolone, is a good choice for short-course therapy of uncomplicated, mild to moderate UTI. Clinical trial data show high patient compliance with the 3-day regimen (95.6%) and a high eradication rate for all pathogens (96.4%).

B. Trimethoprim sulfamethoxazole (TMP-SMZ, Bactrim, Septra) -Trimethoprim sulfamethoxazole is a commonly used medication for treatment of a complicated UTI, such as pyelonephritis.

C. Nitrofurantoin (Macrodantin, Furadantin) - Nitrofurantoin is a commonly used medication for treatment of a complicated UTI, such as pyelonephritis.

D. Ciprofloxacin (Cipro) - Ciprofloxacin is a good choice for treatment of a complicated UTI. Recent studies have found ciprofloxacin to be significantly more effective than TMP-SMX in community-based patients and in nursing home residents.


255. Which of the following terms refers to difficult or painful sexual intercourse?

A. Dyspareunia- Dyspareunia is a common problem of the aged female.
Amenorrhea


B. Amenorrhea refers to absence of menstrual flow. -Dysmenorrhea
Dysmenorrhea refers to painful menstruation.


C. Endometriosis- Endometriosis is a condition in which endometrial tissue seeds in other areas of the pelvis.


256. The opening into the vagina on the perineum is termed the

A. introitus.-The introitus is the vaginal orifice.

B. adnexa.- Adnexa is a term used to describe the fallopian tubes and ovaries together.

C. cervix.- The cervix is the bottom (interior) part of the uterus that is located in the vagina.


D. hymen.- The hymen is a tissue that may cover the vaginal opening partially or completely before vaginal penetration.

257. Which of the following hormones is primarily responsible for stimulating the production of progesterone?

A. Luteinizing hormone-
Luteinizing hormone is released by the pituitary gland.


B. Follicle-stimulating hormone- Follicle-stimulating hormone is responsible for stimulating the ovaries to secrete estrogen.


C. Estrogen- Estrogens are responsible for developing and maintaining the female reproductive tract.


D. Androgen- Androgens, secreted in small amounts by the ovaries, are involved in early development of the follicle and also affect the female libido.

258. When the results of a Pap smear are reported as class 5, the nurse recognizes that the common interpretation is

A. malignant.-
A class 5 Pap smear, according to the Bethesda Classification, indicates squamous cell carcinoma.

B. normal.- A class 1 Pap smear is interpreted as normal.
probably normal.


C. A class 2 Pap smear is interpreted as probably normal. - suspicious.

D. A class 3 Pap smear is interpreted as suspicious.

259. For women aged 19-39 years, recommended health screening diagnostic testing includes which of the following?

A. Pap smear-
A Pap smear is recommended for women aged 19-39 years, as well as for women aged 40 and older.

B. Mammography- Mammography is recommended for health screening for women aged 40 years and older.

C. Cholesterol and lipid profile- Cholesterol and lipid profile is recommended for women aged 40 years and older.

D. Bone mineral density testing- Bone mineral density testing is recommended for women aged 40 years and older.


260. Which of the following statements reflects nursing care of the woman with mild to moderate ovarian hyperstimulation syndrome (OHSS)?

A. Advise the patient to decrease her activity, monitor her urine output and to return for frequent office visits. -Management in mild and moderate cases of OHSS consists of decreased activity, monitoring of urine output and frequent office visits as designated by the reproductive endocrinologist.

B. Advise the patient to measure her weight and abdominal circumference daily. -Treatment of severe, not mild to moderate, OHSS includes daily measurements of weight and abdominal circumference.

C. Advise the patient to monitor her heart rate and to report if her pulse falls below 60 beats per minute. - Symptoms of OHSS include abdominal discomfort, distention, weight gain and ovarian enlargement.

D. Prepare the patient for immediate hospitalization. -The patient with severe OHSS is hospitalized for monitoring and treatment.


261. Which of the following terms is used to describe a procedure in which cervical tissue is removed as result of detection of abnormal cells?

A. Conization- The procedure is also called a cone biopsy.

B. Colporrhaphy- Colporrhaphy refers to repair of the vagina.

C. Cryotherapy- Cryotherapy refers to destruction of tissue by freezing.

D. Perineorrhaphy- Perineorrhaphy refers to sutural repair of perineal lacerations.


262. Of the following terms, which is used to refer to a type of gestational trophoblastic neoplasm?

A. Hydatidiform mole-
Hydatidiform mole occurs in 1 in 1000 pregnancies.

B. Dermoid cyst- A dermoid cyst is an ovarian tumor of undefined origin that consists of undifferentiated embryonal cells.

C. Doderlein's bacilli- Doderlein's bacilli is one component of normal vaginal flora.

D. Bartholin's cyst- Bartholin's cyst is a cyst in a paired vestibular gland in the vulva.

263. When the female client reports a frothy yellow-brown vaginal discharge, the nurse suspects the client has a vaginal infection caused by

A. trichomonas vaginalis.- Trichomonas vaginalis causes a frothy yellow-white or yellow-brown vaginal discharge.

B. candida albicans.- Candidiasis causes a white, cheeselike discharge clinging to the vaginal epithelium.

C. gardnerella vaginalis.- Gardnerella vaginalis causes a gray-white to yellow-white discharge clinging to the external vulva and vaginal walls.

D. chlamydia.- Chlamydia causes a profuse purulent discharge.

264. The nurse providing education regarding sexually transmitted diseases includes which of the following statements regarding herpes virus 2 (herpes genitalis)?

A. In pregnant women with active herpes virus, babies delivered vaginally may become infected with the virus.-
Therefore, a cesarean delivery may be performed if the virus recurs near the time of delivery.

B. Transmission of the virus requires sexual contact.- Asexual transmission by contact with wet surfaces or self-transmission (i.e., touching a cold sore and then touching the genital area) can occur.


C. Transmission occurs only when the carrier has symptoms.-Transmission is possible even when the carrier does not have symptoms.

D. The virus is very difficult to kill.- Usually, the virus is killed at room temperature by drying.

265. An opening between the bladder and the vagina is called a

A. vesicovaginal fistula.- A vesicovaginal fistula may occur because of tissue injury sustained during surgery, vaginal delivery, or a disease process.

B. cystocele.- A cystocele is a downward displacement of the bladder toward the vaginal orifice.

C. rectocele.- A rectocele is a bulging of the rectum into the vagina.

D. rectovaginal fistula.- A rectovaginal fistula is an opening between the rectum and the vagina.

266. Which of the following statements defines laparoscopic myomectomy—an alternative to hysterectomy for the treatment of excessive bleeding due to fibroids?

A. Removal of fibroids through a laparoscope inserted through a small abdominal incision. - Laparoscopic myomectomy is the removal of fibroids through a laparoscope inserted through a small abdominal incision.

B. Cauterization and shrinking of fibroids using a laser or electrical needle Laparoscopic myolysis is the procedure in which a laser or electrial needles are used to cauterize and shrink the fibroid.

C. Coagulation of the fibroids using electrical current. Laparoscopic cryomyolysis is the procedure in which electric current is used to coagulate the fibroids.

D. Resection of the fibroids using a laser through a hyserscope passed through the cervix. Hysteroscopic resection of myomas is the procedure in which a laser is used through a hyserscope passed through the cervix; no incision or overnight stay is needed.

267. Stage 3 of breast development, according to Tanner, occurs when

A. the areola (a darker tissue ring around the nipple) develops.- Stage 3 also involves further enlargement of breast tissue.

B. breast budding begins.- Breast budding is the first sign of puberty in a female.

C. the areola and nipple form a secondary mound on top of breast tissue.
In stage 4, the nipple and areola form a secondary mound on top of breast tissue.

D. the breast develops into a single contour- In stage 5, the female demonstrates continued development of a larger breast with a single contour.

268. When the female patient demonstrates thickening, scaling, and erosion of the nipple and areola, the nurse recognizes that the patient is exhibiting signs of

A. Paget's disease.-
Paget's disease is a malignancy of mammary ducts with early signs of erythema of nipple and areola.

B..acute mastitis.- Acute mastitis is demonstrated by nipple cracks or abrasions along with reddened and warm breast skin and tenderness.

C. fibroadenoma.- Fibroadenoma is characterized as the occurrence of a single, nontender mass that is firm, mobile, and not fixed to breast tissue or chest wall.

D. peau d'orange (edema).- Peau d'orange is associated with the breast and demonstrates an orange peel apearance of breast skin with enlargement of skin pores.


269. The nurse teaches the female patient who is premenopausal to perform breast self-examination (BSE)

A. on day 5 to day 7, counting the first day of menses as day 1.-BSE is best performed after menses, when less fluid is retained.

B. with the onset of menstruation - Because most women notice increased tenderness, lumpiness, and fluid retention before their menstrual period, BSE is not recommended with the onset of menses.

C. on day 2 to day 4, counting the first day of menses as day 1.- Because the tenderness, lumpiness, and fluid retention problems noticed by women in relation to onset of menses generally continue through menses, BSE is not recommended during that time.

D. any time during the month.- Because most women notice increased tenderness, lumpiness, and fluid retention before their menstrual period, BSE is best performed when the time for menses is taken into account.

270. Which type of biopsy is used for nonpalpable lesions found on mammography?

A. Stereotactic- Stereotactic biopsy utilizes computer location of the suspicious area found on biopsy, followed by core needle insertion and sampling of tissue for pathologic examination.

B. Excisional- Excisional biopsy is the usual procedure for any palpable breast mass.

C. Incisional- Incisional biopsy is performed on a palpable mass when tissue sampling alone is required.
D. Tru-Cut core- Tru-Cut core biopsy is used when a tumor is relatively large and close to the skin surface.

271. The nurse recognizes which of the following statements as accurately reflecting a risk factor for breast cancer?

A. Mother affected by cancer before 60 years of age- Risk for breast cancer increases twofold if first-degree female relatives (sister, mother, or daughter) had breast cancer.

B. Onset of menses before 14 years of age- Increased risk is associated with early menarche (i.e., menses beginning before 12 years of age).

C. Multiparity- Nulliparity and later maternal age for first birth are associated with increased risk for breast cancer.

D. No alcohol consumption- Alcohol use remains controversial; however, a slightly increased risk is found in women who consume even one drink daily and doubles among women drinking three drinks daily.

272. Which of the following terms is used to describe removal of the breast tissue and an axillary lymph node dissection leaving muscular structure intact as surgical treatment of breast cancer?

A. Modified radical mastectomy-
A modified radical mastectomy leaves the pectoralis major and minor muscles intact.

B. Segmental mastectomy- In a segmental mastectomy, varying amounts of breast tissue are removed, including the malignant tissue and some surrounding tissue to ensure clear margins.

C. Total mastectomy- In a total mastectomy, breast tissue only is removed.

D. Radical mastectomy- Radical mastectomy includes removal of the pectoralis major and minor muscles in addition to breast tissue and axillary lymph node dissection.

273. Ductal lavage is used for

A. women at higher risk for benign proliferative breast disease. - Performed in the doctor's office, a microcatheter is inserted through the nipple while instilling saline and retrieving the fluid for analysis. It has been shown to identify atypical cells in this population and has been found to be adept at detecting cellular changes within the breast tissue.

B. women at low risk for breast cancer.- Ductal lavage is used for women at higher risk, not low risk, for benign proliferative breast disease.

C. screening women over age 65.- Ductal lavage is used for women at higher risk for benign proliferative breast disease; it is not used as a screening tool.

D. women with breast implants.- Ductal lavage is used for women at higher risk for benign proliferative breast disease; it is not specific for women with breast implants.

274. The 2000 NIH Consensus Development Conference Statement states that what percentage of women with invasive breast cancer should consider the option of systemic chemotherapy, not just women whose tumors are greater than 1cm in size?

A. 100% (all)- The 2000 Consensus Development Conference Statement states that all women with invasive breast cancer should consider the option of systemic chemotherapy, not just women whose tumors are greater than 1 cm in size.

B. 75%- All women (100%) with invasive breast cancer should consider the option of systemic chemotherapy, not just women whose tumors are greater than 1 cm in size.

C. 50%- All women (100%) with invasive breast cancer should consider the option of systemic chemotherapy, not just women whose tumors are greater than 1 cm in size.

D. C25%- All women (100%) with invasive breast cancer should consider the option of systemic chemotherapy, not just women whose tumors are greater than 1 cm in size.

275. Which of the following terms refers to surgical removal of one of the testes?

A. Orchiectomy- Orchiectomy is required when the testicle has been damaged.

B. Circumcision- Circumcision is excision of the foreskin, or prepuce, of the glans penis.

C. Vasectomy- Vasectomy is the ligation and transection of part of the vas deferens to prevent the passage of the sperm from the testes.

D. Hydrocelectomy- Hydrocelectomy describes the surgical repair of a hydrocele, a collection of fluid in the tunica vaginalis.

276. The term or disease associated with buildup of fibrous plaques in the sheath of the corpus cavernosum causing curvature of the penis when it is erect is known as

A. Peyronie's disease.-
Peyronie's disease may require surgical removal of the plaques when the disease makes sexual intercourse painful, difficult, or impossible.

B. Bowen's disease- Bowen's disease refers to a form of squamous cell carcinoma in situ of the penile shaft.

C. phimosis.- Phimosis refers to the condition in which the foreskin is constricted so that it cannot be retracted over the glans.

D. priapism.- Priapism refers to an uncontrolled, persistent erection of the penis occurring from either neural or vascular causes.

277. Which of the following terms is used to describe the opening of the urethra on the dorsum of the penis?

A. Epispadias
- Epispadias is a congenital anomaly in which the urethral opening is on the dorsum of the penis and is usually repaired through plastic surgery when the boy is very young.

B. Hypospadias- Hypospadias is a congenital anomaly in which the urethral opening is on the underside of the penis and is usually repaired through plastic surgery when the boy is very young.

C. Urethral stricture- Urethral stricture is a condition in which a section of urethra is narrowed.

D. Urethritis- Urethritis refers to inflammation of the urethra and is commonly associated with sexually transmitted disease.

278. The nurse teaches the patient who has been prescribed Viagra which of the following guidelines?

A. Do not take more than one tablet per day of your prescribed dose.-Taking Viagra more than once a day will not improve its effects and the patient may experience back and leg aches as well as nausea and vomiting.

B. Viagra should be taken immediately before intercourse.- Viagra should be taken one hour before intercourse.

C. Viagra will result in erection formation.- Viagra will not create the erection; the erection must be created by sexual stimulation.

D. Viagra will restore sex drive.- Viagra will not restore desire or sex drive.

279. The obstructive and irritative symptom complex caused by benign prostatic hypertrophy is termed

A. prostatism.- Symptoms of prostatism include increased frequency of urination, nocturia, urgency, dribbling, and a sensation that the bladder has not completely emptied.

B. prostatitis.- Prostatitis is an inflammation of the prostate gland.

C. prostaglandin.- Prostaglandins are physiologically active substances present in tissues with vasodilator properties.

D. prostatectomy.- Prostatectomy refers to the surgical removal of the prostate gland.

280. Proteins formed when cells are exposed to viral or foreign agents that are capable of activating other components of the immune system are referred to as

A. interferons.- Interferons are biologic response modifiers with nonspecific viricidal proteins.

B. antibodies.- Antibodies are protein substances developed by the body in response to and interacting with a specific foreign substance.

C. antigens.- Antigens are substances that induce formation of antibodies.

D. complements.- Complement refers to a series of enzymatic proteins in the serum that, when activated, destroy bacteria and other cells.


281. Cytotoxic T cells

A. lyse cells infected with virus. -Cytotoxic T cells play a role in graft rejection.

B. are important in producing circulating antibodies.- B cells are lymphocytes important in producing circulating antibodies.

C. attack foreign invaders (antigens) directly. -Helper T cells are lymphocytes that attack antigens directly.

D. decrease B cell activity to a level at which the immune system is compatible with life.- Suppressor T cells are lymphocytes that decrease B-cell activity to a level at which the immune system is compatible with life.

282. During which stage of the immune response does the circulating lymphocyte containing the antigenic message return to the nearest lymph node?

A. Proliferation- Once in the node, the sensitized lymphocyte stimulates some of the resident dormant T and B lymphocytes to enlarge, divide, and proliferate.

B. Recognition- In the recognition stage, the immune system distinguishes an invader as foreign, or non-self.

C. Response- In the response stage, the changed lymphocytes function either in a humoral or cellular fashion.

D. Effector- In the effector stage, either the antibody of the humoral response or the cytotoxic T cell of the cellular response reaches and couples with the antigen on the surface of the foreign invader.

283. Which of the following responses identifies a role of T lymphocytes?

A. Transplant rejection- Transplant rejection and graft-versus-host disease are cellular response roles of T cells.

B. Anaphylaxis- Anaphylaxis is a humoral response role of B-lymphocytes.

C. Allergic hay fever and asthma- Allergic hay fever and asthma are humoral response roles of B-lymphocytes.

D. Bacterial phagocytosis and lysis- Bacterial phagocytosis and lysis are humoral response roles of B-lymphocytes.

284. Of the following classifications of medications, which is known to inhibit prostaglandin synthesis or release?

A. Nonsteroidal anti-inflammatory drugs (NSAIDs) in large doses. NSAIDs include aspirin and ibuprofen.

B. Antibiotics (in large doses)- Antibiotics in large doses are known to cause bone marrow suppression.

C. Adrenal corticosteroids- Adrenal corticosteroids are known to cause immunosuppression.

D. Antineoplastic agents- Antineoplastic agents are known to cause immunosuppression.

285. Which of the following statements reflect current stem cell research?

A. The stem cell is known as a precursor cell that continually replenishes the body's entire supply of both red and white cells. - The stem cell is known as a precursor cell that continually replenishes the body's entire supply of both red and white cells. Stem cells comprise only a small portion of all types of bone marrow cells.

B. Stem cell transplantation can restore immune system functioning. - Research conducted with mouse models has demonstrated that once the immune system has been destroyed experimentally, it can be completely restored with the implantation of just a few purified stem cells.

C. Stem cell transplantion has been performed in the laboratory only. - Stem cell transplantation has been carried out in human subjects with certain types of immune dysfunction such as severe combined immunodeficiency (SCID).

D. Clinical trials are underway in patients with acquired immune deficiencies only.
Clinical trails are underway in patients with a variety of disorders with an autoimmune component including systemic lupus erythematosus, rheumatoid arthritis, scleroderma, and multiple sclerosis.

286. The nurse's base knowledge of primary immunodeficiencies includes which of the following statements? Primary immunodeficiencies

A. develop early in life after protection from maternal antibodies decreases.
These disorders may involve one or more components of the immune system.

B. B. occur most commonly in the aged population.- Primary immunodeficiencies are seen primarily in infants and young children.

C. develop as a result of treatment with antineoplastic agents.- Primary immunodeficiencies are rare disorders with genetic origins.

D. disappear with age.- Without treatment, infants and children with these disorders seldom survive to adulthood.

287. Agammaglobulinemia is also known as

A. Bruton's disease.- Bruton's disease is a sex-linked disease that results in infants born with the disorder suffering severe infections soon after birth

B. Nezelof syndrome.- Nezelof syndrome is a disorder involving lack of a thymus gland.

C. Wiskott-Aldrich syndrome.- Wiskott-Aldrich syndrome involves the absence of T cells and B cells and the presence of thrombocytopenia.

D. Common variable immunodeficiency (CVID)- CVID is another term for hypogammaglobulinemia.

288. When the nurse administers intravenous gamma-globulin infusion, she recognizes that which of the following complaints, if reported by the patient, may indicate an adverse effect of the infusion?

A. Tightness in the chest-
Flank pain, tightness in the chest, or hypotension indicates adverse effects of gamma-globulin infusion.

B. Nasal stuffiness- Nasal stuffiness is not recognized as an adverse effect of gamma-globulin infusion.

C. Increased thirst- Increased thirst is not recognized as an adverse effect of gamma-globulin infusion.

D. Burning urination- Burning urination is a sign of urinary tract infection, not an adverse effect of gamma-globulin infusion.

289. Ataxia is the term that refers to

A. uncoordinated muscle movement.- Ataxia-telangiectasia is an autosomal recessive disorder affecting both T-cell and B-cell immunity.

B. vascular lesions caused by dilated blood vessels.- Telangiectasia is the term that refers to vascular lesions caused by dilated blood vessels.

C. inability to understand the spoken word.- Receptive aphasia is an inability to understand the spoken word.

D. difficulty swallowing.- Dysphagia refers to difficulty swallowing.


290. Which of the following microorganisms is known to cause retinitis in people with HIV/AIDS?

A. Cytomegalovirus-
Cytomegalovirus is a species-specific herpes virus.

B. Cryptococcus neoformans- Cryptococcus neoformans is a fungus that causes an opportunistic infection in patients with HIV/AIDS.

C. Mycobacterium avium- Mycobacterium avium is an acid-fast bacillus that commonly causes a respiratory illness.

D. a fungus based on its structure.

291. Of the following blood tests, which confirms the presence of antibodies to HIV?

A. Enzyme-linked immunoabsorbant assay (ELISA). ELISA, as well as Western blot assay, identifies and confirms the presence of antibodies to HIV.

B. Erythrocyte sedimentation rate (ESR)- The ESR is an indicator of the presence of inflammation in the body.

C. p24 antigen- The p24 antigen is a blood test that measures viral core protein.

D. Reverse transcriptase- Reverse transcriptase is not a blood test. Rather, it is an enzyme that transforms single-stranded RNA into a double-stranded DNA.

292. When assisting the patient to interpret a negative HIV test result, the nurse informs the patient that the results mean

A. his body has not produced antibodies to the AIDS virus. A negative test result indicates that antibodies to the AIDS virus are not present in the blood at the time the blood sample for the test is drawn.

B. he has not been infected with HIV. A negative test result should be interpreted as demonstrating that if infected, the body has not produced antibodies (which take from 3 weeks to 6 months or longer). Therefore, subsequent testing of an at-risk patient must be encouraged.

C. he is immune to the AIDS virus. The test result does not mean that the patient is immune to the virus, nor does it mean that the patient is not infected. It just means that the body may not have produced antibodies yet.

D. antibodies to the AIDS virus are in his blood. When antibodies to the AIDS virus are detected in the blood, the test is interpreted as positive.

293. Which of the following substances may be used to lubricate a condom?

A. K-Y jelly- K-Y jelly is water-based and will provide lubrication while not damaging the condom.

B. Skin lotion- The oil in skin lotion will cause the condom to break.

C. Baby oil- Baby oil will cause the condom to break.

D. Petroleum jelly- The oil in petroleum jelly will cause the condom to break.

294. More than 500 CD4+ T lymphocytes/mm3 indicates which stage of HIV infection?

A. CDC category A - HIV asymptomatic. - More than 500 CD4+ T lymphocytes/mm3 indicates CDC category A - HIV asymptomatic.

B. Primary infection (acute HIV infection or acute HIV syndrome).-The period from infection with HIV to the development of antibodies to HIV is know as primary infection.

C. CDC category B - HIV symptomatic- 200-499 CD4+ T lymphocytes/mm3 indicates CDC category B - HIV symptomatic.

D. CDC category C – AIDS- Less than 200 CD4+ T lymphocytes/mm3 indicates CDC category C - AIDS.

295. The term used to define the balance between the amount of HIV in the body and the immune response is

A. viral set point- The viral set point is the balance between the amount of HIV in the body and the immune response.

B. window period- During the primary infection period, the window period occurs since a person is infected with HIV but negative on the HIV antibody blood test.

C. primary infection stage. The period from infection with HIV to the development of antibodies to HIV is known as the primary infection stage.

D. viral clearance rate- The amount of virus in circulation and the number of infected cells equals the rate of viral clearance.

296. Which of the following statements reflect the treatment of HIV infection?

A. Treatment of HIV infection for an individual patient is based on the clinical condition of the patient, CD4 T cell count level, and HIV RNA (viral load)

Although specific therapies vary, treatment of HIV infection for an individual patient is based on three factors: the clinical condition of the patient, CD4 T cell count level, and HIV RNA (viral load).

B.Treatment should be offered to all patients once they reach CDC category B - HIV symptomatic.- Treatment should be offered to all patients with the primary infection (acute HIV syndrome).

C. Treatment should be offered to only selected patients once they reach CDC category B - HIV symptomatic.- Treatment should be offered to all patients with the primary infection (acute HIV syndrome).

D. Treatment should be offered to individuals with plasma HIV RNA levels less than 55,000 copies/mL (RT-PCR assay.) - In general, treatment should be offered to individuals with fewer than 350 CD4+ T cells/mm3 or plasma HIV RNA levels exceeding 55,000 copies/mL (RT-PCR assay).

297. Which of the following body substances causes increased gastric secretion, dilation of capillaries, and constriction of the bronchial smooth muscle?

A. Histamine-
When cells are damaged, histamine is released.

B. Bradykinin- Bradykinin is a polypeptide that stimulates nerve fibers and causes pain.

C. Serotonin- Serotonin is a chemical mediator that acts as a potent vasoconstrictor and bronchoconstrictor.

D. Prostaglandin- Prostaglandins are unsaturated fatty acids that have a wide assortment of biologic activity.

298. Which type of hypersensitivity reaction involves immune complexes formed when antigens bind to antibodies?

A. Type III-
Type III hypersensitivity is associated with systemic lupus erythematosus, rheumatoid arthritis, serum sickness, certain types of nephritis, and some types of bacterial endocarditis.

B. Type I- Type I or anaphylactic hypersensitivity is an immediate reaction, beginning within minutes of exposure to an antigen.

C. Type II- Type II, or cytotoxic, hypersensitivity occurs when the system mistakenly identifies a normal constituent of the body as foreign.

D. Type IV- Type IV, or delayed-type, hypersensitivity occurs 24-72 hours after exposure to an allergen.

299. When the patient's eosinophil count is 50-90% of blood leukocytes, the nurse interprets the result as

A. indicative of idiopathic hypereosinophilic syndrome.-
When eosinophils make up 50-90% of white cell count, the patient is demonstrating severe eosinophilia.

B. indicating an allergic disorder.- Moderate eosinophilia, 15-40% of white cell count consisting of eosinophils, are found in patients with allergic disorders.

C. suggesting an allergic reaction.- A level between 5 and 15% eosinophils is nonspecific but does suggest allergic reaction.

D. normal.- Eosinophils normally make up 1-3% of the total number of white blood cells.

300. Which of the following interventions is the single most important aspect for the patient at risk for anaphylaxis?

A. Prevention- People who have experienced food, medication, idiopathic, or exercise-induced anaphylactic reactions should always carry an emergency kit containing epinephrine for injection to prevent the onset of the reaction upon exposure.

B. Use of antihistamines- While helpful, the patient may require epinephrine to treat a potential reaction.

C. Desensitization- While helpful, there must be no lapses in desensitization therapy because this may lead to the reappearance of an allergic reaction when the medication is re-instituted.

D. Wearing of medical alert bracelet- The medical alert bracelet will assist those rendering aid to the patient who has experienced an anaphylactic reaction.

301. When the nurse observes diffuse swelling involving the deeper skin layers in the patient who has experienced an allergic reaction, the nurse records the finding as

A. angioneurotic edema.-
The area of skin demonstrating angioneurotic edema may appear normal but often has a reddish hue and does not pit.

B. urticaria.- Urticaria (hives) is characterized as edematous skin elevations that vary in size and shape, itch, and cause local discomfort.

C. contact dermatitis.- Contact dermatitis refers to inflammation of the skin caused by contact with an allergenic substance, such as poison ivy.

D. pitting edema.- Pitting edema is the result of increased interstitial fluid and associated with disorders such as congestive heart failure.

302. Atopic allergic disorders are characterized by

A. a hereditary predisposition.- Atopic allergic disorders are characterized by a hereditary predisposition and production of a local reaction to IgE antibodies produced in response to common environmental allergens.

B. an IgA-mediated reaction. - Atopic and nonatopic allergic disorders are IgE-mediated allergic reactions.

C. production of a systemic reaction.- Atopic allergic disorders are characterized by a hereditary predisposition and production of a local reaction to IgE antibodies produced in response to common environmental allergens.

D. a response to physiologic allergens. - Atopic allergic disorders are characterized by a hereditary predisposition and production of a local reaction to IgE antibodies produced in response to common environmental allergens.

303.The nurse teaches the patient with allergies about anaphylaxis including which of the following statements?

A. The most common cause of anaphylaxis is penicillin. - The most common cause of anaphylaxis, accounting for about 75% of fatal anaphylactic reactions in the U.S., is penicillin.

B. Anaphylactoid (anaphylaxis-like) reactions are commonly fatal. -Although possibly severe, anaphylactoid reactions are rarely fatal.

C. The most common food item causing anaphylaxis is chocolate. - Food items that are common causes of anaphylaxis include peanuts, tree nuts, shellfish, fish, milk, eggs, soy and wheat.

D. Systemic reactions include urticaria and angioedema- Local reactions usually involve urticaria and angioedema at the site of the antigen exposure. Systemic reactions occur within about 30 minutes of exposure involving cardiovascular, respiratory, gastrointestinal, and integumentary organ systems.

304. Which of the following statements describes the clinical manifestations of a delayed hypersensitivity (type IV) allergic reaction to latex?

A. Signs and symptoms are localized to the area of exposure, usually the back of the hands. - Clinical manifestations of a delayed hypersensitivity reaction are localized to the area of exposure.

B. Signs and symptoms can be eliminated by changing glove brands or using powder-free gloves.- Clinical manifestations of an irritant contact dermatitis can be eliminated by changing glove brands or using powder-free gloves.

C. Signs and symptoms may worsen when hand lotion is applied before donning latex gloves. - With an irritant contact dermatitis, avoid use of hand lotion before donning gloves as this may worsen symptoms as lotions may leach latex proteins from the gloves.

D. Signs and symptoms occur within minutes after exposure to latex.- Described as a latex allergy, when clinical manifestations occur within minutes after exposure to latex, an immediate hypersensitivity (type I) allergic reaction has occurred.

305. Which of the following terms refers to fixation or immobility of a joint?


A. Ankylosis- Ankylosis may result from disease or scarring due to trauma.

B. Hemarthrosis- Hemarthrosis refers to bleeding into the joint.

C. Diarthrodial- Diarthrodial refers to a joint with two freely moveable parts.

D. Arthroplasty- Arthroplasty refers to replacement of a joint.

306. Accumulation of crystalline depositions in articular surfaces, bones, soft tissue, and cartilage is referred to as

A. tophi.-
Tophi, when problematic, are surgically excised.

B. subchondral bone.- Subchondral bone refers to a bony plate that supports the articular cartilage.

C. pannus.- Pannus refers to newly formed synovial tissue infiltrated with inflammatory cells.

D. joint effusion.- Joint effusion refers to the escape of fluid from the blood vessels or lymphatics into the joint cavity.

307. Passive range-of-motion exercises are indicated during which stage of rheumatic disease?

A. Acute- Passive range of motion is indicated because the patient is unable to perform exercises alone during an acute stage of rheumatic disease.

B. Subacute- Active assistive or active range of motion is recommended during the subacute stage of rheumatic diseases.

C. Inactive- Active range of motion and isometrics are recommended during the inactive stage of rheumatic diseases.

D. Remission- Active range of motion and isometrics are recommended during the remission stage of rheumatic diseases.

308. Which of the following connective tissue disorders is characterized by insoluble collagen being formed and accumulating excessively in the tissues?

A. Scleroderma-
Scleroderma occurs initially in the skin but also occurs in blood vessels, major organs, and body systems, potentially resulting in death.

B. Rheumatoid arthritis- Rheumatoid arthritis results from an autoimmune response in the synovial tissue with damage taking place in body joints.

C. Systemic lupus erythematosus- SLE is an immunoregulatory disturbance that results in increased autoantibody production.

D. Polymyalgia rheumatic- In polymyalgia rheumatic, immunoglobulin is deposited in the walls of inflamed temporal arteries.

309. Osteoarthritis is known as a disease that

A. is the most common and frequently disabling of joint disorders.-The functional impact of osteoarthritis on quality of life, especially for elderly patients, is often ignored.

B. affects young males.- Reiter's syndrome is a spondyloarthropathy that affects young adult males and is characterized primarily by urethritis, arthritis, and conjunctivitis.

C. requires early treatment because most of the damage appears to occur early in the course of the disease.- Psoriatic arthritis, characterized by synovitis, polyarthritis, and spondylitis requires early treatment because of early damage caused by disease.

D. affects the cartilaginous joints of the spine and surrounding tissues.- Ankylosing spondylitis causes the described problem and is usually diagnosed in the second or third decade of life.

310. Which of the following newer pharmacological therapies used for the treatment of osteoarthritis is thought to improve cartilage function and retard degradation as well as have some anti-inflammatory effects?


A. Viscosupplementation - Viscosupplementation, the intraarticular injection of hyaluronic acid, is thought to improve cartilage function and retard degradation. It may also have some anti-inflammatory effects.

B. Glucosamine- Glucosamine and chondroitin are thought to improve tissue function and retard breakdown of cartilage.

C. Chondroitin- Chondroitin and glucosamine are thought to improve tissue function and retard breakdown of cartilage.

D. Capsaicin- Capsaicin is a topical analgesic.

311. Which of the following statements reflect nursing interventions in the care of the patient with osteoarthritis?

A. Encourage weight loss and an increase in aerobic activity.-
Weight loss and an increase in aerobic activity such as walking, with special attention to quadriceps strengthening are important approaches to pain management.

B. Provide an analgesic after exercise.- Patients should be assisted to plan their daily exercise at a time when the pain is least severe, or plan to use an analgesic, if appropriate, prior to their exercise session.

C. Assess for the gastrointestinal complications associated with COX-2 inhibitors.
Gastrointestinal complications, especially GI bleeding, are associated with the use of nonsteroidal anti-inflammatory drugs (NSAIDs).

D. Avoid the use of topical analgesics.- Topical analgesics such as capsaicin and methylsalicylate may be used for pain management.

312. Fibromyalgia is a common condition that

A. involves chronic fatigue, generalized muscle aching and stiffness. - Fibromyalgia, is a common condition that involves chronic fatigue, generalized muscle aching, and stiffness.

B. is caused by a virus. - The cause is unknown and no pathological characteristics have been identified that are specific for the condition

C. is treated by diet, exercise, and physical therapy. - Treatment consists of attention to the specific symptoms reported by the patient. NSAIDs may be used to treat the diffuse muscle aching and stiffness. Tricyclic antidepressants are used to improve or restore normal sleep patterns and individualized programs of exercise are used to decrease muscle weakness and discomfort and to improve the general de-conditioning that occurs in these individuals

D. usually lasts for less than two weeks. - Fibromyalgia, is a common condition that involves chronic fatigue, generalized muscle aching, and stiffness. It is very typical for patients to have endured their symptoms for a long period of time.

313. Which of the following terms refers to a condition characterized by destruction of the melanocytes in circumscribed areas of the skin?

A. Vitiligo
- Vitiligo results in the development of white patches that may be localized or widespread.

B. Hirsutism- Hirsutism is the condition of having excessive hair growth.

C. Lichenification- Lichenification refers to a leathery thickening of the skin.

D. Telangiectases- Telangiectases refers to red marks on the skin caused by stretching of the superficial blood vessels.

314. Of the following types of cells, which are believed to play a significant role in cutaneous immune system reactions?

A. Langerhans' cells- Langerhans' cells are common to the epidermis and are accessory cells of the afferent immune system process.

B. Merkel's cells- Merkel's cells are the receptor cells in the epidermis that transmit stimuli to the axon via a chemical response.

C. Melanocytes- Melanocytes are special cells of the epidermis that are primarily involved in producing melanin, which colors the hair and skin.

D. Phagocytes- Phagocytes are white blood cells that engulf and destroy foreign materials.

315. When the nurse assesses the patient and observes blue-red and dark brown plaques and nodules, she recognizes that these manifestations are associated with

A. Kaposi's sarcoma.- Kaposi's sarcoma is a frequent comorbidity of the patient with AIDS.

B. platelet disorders.- With platelet disorders, the nurse observes ecchymoses (bruising) and purpura (bleeding into the skin).

C. allergic reactions.- Urticaria (wheals or hives) is the manifestation of allergic reactions.

D. syphilis.- A painless chancre or ulcerated lesion is a typical finding in the patient with syphilis.

316. The nurse reading the physician's report of an elderly patient’s physical examination knows a notation that the patient demonstrates xanthelasma refers to

A. yellowish waxy deposits on upper eyelids.- The change is a common, benign manifestation of aging skin or it can sometimes signal hyperlipidemia.

B. liver spots.- Solar lentigo is the term that refers to liver spots.

C. dark discoloration of the skin.- Melasma is the term that refers to dark discoloration of the skin.

D. bright red moles.- Cherry angioma is the term that is used to describe a bright red mole.

317. The nurse notes that the patient demonstrates generalized pallor and recognizes that this finding may be indicative of

A. anemia.-
In the light-skinned individual, generalized pallor is a manifestation of anemia. In brown- and black-skinned individuals, anemia is demonstrated as a dull skin appearance.

B. albinism.- Albinism is a condition of total absence of pigment in which the skin appears whitish pink.

C. vitiligo.- Vitiligo is a condition characterized by the destruction of the melanocytes in circumscribed areas of skin, resulting in patchy, milky white spots.

D. local arterial insufficiency.- Local arterial insufficiency is characterized by marked localized pallor.


318. Which of the following terms refers most precisely to a localized skin infection of a single hair follicle?

A. Furuncle
- Furuncles occur anywhere on the body, but are most prevalent in areas subjected to irritation, pressure friction, and excessive perspiration, such as the back of the neck, the axillae, or the buttocks.

B. Carbuncle- A carbuncle is a localized skin infection involving several hair follicles.

C. Chelitis.- Chelitis refers to dry cracking at the corners of the mouth.

D. Comedone.- Comedones are the primary lesions of acne, caused by sebum blockage in the hair follicle.

319. The nurse recommends which of the following types of therapeutic baths for its antipruritic action?

A. Colloidal (Aveeno, oatmeal)- Aveeno or oatmeal baths are recommended to decrease itching associated with a dermatologic disorder.

B. Sodium bicarbonate (baking soda)- Baking soda baths are cooling but dangerous. The tub gets very slippery and a bath mat must be used in the tub.

C. Water- Water baths have the same effect as wet dressings, not known to counteract itching.

D. Saline- Saline baths have the same effects as saline dressings, not known to counteract itching.

320. Which of the following materials consists of a powder in water?

A. Suspension- A suspension requires shaking before application, exemplified by calamine lotion.

B. Hygroscopic agent- A hygroscopic agent is a powder that acts to absorb and retain moisture from the air and to reduce friction between surfaces.

C. Paste- A paste is a mixture of powder and ointment.

D. Linament- A linament is a lotion with oil added to prevent crusting.

321. Which of the following skin conditions is caused by staphylococci, streptococci, or multiple bacteria?

A. Impetigo- Impetigo is seen at all ages, but is particularly common among children living under poor hygienic conditions.

B. Scabies- Scabies is caused by the itch mite.

C. Pediculosis capitis- Pediculosis capitis is caused by head lice.

D. Poison ivy- Poison ivy is a contact dermatitis caused by the oleoresin given off by a particular form of ivy.


322.The nurse teaches the patient who demonstrates herpes zoster (shingles) that

A. the infection results from reactivation of the chickenpox virus.-
It is assumed that herpes zoster represents a reactivation of latent varicella (chickenpox) virus and reflects lowered immunity.

B. once a patient has had shingles, they will not have it a second time.- It is believed that the varicella zoster virus lies dormant inside nerve cells near the brain and spinal cord and is reactivated with weakened immune systems and cancers.

C. a person who has had chickenpox can contract it again upon exposure to a person with shingles.- A person who has had chickenpox is immune and, therefore, not at risk of infection after exposure to patients with herpes zoster.

D. There are no known medications that affect the course of shingles.-There is some evidence that infection is arrested if oral antiviral agents are administered within 24 hours of the initial eruption.

323. Development of malignant melanoma is associated with which of the following risk factors?

A. Individuals with a history of severe sunburn-
Ultraviolet rays are strongly suspected as the etiology of malignant melanoma.

B. African-American heritage- Fair-skinned, blue-eyed, light-haired people of Celtic or Scandinavian origin are at higher risk for development of malignant melanoma.

C. People who tan easily- People who burn and do not tan are at risk for development of malignant melanoma.

D. Elderly individuals residing in the Northeast- Elderly individuals who retire to the southwestern United States appear to have a higher incidence of development of malignant melanoma.

324. When caring for a patient receiving autolytic debridement therapy, the nurse

A. advises the patient about the foul odor that will occur during therapy.- During autolytic debridement therapy a foul odor will be produced by the breakdown of cellular debris. This odor does not indicate that the wound is infected.

B. ensures that the dressing is kept dry at all times.- During autolytic debridement therapy the wound is kept moist.

C. ensures that the wound is kept open to the air for at least six hours per day.-During autolytic debridement therapy the wound is covered with an occlusive dressing.

D. Uses an enzymatic debriding agent such as Pancrease. - Commercially available enzymatic debriding agents include Accuzyme, Clooagenase, Granulex, and Zymase.

325. Which of the following reflect the pathophysiology of cutaneous signs of HIV disease?

A. Immune function deterioration- Cutaneous signs may be the first manifestations of HIV, appearing in more than 90 per cent of HIV infected patients as the immune function deteriorates. Common complaints include pruritis, folliculitis, and chronic actinic dermatitis.

B. High CD4 count- Cutaneous signs of HIV disease correlate to low CD4 counts.

C. Genetic predisposition- Cutaneous signs of HIV disease appear as immune function deteriorates.

D. Decrease in normal skin flora- Cutaneous signs of HIV disease appear as immune function deteriorates.

326. Most skin conditions related to HIV disease may be helped primarily by

A. highly active antiretroviral therapy (HAART).-The goals of all HIV-related conditions include improvement of CD4 count and lowering of viral load. Initiation of HAART (highly active antiretroviral therapy) will help improve most skin conditions related to HIV disease. Symptomatic relief will be required until the skin condition improves.

B. symptomatic therapies.- Initiation of HAART (highly active antiretroviral therapy) will help improve most skin conditions related to HIV disease. Symptomatic relief will be required until the skin condition improves.

C. low potency topical corticosteroid therapy.- High-potency, not low-potency, topical corticosteroid therapy may be helpful for some skin conditions.

D. improvement of the patient's nutritional status.- Improvement of the patient's nutritional status is beneficial for the overall treatment of HIV disease; it is not specific for treatment of skin conditions.

327. Which of the following terms refers to a graft derived from one part of a patient's body and used on another part of that same patient's body?

A. Autograft-
Autografts of full-thickness and pedicle flaps are commonly used for reconstructive surgery, months or years after the initial injury.

B. Allograft- An allograft is a graft transferred from one human (living or cadaveric) to another human.

C. Homograft- A homograft is a graft transferred from one human (living or cadaveric) to another human.

D. Heterograft- A heterograft is a graft obtained from an animal of a species other than that of the recipient.

328. When the emergency nurse learns that the patient suffered injury from a flash flame, the nurse anticipates which depth of burn?

A. Deep partial thickness-
A deep partial thickness burn is similar to a second-degree burn and is associated with scalds and flash flames.

B. Superficial partial thickness- Superficial partial thickness burns are similar to first-degree burns and are associated with sunburns.

C. Full thickness- Full thickness burns are similar to third-degree burns and are associated with direct flame, electric current, and chemical contact.

D. Superficial- Injury from a flash flame is not associated with a burn that is limited to the epidermis.

329. Regarding emergency procedures at the burn scene, the nurse teaches which of the following guidelines?

A. Never wrap burn victims in ice.-
Such procedure may worsen the tissue damage and lead to hypothermia in patients with large burns.

B. Apply ice directly to a burn area.- Ice must never be applied directly to a burn because it may worsen the tissue damage.

C. Never apply water to a chemical burn.- Chemical burns resulting from contact with a corrosive material are irrigated immediately.

D. Maintain cold dressings on a burn site at all times.- Such procedures may worsen the tissue damage and lead to hypothermia in patients with large burns.

330. The first dressing change for an autografted area is performed

A. as soon as foul odor or purulent drainage is noted, or 3-5 days after surgery.
A foul odor or purulent infection may indicate infection and should be reported to the surgeon immediately.

B. within 12 hours after surgery.- The first dressing change usually occurs 3-5 days after surgery.

C. within 24 hours after surgery.- The first dressing change usually occurs 3-5 days after surgery.

D. as soon as sanguineous drainage is noted.- Sanguineous drainage on a dressing covering an autograft is an anticipated abnormal observation postoperatively.

331. Which of the following observations in the patient who has undergone allograft for treatment of burn site must be reported to the physician immediately?

A. Crackles in the lungs- Crackles in the lungs may indicate a fluid buildup indicative of congestive heart failure and pulmonary edema.

B. Pain at the allograft donor site- Pain at the allograft donor site is anticipated, since the nerve endings have been stimulated.

C. Sanguineous drainage at the allograft donor site- Sanguineous drainage at the allograft donor site is anticipated, since upper layers of tissue have been removed.

D. Decreased pain at the allograft recipient site- Decreased pain at the recipient site is anticipated since the wound has been protected by the graft.

332. Which of the following factors are associated with increased fluid requirements in the management of patients with burn injury?

A. Inhalation injuries-
Factors associated with increased fluid requirements include inhalation injuries, delayed resuscitation, scald burn injuries, high-voltage electrical injuries, hyperglycemia, alcohol intoxification and chronic diuretic therapy.

B. Chemical burn injuries- Chemical burn injuries are not associated with increased fluid requirements.

C. Low-voltage electrical injuries- Low-voltage electrical injuries are not associated with increased fluid requirements.

D. Hypoglycemia- Hypoglycemia is not associated with increased fluid requirements.

333. antimicrobial barrierÒActicoat dressings used in the treatment of burn wounds can be left in place for

A. five days.- antimicrobial barrier dressings can be left in place forÒActicoat up to five days thus helping to decrease discomfort to the patient, decrease costs of dressing supplies, and decrease nursing time involved in burn dressing changes.

B. seven to ten days.- antimicrobial barrier dressings
ÒActicoat can be left in place for up to five days.

C. three days.- Acticoat antimicrobial barrier dressings can be left in place for up to five days.

D. two days.- antimicrobial barrier dressings can be left in
ÒActicoat place for up to five days.

334. A new biosynthetic dressing used , is used to treat
Òin the treatment of burns, TransCyte

A. burns of indeterminate depth
- is used to treat burns in which the depth isÒTransCyte indeterminate or between superficial and deep partial thickness in depth.

B. partial-thickness burns.- is a temporary biosynthetic
ÒBCG Matrix wound covering intended for use with partial-thickness burns and donor sites

C. superficial burns.- is used to treat burns in which
ÒTransCyte the depth is indeterminate or between superficial and deep partial thickness in depth.

D. donor sites.- is a temporary biosynthetic wound
ÒBCG Matrix covering intended for use with partial-thickness burns and donor sites

335. Which of the following statements reflect current research regarding the utilization of non-pharmacological measures in the management of burn pain?

A. Music therapy may provide reality orientation, distraction, and sensory stimulation.
- Researchers have found that music affects both the physiologic and psychological aspects of the pain experience. Music diverts the patient’s attention away from the painful stimulus. Music may also provide reality orientation, distraction, and sensory stimulation. It also allows for patient self-expression.

B. Music therapy diverts the patient's attention toward painful stimulus. -Music diverts the patient’s attention away from, not toward, the painful stimulus.

C. Humor therapy has not proven effective in the management of burn pain. -Humor therapy has proven effective in the management of burn pain.

D. Pet therapy has proven effective in the management of burn pain. -Pet therapy has not proven effective in the management of burn pain.

336. The most important intervention in the nutritional support of a patient with a burn injury is to provide adequate nutrition and calories to:

A. decrease catabolism.-
The most important intervention in the nutritional support of a patient with a burn injury is to provide adequate nutrition and calories to decrease catabolism. Nutritional support with optimized protein intake can decrease the protein losses by approximately 50%.

B. increase metabolic rate.- A marked increase in metabolic rate is seen after a burn injury; interventions are instituted to decrease metabolic rate and catabolism.

C. increase glucose demands.- A marked increase in glucose demands are seen after a burn injury; interventions are instituted to decrease glucose demands and catabolism.

D. increase skeletal muscle breakdown. - Rapid skeletal muscle breakdown with amino acids serving as the energy source is seen after a burn injury; interventions are instituted to decrease catabolism.

337. Which of the following terms refers to the absence of the natural lens?

A. Aphakia- When a cataract is extracted, and an intraocular lens implant is not used, the patient demonstrates aphakia.

B. Scotoma- Scotoma refers to a blind or partially blind area in the visual field.

C. Keratoconus- Keratoconus refers to a cone-shaped deformity of the cornea.

D. Hyphema- Hyphema refers to blood in the anterior chamber of the eye.

338. Edema of the conjunctiva is termed

A. chemosis.- Chemosis is a common manifestation of pink-eye.

B. papilledema.- Papilledema refers to swelling of the optic disk due to increased intracranial pressure.

C. proptosis.- Proptosis is the downward displacement of the eyeball.

D. strabismus.- Strabismus is a condition in which there is a deviation from perfect ocular alignment.

339. When the patient tells the nurse that his vision is 20/200, and asks what that means, the nurse informs the patient that a person with 20/200 vision

A. sees an object from 20 feet away that a person with normal vision sees from 200
feet away.- The fraction 20/20 is considered the standard of normal vision.

B. sees an object from 200 feet away that a person with normal vision sees from 20 feet away. Most people, positioned 20 feet from the eye chart, can see the letters designated as 20/20 from a distance of 20 feet.-

C. sees an object from 20 feet away that a person with normal vision sees from 20 feet away.- The standard of normal vision, 20/20 means that the patient can read the 20/20 line from a distance of 20 feet.

D. sees an object from 200 feet away that a person with normal vision sees from 200 feet away.- In order to read the 20/20 line, the person of normal vision will be standing at a distance of 20 feet from the chart.

340. Which type of glaucoma presents an ocular emergency?

A. Acute angle-closure glaucoma- Acute angle-closure glaucoma results in rapid progressive visual impairment.

B. Normal tension glaucoma- Normal tension glaucoma is treated with topical medication.

C. Ocular hypertension- Ocular hypertension is treated with topical medication.

D. Chronic open-angle glaucoma- Chronic open-angle glaucoma is treated initially with topical medications, with oral medications added at a later time.

341. Which of the following categories of medications increases aqueous fluid outflow in the patient with glaucoma?

A. Cholinergics-
Cholinergics increase aqueous fluid outflow by contracting the ciliary muscle, causing miosis, and opening the trabecular meshwork.

B. Beta-blockers- Beta-blockers decrease aqueous humor production.

C. Alpha-adrenergic agonists- Alpha-adrenergic agonists decrease aqueous humor production.

D. Carbonic anhydrase inhibitors- Carbonic anhydrase inhibitors decrease aqueous humor production.

342. Which of the following statements describe refractive surgery?

A. Refractive surgery is an elective, cosmetic surgery performed to reshape the cornea. - Refractive surgery is an elective procedure and is considered a cosmetic procedure (to achieve clear vision without the aid of prosthetic devices). It is performed to reshape the cornea for the purpose of correction of all refractive errors.

B. Refractive surgery will alter the normal aging of the eye. - Refractive surgery will not alter the normal aging process of the eye.

C. Refractive surgery may be performed on all patients, even if they have underlying health conditions. - Patients with conditions that are likely to adversely affect corneal wound healing (corticosteroid use, immunosuppression, elevated IOP) are not good candidates for the procedure.

D. Refractive surgery may be performed on patients with an abnormal corneal structure as long as they have a stable refractive error. - The corneal structure must be normal and refractive error stable.

343. The nurse knows that a postoperative vision-threatening complication of LASIK refractive surgery, diffuse lamellar keratitis (DLK) occurs

A. in the first week after surgery. -
DLK is a peculiar, non-infectious, inflammatory reaction in the lamellar interface after LASIK. It is characterized by a white granular, diffuse culture-negative lamellar keratitis occurring in the first week after surgery. Studies suggest that since no single agent appears to be solely the cause of DLK, a multifactorial etiology is likely.

B. 1 month after surgery.- DLK occurs in the first week after surgery.

C. 2-3 months after surgery.- DLK occurs in the first week after surgery.

D. 6 months after surgery. - DLK occurs in the first week after surgery.

344. The nurse advises the patient undergoing photodynamic therapy (PDT) for macular degeneration to avoid exposure to direct sunlight or bright lights for

A. the first five days after the procedure.-
Photodynamic therapy includes the use of verteporfin, a light-activated dye. The dye within the blood vessels near the surface of the skin could become activated with exposure to strong light, such as sunlight or bright lights. Ordinary indoor light is not a problem. The patient should be counseled to wear protective clothing, such as long-sleeved shirts, sunglasses, and wide-brimmed hats, if the patient has to go outdoors during daylight hours in the first five days post-treatment. Inadvertent sunlight exposure can lead to severe blistering of the skin and sunburn.

B. the first 24 hours after the procedure.- The patient should avoid exposure to direct sunlight or bright lights for the first five days post-treatment.

C. two weeks after the procedure.- The patient should avoid exposure to direct sunlight or bright lights for the first five days post-treatment.

D. the first month after the procedure. - The patient should avoid exposure to direct sunlight or bright lights for the first five days post-treatment.

345. Retinoblastoma is the most common eye tumor of childhood; it is hereditary in

A. 30-40% of cases.- Retinoblastoma can be hereditary or nonhereditary. It is hereditary in 30-40% of cases. All bilateral cases are hereditary.

B. 10-20% of cases.- Retinoblastoma is hereditary in 30-40% of cases.

C. 25-50% of cases.- Retinoblastoma is hereditary in 30-40% of cases.

D. 50-75% of cases.- Retinoblastoma is hereditary in 30-40% of cases.

346. Which of the following terms refers to altered sensation of orientation in space?

A. Dizziness- Dizziness may be associated with inner ear disturbances.

B. Vertigo- Vertigo is the illusion of movement where the individual or the surroundings are sensed as moving.

C. Tinnitus- Tinnitus refers to a subjective perception of sound with internal origin.

D. Nystagmus- Nystagmus refers to involuntary rhythmic eye movement.

347. Of the following terms, which describes a condition characterized by abnormal spongy bone formation around the stapes?

A. Otosclerosis
- Otosclerosis is more common in females than males and is frequently hereditary.

B. Middle ear effusion- A middle ear effusion is denoted by fluid in the middle ear without evidence of infection.

C. Chronic otitis media- Chronic otitis media is defined as repeated episodes of acute otitis media causing irreversible tissue damage and persistent tympanic membrane perforation.

D. Otitis externa- Otitis externa refers to inflammation of the external auditory canal.

348. Ossiculoplasty is defined as

A. surgical reconstruction of the middle ear bones.- Ossiculoplasty is performed to restore hearing.

B. surgical repair of the eardrum.- Surgical repair of the eardrum is termed tympanoplasty.

C. incision into the tympanic membrane.- Tympanotomy or myringotomy is the term used to refer to incision into the tympanic membrane.

D. incision into the eardrum.- Tympanotomy or myringotomy is the term used to refer to incision into the tympanic membrane.

349. Which of the following terms refers to surgical repair of the tympanic membrane?

A. Tympanoplasty- Tympanoplasty may be necessary to repair a scarred eardrum.

B. Tympanotomy- A tympanotomy is an incision into the tympanic membrane.

C. Myringotomy- A myringotomy is an incision into the tympanic membrane.

D. Ossiculoplasty- An ossiculoplasty is a surgical reconstruction of the middle ear bones to restore hearing.

350. Of the following tests, which uses a tuning fork between two positions to assess hearing?

A. Rinne's
- In the Rinne's test, the examiner shifts the stem of a vibrating tuning fork between two positions to test air conduction of sound and bone conduction of sound.

B. Whisper- The whisper test involves covering the untested ear and, whispering from a distance of 1 or 2 feet from the unoccluded ear, and the ability of the patient to repeat what was whispered.

C. Watch tick- The watch tick test relies on the ability of the patient to perceive the high-pitched sound made by a watch held at the patient's auricle.

D. Weber's- The Weber's test uses bone conduction to test lateralization of sound.

351. Which of the following conditions of the inner ear is associated with normal hearing?

A. Vestibular neuronitis-
Vestibular neuronitis is a disorder of the vestibular nerve characterized by severe vertigo with normal hearing.

B. Meniere's disease- Meniere's disease is associated with progressive sensorineural hearing loss.

C. Labyrinthitis- Labyrinthitis is associated with varying degrees of hearing loss.

D. Endolymphatic hydrops- Endolymphatic hydrops refers to dilation in the endolymmphatic space associated with Meniere's disease.

352. Of the following terms, which refers to the progressive hearing loss associated with aging?

A. Presbycusis- Both middle and inner ear age-related changes result in hearing loss.

B. Exostoses- Exostoses refers to small, hard, bony protrusions in the lower posterior bony portion of the ear canal.

C. Otalgia- Otalgia refers to a sensation of fullness or pain in the ear.

D. Sensorineural hearing loss- Sensorineural hearing loss is loss of hearing related to damage of the end organ for hearing and/or cranial nerve VIII.

353. Which of the following statements describes benign paroxysmal positional vertigo (BPPV)?

A. The vertigo is usually accompanied by nausea and vomiting; however hearing impairment does not generally occur.
BPPV is a brief period of incapacitating vertigo that occurs when the position of the patient's head is changed with respect to gravity. The vertigo is usually accompanied by nausea and vomiting; however hearing impairment does not generally occur.

B. The onset of BPPV is gradual. - The onset of BPPV is sudden and followed by a predisposition for positional vertigo, usually for hours to weeks but occasionally for months or years.

C. canal. This debris is formed from small crystals of calcium carbonate from the inner ear structure, the utricle.

D. BPPV is stimulated by the use of certain medication such as acetaminophen.
BPPV is frequently stimulated by head trauma, infection, or other events.

354. Nursing management of the patient with acute symptoms of benign paroxysmal positional vertigo includes which of the following?

A. Bed rest - Bed rest is recommended for patients with acute symptoms. Canalith repositioning procedures (CRP) may be used to provide resolution of vertigo, and patients with acute vertigo may be medicated with meclizine for 1-2 weeks.

B. The Epley repositioning procedure- The Epley procedure is not recommended for patients with acute vertigo.

C. Meclizine for 2-4 weeks- Patients with acute vertigo may be medicated with meclizine for 1-2 weeks.

D. The Dix-Hallpike procedure.- The Dix-Hallpike test is an assessment test used to evaluate for BPPV.

355. Which of the following terms refers to the inability to recognize objects through a particular sensory system?

A. Agnosia-
Agnosia may be visual, auditory, or tactile.

B. Dementia- Dementia refers to organic loss of intellectual function.

C. Ataxia- Ataxia refers to the inability to coordinate muscle movements.

D. Aphasia- Aphasia refers to loss of the ability to express oneself or to understand language.

356. Which of the following terms refers to weakness of both legs and the lower part of the trunk?

A. Paraparesis- Paraparesis is a frequent manifestation of degenerative neurologic disorders.

B. Hemiplegia- Hemiplegia refers to paralysis of one side of the body or a part of it due to an injury to the motor areas of the brain.

C. Quadriparesis- Quadriparesis refers to weakness that involves all four extremities.

D. Paraplegia- Paraplegia refers to paralysis of both legs and the lower trunk.

357. Of the following neurotransmitters, which demonstrates inhibitory action, helps control mood and sleep, and inhibits pain pathways?

A. Serotonin-
The sources of serotonin are the brain stem, hypothalamus, and dorsal horn of the spinal cord.

B. Enkephalin- Enkephalin is excitatory and associated with pleasurable sensations.

C. Norepinephrine- Norepinephrine is usually excitatory and affects mood and overall activity.

D. Acetylcholine-Acetylcholine is usually excitatory, but the parasympathetic effects are sometimes inhibitory.

358. The lobe of the brain that contains the auditory receptive areas is the ____________ lobe.

A. temporal- The temporal lobe plays the most dominant role of any area of the cortex in cerebration.

B. frontal- The frontal lobe, the largest lobe, controls concentration, abstract thought, information storage or memory, and motor function.

C. parietal- The parietal lobe contains the primary sensory cortex, which analyzes sensory information and relays interpretation to the thalamus and other cortical areas.

D. occipital- The occipital lobe is responsible for visual interpretation.

359. The lobe of the brain that is the largest and controls abstract thought is the ____________ lobe.

A. frontal- The frontal lobe also controls information storage or memory and motor function.

B. temporal- The temporal lobe contains the auditory receptive area.

C. parietal- The parietal lobe contains the primary sensory cortex, which analyzes sensory information and relays interpretation to the thalamus and other cortical areas.

D. occipital -The occipital lobe is responsible for visual interpretation.

360. Which of the following terms is used to describe the fibrous connective tissue that covers the brain and spinal cord?

A. Meninges- The meninges have three layers, the dura mater, arachnoid mater, and pia mater.

B. Dura mater- The dura mater is the outmost layer of the protective covering of the brain and spinal cord.

C. Arachnoid mater- The arachnoid is the middle membrane of the protective covering of the brain and spinal cord.

D. Pia mater-The pia mater is the innermost membrane of the protective covering of the brain and spinal cord.

361. The cranial nerve that is responsible for salivation, tearing, taste, and sensation in the ear is the _____________________ nerve.

A. vestibulocochlear- The vestibulocochlear (VII) cranial nerve is responsible for hearing and equilibrium.

B. oculomotor- The oculomotor (III) cranial nerve is responsible for the muscles that move the eye and lid, pupillary constriction, and lens accommodation.

C. trigeminal- The trigeminal (V) cranial nerve is responsible for facial sensation, corneal reflex, and mastication.

D. facial- The facial (VII) nerve controls facial expression and muscle movement.

362. The cranial nerve that is responsible for muscles that move the eye and lid is the _____________________ nerve.

A. oculomotor
- The oculomotor (III) cranial nerve is also responsible for pupillary constriction and lens accommodation.

B. trigeminal- The trigeminal (V) cranial nerve is responsible for facial sensation, corneal reflex, and mastication.

C. vestibulocochlear- The vestibulocochlear (VII) cranial nerve is responsible for hearing and equilibrium.

D. facial- The facial (VII) nerve is responsible for salivation, tearing, taste, and sensation in the ear.

363. The cranial nerve that is responsible for facial sensation and corneal reflex is the _____________________ nerve.

A. trigeminal-
The trigeminal (V) cranial nerve is also responsible for mastication.

B. oculomotor- The oculomotor (III) cranial nerve is responsible for the muscles that move the eye and lid, pupillary constriction, and lens accommodation.

C. vestibulocochlear-The vestibulocochlear (VII) cranial nerve is responsible for hearing and equilibrium.

D. facial- The facial nerve is responsible for salivation, tearing, taste, and sensation in the ear.

364. Upper motor neuron lesions cause

A. no muscle atrophy.- Upper motor neuron lesions do not cause muscle atrophy but do cause loss of voluntary control.

B. decreased muscle tone.- Lower motor neuron lesions cause decreased muscle tone.

C. flaccid paralysis.- Lower motor neuron lesions cause flaccid paralysis.

D. absent or decreased reflexes.- Lower motor neuron lesions cause absent or decreased reflexes.

365. Lower motor neuron lesions cause

A. flaccid muscle paralysis.- Lower motor neuron lesions cause flaccid muscle paralysis, muscle atrophy, decreased muscle tone, and loss of voluntary control.

B. increased muscle tone.- Upper motor neuron lesions cause increased muscle tone.

C. no muscle atrophy.- Upper motor neuron lesions cause no muscle atrophy.

D. hyperactive and abnormal reflexes.- Upper motor neuron lesions cause hyperactive and abnormal reflexes.

366. The percentage of patients over the age of 70 admitted to the hospital with delirium is about

A. 25%.-
About 25% of patients over the age of 70 admitted to the hospital have delirium. The cause is often reversible and treatable (as in drug toxicity, vitamin B12 deficiency or thyroid disease) or chronic and irreversible. Depression may produce impairment of attention and memory.

B. 10%.- About 25% of patients over the age of 70 admitted to the hospital have delirium.

C. 40%. About 25% of patients over the age of 70 admitted to the hospital have delirium.

D. 50%.- About 25% of patients over the age of 70 admitted to the hospital have delirium.

367. Structural and motor changes related to aging that may be assessed in geriatric patients during an examination of neurologic function include which of the following?

A. Decreased or absent deep tendon reflexes-
Structural and motor changes related to aging that may be assessed in geriatric patients include decreased or absent deep tendon reflexes.

B. Increased pupillary responses- Pupillary responses are reduced or may not appear at all in the presence of cataracts

C. Increased autonomic nervous system responses.-There is an overall slowing of autonomic nervous system responses

D. Enhanced reaction and movement times- Strength and agility are diminished and reaction and movement times are decreased.

368. What safety actions does the nurse need to take for a patient on oxygen therapy who is undergoing magnetic resonance imaging (MRI)?

A. Ensure that no patient care equipment containing metal enters the room where the MRI is located. - For patient safety the nurse must make sure no patient care equipment (e.g., portable oxygen tanks) that contains metal or metal parts enters the room where the MRI is located. The magnetic field generated by the unit is so strong that any metal-containing items will be strongly attracted and can literally be pulled away with such great force that they can fly like projectiles towards the magnet.

B. Securely fasten the patient's portable oxygen tank to the bottom of the MRI table after the patient has been positioned on the top of the MRI table.- For patient safety the nurse must make sure no patient care equipment (e.g., portable oxygen tanks) that contains metal or metal parts enters the room where the MRI is located.

C. Check the patient's oxygen saturation level using a pulse oximeter after the patient has been placed on the MRI table.- For patient safety the nurse must make sure no patient care equipment (e.g., portable oxygen tanks) that contains metal or metal parts enters the room where the MRI is located.

D. No special safety actions need to be taken.- For patient safety the nurse must make sure no patient care equipment (e.g., portable oxygen tanks) that contains metal or metal parts enters the room where the MRI is located.
369. Which of the following terms refer to a method of recording, in graphic form, the electrical activity of the muscle?

A. Electromyogram- Electromyogram is a method of recording, in graphic form, the electrical activity of the muscle.

B. Electroencephalogram- Electroencephalogram is a method of recording, in graphic form, the electrical activity of the brain.

C. Electrocardiography- Electrocardiography is performed to assess the electrical activity of the heart.

D. Electrogastrography- Electrogastrography is an electrophysiologic study performed to assess gastric motility disturbances.

370. Which of the following are sympathetic effects of the nervous system?

A. Dilated pupils- Dilated pupils are a sympathetic effect of the nervous system. Constricted pupils are a parasympathetic effect.

B. Decreased blood pressure- Decreased blood pressure is a parasympathetic effect. Increased blood pressure is a sympathetic effect.

C. Increased peristalsis- Increased peristalsis is a parasympathetic effect. Decreased peristalsis is a sympathetic effect.

D. Decreased respiratory rate- Decreased respiratory rate is a parasympathetic effect. Increased respiratory rate is a sympathetic effect.

371. Lesions in the temporal lobe may result in which of the following types of agnosia?

A. Auditory -
Lesions in the temporal lobe (lateral and superior portions) may result in auditory agnosia.

B. Visual- Lesions in the occipital lobe may result in visual agnosia.

C. Tactile- Lesions in the parietal lobe may result in tactile agnosia.

D. Relationship- Lesions in the parietal lobe (posteroinferior regions) may result in relationship and body part agnosia.

372. When the nurse observes that the patient has extension and external rotation of the arms and wrists and extension, plantar flexion, and internal rotation of the feet, she records the patient's posturing as

A. decerebrate.- Decerebrate posturing is the result of lesions at the midbrain and is more ominous than decorticate posturing.

B. normal.- The described posturing results from cerebral trauma and is not normal.

C. flaccid.- The patient has no motor function, is limp, and lacks motor tone with flaccid posturing.

D. decorticate.- In decorticate posturing, the patient has flexion and internal rotation of the arms and wrists and extension, internal rotation, and plantar flexion of the feet.

373. Monro-Kellie hypothesis refers to

A. the dynamic equilibrium of cranial contents.- The hypothesis states that because of the limited space for expansion within the skull, an increase in any one of the cranial contents (brain tissue, blood, or cerebrospinal fluid) causes a change in the volume of the others.

B. unresponsiveness to the environment.- Akinetic mutism is the phrase used to refer to unresponsiveness to the environment.

C. the brain's attempt to restore blood flow by increasing arterial pressure to overcome the increased intracranial pressure.- Cushing's response is the phrase used to refer to the brain's attempt to restore blood flow by increasing arterial pressure to overcome the increased intracranial pressure.

D. a condition in which the patient is wakeful but devoid of conscious content, without cognitive or affective mental function.- Persistent vegetative state is the phrase used to describe a condition in which the patient is wakeful but devoid of conscious content, without cognitive or affective mental function.

374. A patient who demonstrates an obtunded level of consciousness

A. sleeps almost constantly but can be aroused and can follow simple commands.
An obtunded patient stays awake only with persistent stimulation.

B. has difficulty following commands, and may be agitated or irritable.
A confused patient has difficulty following commands, and may be agitated or irritable.

C. sleeps often and shows slowed speech and thought processes.
A patient who sleeps often and shows slowed speech and thought processes is described as lethargic.

D. does not respond to environmental stimuli.- A comatose patient does not respond to environmental stimuli.

375. An osmotic diuretic, such as Mannitol, is given to the patient with increased intracranial pressure (IICP) in order to

A. dehydrate the brain and reduce cerebral edema.-
Osmotic diuretics draw water across intact membranes, thereby reducing the volume of brain and extracellular fluid.

B. control fever.- Antipyretics and a cooling blanket are used to control fever in the patient with IICP.

C. control shivering.- Chloropromazine (Thorazine) may be prescribed to control shivering in the patient with IICP.

D. reduce cellular metabolic demands.- Medications such as barbiturates are given to the patient with IICP to reduce cellular metabolic demands.

376. Which of the following positions are employed to help reduce intracranial pressure (ICP)?

A. Avoiding flexion of the neck with use of a cervical collar- Use of a cervical collar promotes venous drainage and prevents jugular vein distortion that will increase ICP.

B. Keeping the head flat with use of no pillow- Slight elevation of the head is maintained to aid in venous drainage unless otherwise prescribed.

C. Rotating the neck to the far right with neck support- Extreme rotation of the neck is avoided because compression or distortion of the jugular veins increases ICP.

D. Extreme hip flexion supported by pillows- Extreme hip flexion is avoided because this position causes an increase in intra-abdominal pressure and intrathoracic pressure, which can produce a rise in ICP.

377. Which of the following insults or abnormalities most commonly causes ischemic stroke?

A. Cocaine use- Cocaine is a potent vasoconstrictor and may result in a life-threatening reaction, even with the individual's first unprescribed use of the drug.

B. Arteriovenous malformation- Arteriovenous malformations are associated with hemorrhagic strokes.

C. Trauma- Trauma is associated with hemorrhagic strokes.

D. Intracerebral aneurysm rupture- Intracerebral aneurysm rupture is associated with hemorrhagic strokes.

378. When the patient is diagnosed as having global aphasia, the nurse recognizes that the patient will

A. be unable to form words that are understandable or comprehend the spoken word.
Global aphasia is a combination of expressive and receptive aphasia and presents tremendous challenge to the nurse to effectively communicate with the patient.

B. be unable to comprehend the spoken word.- In receptive aphasia, the patient is unable to form words that are understandable.

C. be unable to form words that are understandable.- In expressive aphasia, the patient is unable to form words that are understandable.

D. be unable to speak at all.- The patient who is unable to speak at all is referred to as mute.

379. Which of the following terms related to aphasia refers to the inability to perform previously learned purposeful motor acts on a voluntary basis?

A. Apraxia
- Verbal apraxia refers to difficulty in forming and organizing intelligible words although the musculature is intact.

B. Agnosia- Agnosia is failure to recognize familiar objects perceived by the senses.

C. Agraphia- Agraphia refers to disturbances in writing intelligible words.

D. Perseveration- Perseveration is the continued and automatic repetition of an activity or word or phrase that is no longer appropriate.

380. Which of the following terms related to aphasia refers to the failure to recognize familiar objects perceived by the senses?

A. Agnosia-
Auditory agnosia is failure to recognize significance of sounds.

B. Agraphia- Agraphia refers to disturbances in writing intelligible words.

C. Apraxia- Apraxia refers to inability to perform previously learned purposeful motor acts on a voluntary basis.

D. Perseveration- Perseveration is the continued and automatic repetition of an activity, word, or phrase that is no longer appropriate.

381. Which of the following terms related to aphasia refers to difficulty reading?

A. Alexia- Alexia or dyslexia may occur in the absence of aphasia.

B. Agnosia- Agnosia is failure to recognize familiar objects perceived by the senses.

C. Agraphia- Agraphia refers to disturbances in writing intelligible words.

D. Perseveration- Perseveration is the continued and automatic repetition of an activity, word, or phrase that is no longer appropriate.

382. Which of the following terms related to aphasia refers to difficulty in selecting appropriate words, particularly nouns?

A. Anomia

B. Acalculia- Acalculia refers to difficulty in dealing with mathematical processes or numerical symbols in general.

C. Dysarthria- Dysarthria refers to defects of articulation due to neurologic causes.

D. Paraphasia- Paraphasia refers to using wrong words, word substitutions, and faults in word usage in both oral and written language.

383. A patient has had neurologic deficits lasting for more than 24 hours, and now the symptoms are resolving. The nurse concludes that the patient has had which type of stroke?

A. Reversible ischemic neurologic deficit- With a reversible ischemic neurologic deficit, the patient has more pronounced signs and symptoms that last more than 24 hours; symptoms resolve in a matter of days without any permanent neurologic deficit.

B. Transient ischemic attack (TIA)- With a TIA, the patient has a temporary episode of neurologic dysfunction that may last a few seconds or minutes but not longer than 24 hours.

C. Stroke in evolution- With a stroke in evolution the patient experiences a worsening of neurological signs and symptoms over several minutes or hours; it is a progressing stroke.

D. Completed stroke- With a completed stroke, the patient's neurological signs and symptoms have stabilized with no indication of further progression of the hypoxic insult to the brain.

384. Which of the following is a modifiable risk factor for transient ischemic attacks and ischemic strokes?

A. History of smoking.- Modifiable risk factors for TIAs and ischemic stroke include hypertension, Type 1 diabetes, cardiac disease, history of smoking, and chronic alcoholism.

B. Thyroid disease- Hypertension, Type 1 diabetes, and cardiac disease are modifiable risk factors for TIAs and ischemic stroke.

C. Social drinking- Chronic alcoholism is a modifiable risk factor for TIAs and ischemic stroke.

D. Advanced age- Advanced age, gender, and race are non-modifiable risk factors for stroke.

385. A patient who has had a previous stroke and is taking warfarin tells the nurse that he started taking garlic to help reduce his blood pressure. The nurse knows that garlic when taken together with warfarin

A. can greatly increase the international normalization ratio (INR) and therefore increase the risk of bleeding.-
Garlic and warfarin taken together can greatly increase the INR, increasing the risk of bleeding.

B. have no drug-drug interactions and therefore may be taken together.- Garlic and warfarin taken together can greatly increase the INR, increasing the risk of bleeding.

C. can cause platelet aggregation and therefore increase the risk of blood clotting.
Garlic and warfarin taken together can greatly increase the INR, increasing the risk of bleeding.

D. may increase cerebral blood flow causing migraine headaches. - Garlic and warfarin taken together can greatly increase the INR, increasing the risk of bleeding.

386. Later signs of increased intracranial pressure (ICP) later include which of the following?

A. Projectile vomiting-
Projectile vomiting may occur with increased pressure on the reflex center in the medulla.

B. Increased pulse rate- As ICP increases, the pulse rate decreases.

C. Decreased blood pressure- As ICP increases, the blood pressure increases.

D. Narrowed pulse pressure- As ICP increases, the pulse pressure (the difference between the systolic and the diastolic pressure) widens.

387. Bleeding between the dura mater and arachnoid membrane is termed

A. subdural hematoma.- A subdural hematoma is bleeding between the dura mater and arachnoid membrane.

B. intracerebral hemorrhage.- Intracerebral hemorrhage is bleeding in the brain or the cerebral tissue with displacement of surrounding structures.

C. epidural hematoma.- An epidural hematoma is bleeding between the inner skull and the dura, compressing the brain underneath.

D. extradural hematoma.- An extradural hematoma is another name for an epidural hematoma.

388. Which of the following statements reflect nursing management of the patient with expressive aphasia?

A. Encourage the patient to repeat sounds of the alphabet. -
Nursing management of the patient with expressive aphasia includes encouraging the patient to repeat sounds of the alphabet.

B. Speak clearly and in simple sentences; use gestures or pictures when able.
Nursing management of the patient with global aphasia includes speaking clearly and in simple sentences and using gestures or pictures when able.

C. Speak slowly and clearly to assist the patient in forming the sounds. - Nursing management of the patient with receptive aphasia includes speaking slowing and clearly to assist the patient in forming the sounds.

D. Frequently reorient the patient to time, place, and situation. - Nursing management of the patient with cognitive deficits, such as memory loss, includes frequently reorienting the patient to time, place, and situation.

389. Health promotion efforts to decrease the risk for ischemic stroke involve encouraging a healthy lifestyle including

A. a low fat, low cholesterol diet, and increasing exercise.-
Health promotion efforts to decrease the risk for ischemic stroke involve encouraging a healthy lifestyle including a low fat, low cholesterol diet, and increasing exercise.

B. eating fish no more than once a month. - Recent evidence suggests that eating fish two or more times per week reduces the risk of thrombotic stroke for women

C. a high protein diet and increasing weight-bearing exercise. - Health promotion efforts to decrease the risk for ischemic stroke involve encouraging a healthy lifestyle including a low fat, low cholesterol diet, and increasing exercise.

D. a low cholesterol, low protein diet, and decreasing aerobic exercise. -Health promotion efforts to decrease the risk for ischemic stroke involve encouraging a healthy lifestyle including a low fat, low cholesterol diet, and increasing exercise.

390. Before the patient diagnosed with a concussion is released from the Emergency Department, the nurse teaches the family or friends who will be tending to the patient to contact the physician or return to the ED if the patient

A. vomits.
- Vomiting is a sign of increasing intracranial pressure and should be reported immediately.

B. complains of headache.- In general, the finding of headache in the patient with a concussion is an expected abnormal observation. However, severe headache should be reported or treated immediately.

C. complains of generalized weakness.- Weakness of one side of the body should be reported or treated immediately.

D. sleeps for short periods of time.- Difficulty in waking the patient should be reported or treated immediately.

391. When the nurse reviews the physician's progress notes for the patient who has sustained a head injury and sees that the physician observed Battle's sign when the patient was in the Emergency Department, the nurse knows that the physician observed

A. an area of bruising over the mastoid bone.-
Battle's sign may indicate skull fracture.

B. a bloodstain surrounded by a yellowish stain on the head dressing.- A bloodstain surrounded by a yellowish stain on the head dressing is referred to as a halo sign and is highly suggestive of a cerebrospinal fluid leak.

C. escape of cerebrospinal fluid (CSF) from the patient's ear.- Escape of CSF from the patient's ear is termed otorrhea.

D. escape of cerebrospinal fluid (CSF) from the patient's nose.- Escape of CSF from the patient's nose is termed rhinorrhea.

392. Which of the following findings in the patient who has sustained a head injury indicate increasing intracranial pressure (ICP)?

A. Widened pulse pressure
- Additional signs of increasing ICP include increasing systolic blood pressure, bradycardia, rapid respirations, and rapid rise in body temperature.

B. Increased pulse- Bradycardia, slowing of the pulse, is an indication of increasing ICP in the head-injured patient.

C. Decreased respirations- Rapid respiration is an indication of increasing ICP in the head-injured patient.

D. Decreased body temperature- A rapid rise in body temperature is regarded as unfavorable because hyperthermia may indicate brain stem damage, a poor prognostic sign.

393. Which of the following nursing interventions is appropriate when caring for the awake and oriented head injury patient?

A. Supply oxygen therapy to keep blood gas values within normal range.-The goal is to keep blood gas values within normal range to ensure adequate cerebral circulation.

B. Do not elevate the head of the bed.- In general, the head of the bed is elevated about 30 degrees to decrease intracranial venous pressure.

C. Encourage the patient to cough every 2 hours.- Coughing should not be encouraged because it increases intracranial pressure.

D. Use restraints if the patient becomes agitated.- Restraints should be avoided because straining against them can increase intracranial pressure. Use of padded side rails and application of mitts are the appropriate interventions in the agitated head-injured patient.

394. Of the following stimuli, which is known to trigger an episode of autonomic hyperreflexia in the patient who has suffered a spinal cord injury?

A. Applying a blanket over the patient.-
An object on the skin or skin pressure may precipitate an autonomic hyperreflexic episode.

B. Diarrhea- In general, constipation or fecal impaction triggers autonomic hyperreflexia.

C. Placing the patient in a sitting position- When the patient is observed to be demonstrating signs of autonomic hyperreflexia, he is placed in a sitting position immediately to lower blood pressure.

D. Voiding- The most common cause of autonomic hyperreflexia is a distended bladder.

395. Risk factors that increase the likelihood of post-traumatic seizures following a head injury include which of the following?

A. Age over 65 years-
Risk factors that increase the likelihood of post-traumatic seizures following a head injury include brain contusion with subdural hematoma, skull fracture, loss of consciousness or amnesia of 1 day or more, and age over 65 years.

B. Loss of consciousness for less than 1 day- Loss of consciousness or amnesia of 1 day or more is a risk factor that increases the likelihood of post-traumatic seizures following a head injury

C. Glasgow Coma Scale (GCS) score less than 10- The GCS assesses level of consciousness; a score of 10 or less indicates the need for emergency attention. It is not a risk factor for post-traumatic seizures.

D. Epidural hematoma- Brain contusion with subdural hematoma is a risk factor that increases the likelihood of post-traumatic seizures following a head injury

396. A post-traumatic seizure classified as early occurs

A. within 1-7 days of injury. -Posttraumatic seizures are classified as immediate (occurring within 24 hours of injury), early, (occurring within 1-7 days of injury) or late, occurring more than 7 days following injury.

B.. within 4 hours of injury. - Posttraumatic seizures are classified as immediate (occurring within 24 hours of injury), early, (occurring within 1-7 days of injury) or late, occurring more than 7 days following injury.

C. within 24 hours of injury.- Posttraumatic seizures occurring within 24 hours of injury are classified as immediate seizures.

D. more than 7 days following surgery. - Posttraumatic seizures occurring more than 7 days following surgery are classified as late seizures.

397. The nurse assesses the dressing of a patient with a basal skull fracture and sees the halo sign - a blood stain surrounded by a yellowish stain. The nurse knows that this sign

A. is highly suggestive of a cerebrospinal fluid (CSF) leak. The halo sign - a blood stain surrounded by a yellowish stain is highly suggestive of a cerebrospinal fluid (CSF) leak.

B. may indicate a subdural hematoma..-The halo sign is highly suggestive of a cerebrospinal fluid (CSF) leak.

C. is highly suggestive of a cerebral contusion. -The halo sign is highly suggestive of a cerebrospinal fluid (CSF) leak.

D. normally occurs within 24 hours following a basal skull fracture.-The halo sign is highly suggestive of a cerebrospinal fluid (CSF) leak.

398. A Glasgow Coma Scale (GCS) score of 7 or less is generally interpreted as

A. coma.
- The Glasgow Coma Scale (GCS) is a tool for assessing a patient's response to stimuli. A score of 7 or less is generally interpreted as coma.

B. a need for emergency attention. - A GCS score of 10 or less indicates a need for emergency attention.

C. least responsive.- A GCS score of 3 is interpreted as least responsive.

D. most responsive.- A GCS score of 15 is interpreted as most responsive.

399. Which of the following terms refers to muscular hypertonicity with increased resistance to stretch?

A. Spasticity- Spasticity is often associated with weakness, increased deep tendon reflexes, and diminished superficial reflexes.

B. Akathesia- Akathesia refers to a restless, urgent need to move around and agitation.

C. Ataxia- Ataxia refers to impaired ability to coordinate movement.

D. Myclonus- Myoclonus refers to spasm of a single muscle or group of muscles.

400. Of the following terms, which refers to blindness in the right or left halves of the visual fields of both eyes?

A. Homonymous hemianopsia-Homonymous hemianopsia occurs with occipital lobe tumors.

B. Scotoma- Scotoma refers to a defect in vision in a specific area in one or both eyes.

C. Diplopia-Diplopia refers to double vision or the awareness of two images of the same object occurring in one or both eyes.

D. Nystagmus- Nystagmus refers to rhythmic, involuntary movements or oscillations of the eyes.

401. Which of the following terms is used to describe rapid, jerky, involuntary, purposeless movements of the extremities?

A. Chorea- Choreiform movements, such as grimacing, may also be observed in the face.

B. Bradykinesia- Bradykinesia refers to very slow voluntary movements and speech.

C. Dyskinesia- Dyskinesia refers to impaired ability to execute voluntary movements.

D. Spondylosis- Spondylosis refers to degenerative arthritis of the cervical or lumbar vertebrae.

402. Which of the phases of a migraine headache usually lasts less than an hour?

A. Aura-
The aura phase occurs in about 20% of patients who have migraines and may be characterized by focal neurological symptoms.

B. Prodrome- The prodrome phase occurs hours to days before a migraine headache.

C. Headache- The headache phase lasts from 4 to 72 hours.

D. Recovery- During the post-headache phase, patients may sleep for extended periods.

403. The most common type of brain neoplasm is the

A. glioma.
- Gliomas are the most common brain neoplasms, accounting for about 45% of all brain tumors.

B. angioma.- Angiomas account for approximately 4% of brain tumors.

C. meningioma.- Meningiomas account for 15-20% of all brain tumors.

D. neuroma.- Neuromas account for 7% of all brain tumors.

404. Which of the following diseases is a chronic, degenerative, progressive disease of the central nervous system characterized by the occurrence of small patches of demyelination in the brain and spinal cord?

A. Multiple sclerosis
- The cause of MS is not known and the disease affects twice as many women as men.

B. Parkinson's disease- Parkinson's disease is associated with decreased levels of dopamine caused by destruction of pigmented neuronal cells in the substantia nigra in the basal ganglia of the brain.

C. Huntington's disease- Huntington's disease is a chronic, progressive, hereditary disease of the nervous system that results in progressive involuntary dance-like movement and dementia.

D. Creutzfeldt-Jakob's disease- Creutzfeldt-Jakob's disease is a rare, transmissible, progressive fatal disease of the central nervous system characterized by spongiform degeneration of the gray matter of the brain.

405. Which of the following diseases is associated with decreased levels of dopamine due to destruction of pigmented neuronal cells in the substantia nigra in the basal ganglia of the brain?

A. Parkinson's disease- In some patients, Parkinson's disease can be controlled; however, it cannot be cured.

B. Multiple sclerosis- Multiple sclerosis is a chronic, degenerative, progressive disease of the CNS characterized by the occurrence of small patches of demyelination in the brain and spinal cord.

C. Huntington's disease- Huntington's disease is a chronic, progressive, hereditary disease of the nervous system that results in progressive involuntary dance-like movement and dementia.

D. Creutzfeldt-Jakob's disease- Creutzfeldt-Jakob's disease is a rare, transmissible, progressive fatal disease of the central nervous system characterized by spongiform degeneration of the gray matter of the brain.

406. Which of the following diseases is a chronic, progressive, hereditary disease of the nervous system that results in progressive involuntary dance-like movement and dementia?

A. Huntington's disease- Because it is transmitted as an autosomal dominant genetic disorder, each child of a parent with HD has a 50% risk of inheriting the illness.

B. Multiple sclerosis- Multiple sclerosis is a chronic, degenerative, progressive disease of the CNS characterized by the occurrence of small patches of demyelination in the brain and spinal cord.

C. Parkinson's disease- Parkinson's disease is associated with decreased levels of dopamine due to destruction of pigmented neuronal cells in the substantia nigra in the basal ganglia of the brain.

D. Creutzfeldt-Jakob's disease- Creutzfeldt-Jakob's disease is a rare, transmissible, progressive fatal disease of the central nervous system characterized by spongiform degeneration of the gray matter of the brain.

407. Which of the following diseases is a rare, transmissible, progressive fatal disease of the central nervous system characterized by spongiform degeneration of the gray matter of the brain?

A. Creutzfeldt-Jakob's disease
- The disease causes severe dementia and myoclonus.

B. Multiple sclerosis- Multiple sclerosis is a chronic, degenerative, progressive disease of the CNS characterized by the occurrence of small patches of demyelination in the brain and spinal cord.

C. Parkinson's disease- Parkinson's disease is associated with decreased levels of dopamine due to destruction of pigmented neuronal cells in the substantia nigra in the basal ganglia of the brain.

D. Huntington's disease- Huntington's disease is a chronic, progressive, hereditary disease of the nervous system that results in progressive involuntary dance-like movement and dementia.

408. Bell's palsy is a disorder of which cranial nerve?

A. Facial (VII)- Bell's palsy is characterized by facial dysfunction, weakness, and paralysis

B. Trigeminal (V)- Trigeminal neuralgia is a disorder of the trigeminal nerve and causes facial pain.

C. Vestibulocochlear (VIII)- Meniere's syndrome is a disorder of the vestibulocochlear nerve.

D. Vagus (X)- Guillain-Barre syndrome is a disorder of the vagus nerve.

409. The most common cause of acute encephalitis in the United States is

A. Herpes Simplex Virus (HSV).- Viral infection is the most common cause of encephalitis. HSV is the most common cause of acute encephalitis in the U.S.

B.Cryptococcus neoformans.- C. neoformans is one of several fungi that may cause fungal encephalitis. Fungal infections of the central nervous system occur rarely in healthy people.

C. Western equine bacteria.-The Western equine encephalitis virus is one of four types of arboviral encephalitis that occur in North America.

D. Candida albicans.- C. albicans is one of several fungi that may cause fungal encephalitis. Fungal infections of the central nervous system occur rarely in healthy people.

410. Which of the following reflects basic nursing measures in the care of the patient with viral encephalitis?

A. Providing comfort measures - Providing comfort measures directed at the headache, include dimmed lights, limited noise, and analgesics are the basic nursing measures in the care of the patient with a viral encephalitis.

B. Administering narcotic analgesics- Narcotic analgesics may mask neurologic symptoms; therefore, they are used cautiously.

C. Administering amphotericin B.- With viral encephalitis, acyclovir therapy is commonly prescribed; Amphotericin B is used in the treatment of fungal encephalitis.

D. Monitoring cardiac output- Nursing management of the patient with viral encephalitis includes monitoring of blood chemistry test results and urinary output to alert the nurse to the presence of renal complications related to acyclovir therapy.


411. Nursing management of the patient with new variant Creutzfeldt-Jakob Disease (nvCJD) includes

A. providing supportive care.- The nvCJD is a progressive fatal disease with no treatment available. Due to the fatal outcome of nvCJD, nursing care is primarily supportive.

B. initiating isolation procedures.- Prevention of disease transmission is an important part of providing nursing care. Although patient isolation is not necessary, use of standard precautions is important. Institutional protocols are followed for blood and body fluid exposure and decontamination of equipment.

C. preparing for organ donation.-Organ donation is not an option because of the risk for disease transmission.

D. administering amphotericin B.- Amphotericin B is used in the treatment of fungal encephalitis; no treatment is available for nvCJD.

412. Three medications referred to as the 'ABC drugs' are currently the main pharmacological therapy for multiple sclerosis. Which of the following statements reflects information to be included in patient teaching?

A. Flu-like symptoms can be controlled with nonsteroidal anti-inflammatory drugs (NSAIDs) and usually resolve after a few months of therapy.-
Seventy-five percent of patients taking one of the interferons experience flu-like symptoms that can be controlled with NSAIDS and usually resolve after a few months of therapy.

B. Take interferon beta-la (Avonex) with food or milk. - Interferon beta-la is given by intramuscular injection once a week.

C. Take interferon beta-1b (Betaseron) at night before bedtime for best effects.
Interferon beta-1b is administered subcutaneously once a week.

D. Take glatiramer acetate (Copaxone) on an empty stomach.- Glatiramer acetate is administed by intramuscular injection once a week.

413. Korsakoff's syndrome is characterized by

A. psychosis, disorientation, delirium, insomnia, and hallucinations. - Korsakoff's syndrome is a personality disorder characterized by psychosis, disorientation, delirium, insomnia, and hallucinations.

B. severe dementia and myocLonus. - Creutzefeldt-Jacob disease results in severe dementia and myoclonus.

C. tremor, rigidity, and bradykinesia. - The three cardinal signs of Parkinson's disease are tremor, rigidity, and bradykinesia.

D. choreiform movement and dementia. - Huntington's disease results in progressive involuntary choreiform (dancelike) movement and dementia.

414. The primary North American vector transmitting arthropod-borne virus encephalitis is the

A. mosquito-
Arthropod vectors transmit several types of viruses that cause encephalitis. The primary vector in North America is the mosquito.

B. tick.- The primary vector in North America is the mosquito.

C. horse. - The primary vector in North America is the mosquito.

D. flea.- The primary vector in North America is the mosquito.

415. The initial symptoms of new variant Creutzfeldt-Jakob Disease (nvCJD) are

A. anxiety, depression, and behavioral changes. - Anxiety, depression, and behavioral changes are the initial symptoms of nvCJD

B. memory and cognitive impairment.- Memory and cognitive impairment occur late in the course of nvCJD

C. diplopia and bradykinesia.- Anxiety, depression and behavioral changes are the initial symptoms of nvCJD

D. akathisia and dysphagia.- Anxiety, depression and behavioral changes are the initial symptoms of nvCJD

416. A patient with fungal encephalitis receiving amphotericin B complaints of fever, chills, and body aches. The nurse knows that these symptoms

A. may be controlled by the administration of diphenhydramine (Benedryl) and
acetaminophen (Tylenol) approximately 30 minutes prior to administration of the amphotericin. -Administration of amphotericin B may cause fever, chills and body aches. The administration of diphenhydramine (Benedryl) and acetaminophen (Tylenol) approximately 30 minutes prior to the administration of amphotericin B may prevent these side effects.

B. indicate renal toxicity and a worsening of the patient's condition. - Renal toxicity due to amphotericin B is dose limiting. Monitoring the serum creatinine and blood urea nitrogen levels may alert the nurse to the development of renal insufficiency and the need to address the patients’ renal status.

C. are primarily associated with infection with Coccidioides immitis and Aspergillus.
Vascular changes are associated with C. immitis and Aspergillus Manifestations of vascular change may include arteritis or cerebral infarction.

D. indicate the need for immediate blood and cerebral spinal fluid (CSF) cultures.
Blood and CSF cultures help diagnosis fungal encephalitis.

417. The patient with Herpes Simplex Virus (HSV) encephalitis is receiving acyclovir (Zovirax). The nurse monitors blood chemistry test results and urinary output for

A. renal complications related to acyclovir Therapy. Monitoring of blood chemistry test results and urinary output will alert the nurse to the presence of renal complications related to acyclovir therapy.

B. signs and symptoms of cardiac insufficiency. -Monitoring of blood chemistry test results and urinary output will alert the nurse to the presence of renal complications related to acyclovir therapy.

C. signs of relapse.- Monitoring of blood chemistry test results and urinary output will alert the nurse to the presence of renal complications related to acyclovir therapy. To prevent relapse treatment with acyclovir should continue for up to 3 weeks.

D. signs of improvement in the patient's condition.
Monitoring of blood chemistry test results and urinary output will alert the nurse to the presence of renal complications related to acyclovir therapy.

418. Medical management of arthropod-borne virus (arboviral) encephalitis is aimed at

A. controlling seizures and increased intracranical pressure. There is no specific medication for arboviral encephalitis. Medical management is aimed at controlling seizures and increased intracranial pressure.

B. preventing renal insufficiency.- Medical management is aimed at controlling seizures and increased intracranial pressure.

C. maintaining hemodynamic stability and adequate cardiac output. - Medical management is aimed at controlling seizures and increased intracranial pressure.

D. preventing muscular atrophy. - Medical management is aimed at controlling seizures and increased intracranial pressure.

419. The patient receiving mitoxantrone (Novantrone) for treatment of secondary progressive multiple sclerosis (MS) is closely monitored for

A. leukopenia and cardiac toxicity. -Mitoxantrone is an antineoplastic agent used primarily to treat leukemia and lyphoma but is also used to treat secondary progressive MS. Patients need to have laboratory tests ordered and the results closely monitored due to the potential for leukopenia and cardiac toxicity.

B. mood changes and fluid and electrolyte alterations. - Patients receiving corticosteroids are monitored for side effects related to corticosteroids such as mood changes and fluid and electrolyte alterations.

C. renal insufficiency.- Patients receiving mitoxantrone are closely monitored for leukopenia and cardiac toxicity.

D. hypoxia.- Patients receiving mitoxantrone are closely monitored for leukopenia and cardiac toxicity.

420. What percentage of patients who survived the polio epidemic of the 1950s are now estimated to have developed post-polio syndrome?

A. 60-80%-
Patients who survived the polio epidemic of the 1950s, many now elderly, are developing new symptoms of weakness, fatigue and musculoskeletal pain. It is estimated that between 60% and 80% of the 640,000 polio survivors are experiencing the phenomenon known as post-polio syndrome.

B. 50%- It is estimated that between 60 and 80% of patients who survived the polio epidemic of the 1950s are now experiencing post-polio syndrome.

C. 25-30%- It is estimated that between 60 and 80% of patients who survived the polio epidemic of the 1950s are now experiencing post-polio syndrome.

D. 10% It is estimated that between 60 and 80% of patients who survived the polio epidemic of the 1950s are now experiencing post-polio syndrome.

421. Which of the following statements describe the pathophysiology of post-polio syndrome?

A. The exact cause is unknown, but aging or muscle overuse is suspected. -The exact cause of post-polio syndrome is not known but researchers suspect that with aging or muscle overuse the neurons not destroyed originally by the poliovirus are unable to continue generating axon sprouts.

B. The exact cause is unknown, but latent poliovirus is suspected.- The exact cause of post-polio syndrome is not known.

C. Post-polio syndrome is caused by an autoimmune response. -The exact cause of post-polio syndrome is not known.

D. Post-polio syndrome is caused by long-term intake of a low-protein, high-fat diet in polio survivors.- The exact cause of post-polio syndrome is not known.

422. Which of the following statements reflect nursing interventions of a patient with post-polio syndrome?

A. No specific medical or surgical treatment is available for this syndrome and therefore nursing plays a pivotal role in the team approach to assisting patients and families in dealing with the symptoms of progressive loss of muscle strength and significant fatigue. Nursing interventions are aimed at slowing the loss of strength and maintaining the physical, psychological and social well being of the patient.

B. Administering antiretroviral agents. - No specific medical or surgical treatment is available for this syndrome.

C. Planning activities for evening hours rather then morning hours. - Patients need to plan and coordinate activities to conserve energy and reduce fatigue. Important activities should be planned for the morning as fatigue often increases in the afternoon and evening.

D. Avoiding the use of heat applications in the treatment of muscle and joint pain. Pain in muscles and joints may be a problem. Nonpharmacologic techniques such as the application of heat and cold are most appropriate because these patients tend to have strong reactions to medications.

423. Which of the following terms is used to describe edema of the optic nerve?

A. Papilledema- Papilledema is edema of the optic nerve.

B. Scotoma- Scotoma is a defect in vision in a specific area in one or both eyes.

C. Lymphedema- Lymphedema is the chronic swelling of an extremity due to interrupted lymphatic ciruclation, typically from an axillary dissection.

D. Angioneurotic edema - Angioneurotic edema is a condition characterized by urticaria and diffuse swelling of the deeper layers of the skin.

424.Degenerative neurologic disorders include which of the following?

A. Huntington's disease- Huntington's disease is a chronic, progressive, degenerative neurologic hereditary disease of the nervous system that results in progressive involuntary choreiform movement and dementia.

B. Paget's disease- Paget's disease is a musculoskeletal disorder, characterized by localized rapid bone turnover, most commonly affecting the skull, femur, tibia, pelvic bones, and vertebrae.

C. Osteomyelitis- Osteomyelitis is an infection of the bone.

D. Glioma- Malignant glioma is the most common type of brain tumor.

425. Bone density testing in patients with post-polio syndrome has demonstrated

A. low bone mass and osteoporosis. –Bone density testing in patients with post-polio syndrome has demonstrated low bone mass and osteoporosis. Thus, the importance of identifying risks, preventing falls, and treating osteoporosis must be discussed with patients and their families.

B. osteoarthritis.- Bone density testing in patients with post-polio syndrome has demonstrated low bone mass and osteoporosis.

C. calcification of long bones.- Bone density testing in patients with post-polio syndrome has demonstrated low bone mass and osteoporosis.

D. no significant findings.- Bone density testing in patients with post-polio syndrome has demonstrated low bone mass and osteoporosis.


426. Which of the following terms refers to mature compact bone structures that form concentric rings of bone matrix?

A. Lamellae-
Lamellae are mineralized bone matrix.

B. Endosteum- Endosteum refers to the marrow cavity lining of hollow bone.

C. Trabecula- Trabecula refers to lattice-like bone structure.

D. Cancellous bone- Cancellous bone refers to spongy, lattice-like bone structure.

427. An osteon is defined as a

A. microscopic functional bone unit.- The center of an osteon contains a capillary.

B. bone-forming cell.- An osteoblast is a bone-forming cell.

C. bone resorption cell.- An osteoclast is a bone resorption cell.

D. mature bone cell.- An osteocyte is a mature bone cell.

428. Which of the following terms refers to the shaft of the long bone?

A. Diaphysis- The diaphysis is primarily cortical bone.

B. Epiphysis- An epiphysis is an end of a long bone.

C. Lordosis- Lordosis refers to an increase in lumbar curvature of spine.

D. Scoliosis- Scoliosis refers to lateral curving of the spine.

429. Paresthesia is the term used to refer t

A. abnormal sensations.- Abnormal sensations, such as burning, tingling, and numbness, are referred to as paresthesias.

B. absence of muscle movement suggesting nerve damage.- The absence of muscle tone suggesting nerve damage is referred to as paralysis.

C. involuntary twitch of muscle fibers.
Involuntary twitch of muscle fibers is referred to as fasciculation.

D. absence of muscle tone.
A muscle which holds no tone is termed flaccid.

430. Which of the following terms refers to a grating or crackling sound or sensation?

A. Crepitus- Crepitus may occur with movement of ends of a broken bone or irregular joint surface.

B. Callus- Callus is fibrous tissue that forms at the fracture site.

C. Clonus- Clonus refers to rhythmic contraction of muscle.

D. Fasciculation- Fasciculation refers to involuntary twitch of muscle fibers.

431. Which of the following terms refers to muscle tension being unchanged with muscle shortening and joint motion?

A. Isotonic contraction- Exercises such as swimming and bicycling are isotonic.

B. Isometric contraction- Isometric contraction is characterized by increased muscle tension, unchanged muscle length, and no joint motion.

C. Contracture- Contracture refers to abnormal shortening of muscle, joint, or both.

D. Fasciculation- Fasciculation refers to involuntary twitch of muscle fibers.

432. During which stage or phase of bone healing after fracture does callus formation occur?

A. Reparative-
Callus formation occurs during the reparative stage but is disrupted by excessive motion at the fracture site

B. Remodeling- Remodeling is the final stage of fracture repair during which the new bone is reorganized into the bone's former structural arrangement.

C. Inflammation-NDuring inflammation, macrophages invade and debride the fracture area.

D. Revascularization- Revascularization occurs within about 5 days after the fracture.

433. During which stage or phase of bone healing after fracture is devitalized tissue removed and new bone reorganized into its former structural arrangement?

A. Remodeling-
Remodeling is the final stage of fracture repair.

B. Inflammation- During inflammation, macrophages invade and debride the fracture area.

C. Revascularization - Revascularization occurs within about 5 days after the fracture.

D. Reparative Callus formation occurs during the reparative stage but is disrupted by excessive motion at the fracture site.

434. Which nerve is assessed when the nurse asks the patient to spread all fingers?

A. Ulnar - Asking the patient to spread all fingers allows the nurse to assess motor function affected by ulnar innervation while pricking the fat pad at the top of the small finger allows assessment of the sensory function affected by the ulnar nerve.

B. Peroneal- The peroneal nerve is assessed by asking the patient to dorsiflex the ankle and extend the toes.

C. Radial- The radial nerve is assessed by asking the patient to stretch out the thumb, then the wrist, and then the fingers at the metacarpal joints.

D. Median- The median nerve is assessed by asking the patient to touch the thumb to the little finger.

435. Which nerve is assessed when the nurse asks the patient to dorsiflex the ankle and extend the toes?

A. Peroneal- The motor function of the peroneal nerve is assessed by asking the patient to dorsiflex the ankle and extend the toes while the sensory function is assessed by pricking the skin between the great and center toes.

B. Radial- The radial nerve is assessed by asking the patient to stretch out the thumb, then the wrist, and then the fingers at the metacarpal joints.

C. Median- The median nerve is assessed by asking the patient to touch the thumb to the little finger.

D. Ulnar- Asking the patient to spread all fingers allows the nurse to assess motor function affected by ulnar innervation.

436. Which of the following statements reflect the progress of bone healing?

A. Serial x-rays are used to monitor the progress of bone healing.-Serial x-rays are used to monitor the progress of bone healing.

B All fracture healing takes place at the same rate no matter the type of bone fractured. - The type of bone fractured, the adequacy of blood supply, the surface contact of the fragments, and the general health of the person influence the rate of fracture healing.

C. The age of the patient influences the rate of fracture healing.- The type of bone fractured, the adequacy of blood supply, the surface contact of the fragments, and the general health of the person influence the rate of fracture healing.

D. Adequate immobilization is essential until there is ultrasound evidence of bone formation with ossification.- Adequate immobilization is essential until there is x-ray evidence of bone formation with ossification.

437. Diminished range of motion, loss of flexibility, stiffness, and loss of height are history and physical findings associated with age-related changes of the

A. joints. History and physical findings associated with age-related changes of the joints include diminished range of motion, loss of flexibility, stiffness, and loss of height.

B. bones.- History and physical findings associated with age-related changes of bones include loss of height, posture changes, kyphosis, flexion of hips and knees, back pain, osteoporosis, and fracture.

C. muscles. - History and physical findings associated with age-related changes of muscles include loss of strength, diminished agility, decreased endurance, prolonged response time (diminished reaction time), diminished tone, a broad base of support, and a history of falls.

D. ligaments.- History and physical findings associated with age-related changes of ligaments include joint pain on motion that resolves with rest, crepitus, joint swelling/enlargement, and degenerative joint disease (osteoarthritis).

438. Fracture healing occurs in four areas, including the

A. external soft tissue.- Fracture healing occurs in four areas, including the bone marrow, bone cortex, periosteum, and the external soft tissue, where a bridging callus (fibrous tissue) stabilizes the fracture.

B. cartilage.- Fracture healing occurs in four areas, including the bone marrow, bone cortex, periosteum, and the external soft tissue. Cartilage is special tissue at the ends of bone.

C. bursae.- Fracture healing occurs in four areas, including the bone marrow, bone cortex, periosteum, and the external soft tissue. The bursae is a fluid-filled sac found in connective tissue, usually in the area of joints.

D. fascia.- Fracture healing occurs in four areas, including the bone marrow, bone cortex, periosteum, and the external soft tissue. Fascia is fibrous tissue that covers, supports, and separates muscles.

439. Which of the following is an indicator of neurovascular compromise?

A. Capillary refill more than 3 seconds - Capillary refill more than 3 seconds is an indicator of neurovascular compromise. Other indicators include cool skin temperature, pale or cyanotic color, weakness, paralysis, paresthesia, unrelenting pain, pain on passive stretch, and absence of feeling.

B. Warm skin temperature- Cool skin temperature is an indicator of neurovascular compromise.

C. Diminished pain- Unrelenting pain is an indicator of neurovascular compromise.

D. Pain on active stretch. - Pain on passive stretch is an indicator of neurovascular compromise.

440. Which of the following terms refers to moving away from midline?

A. Abduction- Abduction is moving away from midline.

B. Adduction- Adduction is moving toward midline.

C. Inversion- Inversion is turning inward.

D. Eversion - Eversion is turning outward.

441. Surgical fusion of a joint is termed

A. arthrodesis.- Arthrodesis of a joint is created surgically to treat chronic pain.

B. open reduction with internal fixation (ORIF).- ORIF refers to surgery to repair and stabilize a fracture.

C. heterotrophic ossification.- Heterotrophic ossification refers to formation of bone in the periprosthetic space.

D. arthroplasty.- Arthroplasty refers to surgical repair of a joint or joint replacement.

442. Which of the following devices is designed specifically to support and immobilize a body part in a desired position?

A.Splint-
A splint may be applied to a fractured extremity initially until swelling subsides.

B. Brace- A brace is an externally applied device to support a body part, control movement, and prevent injury.

C. Continuous passive motion (CPM) device- A CPM device is an instrument that moves a body part to promote healing and circulation.

D. Trapeze- A trapeze is an overhead patient-helping device to promote patient mobility in bed.

443. When caring for the patient in traction, the nurse is guided by which of the following principles?

A. Skeletal traction is never interrupted.-
Skeletal traction is applied directly to the bone and is never interrupted.

B. Weights should rest on the bed.- In order to be effective, weights must hang freely and not rest on the bed or floor.

C. Knots in the ropes should touch the pulley.- Knots in the rope or the footplate must not touch the pulley or the foot of the bed.

D. Weights are removed routinely.- Traction must be continuous to be effective in reducing and immobilizing fractures.

444. Meniscectomy refers to the

A. replacement of one of the articular surfaces of a joint.
The most common site for meniscectomy is the knee.

B. incision and diversion of the muscle fascia.- Fasciotomy refers to the incision and diversion of the muscle fascia to relieve muscle constriction.

C. excision of damaged joint fibrocartilage.- Hemiarthroplsty refers to the replacement of one of the articular surfaces of a joint.

D. removal of a body part.- Amputation refers to the removal of a body part.

445. In order to avoid hip dislocation after replacement surgery, the nurse teaches the patient which of the following guidelines?

A. Never cross the affected leg when seated.- Crossing the affected leg may result in dislocation of the hip joint after total hip replacement.

B. Keep the knees together at all times.- The patient should be taught to keep the knees apart at all times.

C. Avoid placing a pillow between the legs when sleeping.- The patient should be taught to put a pillow between the legs when sleeping.

D. Bend forward only when seated in a chair.- The patient should be taught to avoid bending forward when seated in a chair.

446.Injury to the ______ nerve as a result of pressure is a cause of footdrop.

A. Peroneal- Injury to the peroneal nerve as a result of pressure is a cause of footdrop.

B. Sciatic- Injury to the peroneal nerve as a result of pressure is a cause of footdrop.

C. Femoral- Injury to the peroneal nerve as a result of pressure is a cause of footdrop.

D. Achilles- Injury to the peroneal nerve as a result of pressure is a cause of footdrop.

447.The nurse teaching the patient with a cast about home care includes which of the following instructions?

A. Dry a wet fiberglass cast thoroughly using a hair dryer on a cool setting to avoid
skin problems. - Instruct the patient to keep the cast dry and to dry a wet fiberglass cast thoroughly using a hair dryer on a cool setting to avoid skin problems; do not cover it with plastic or rubber.

B. Cover the cast with plastic or rubber. - A cast should be kept dry; do not cover it with plastic or rubber because this causes condensation, which dampens the cast and skin.

C. Keep the cast below heart level. - A casted extremitiy should be elevated frequently to heart level to prevent swelling.

D. Fix a broken cast by applying tape. - A broken cast should be reported to the physician; the patient should not attempt to fix it.

448. A continuous passive motion (CPM) device applied after knee surgery

A. promotes healing by increasing circulation and movement of the knee joint.
A CPM device applied after knee surgery promotes healing by increasing circulation and movement of the knee joint.

B. provides active range of motion. -A CPM device provides passive range of motion.

C. promotes healing by immobilizing the knee joint. - A CPM device applied after knee surgery promotes healing by increasing circulation and movement of the knee joint.

D. prevents infection and controls edema and bleeding. - A CPM device applied after knee surgery promotes healing by increasing circulation and movement of the knee joint.

449. Which of the following terms refers to disease of a nerve root?

A. Radiculopathy- When the patient reports radiating pain down the leg, he is describing radiculopathy.

B. Involucrum- Involucrum refers to new bone growth around the sequestrum.

C. Sequestrum- Sequestrum refers to dead bone in an abscess cavity.

D. Contracture- Contracture refers to abnormal shortening of muscle or fibrosis of joint structures.

450. Of the following common problems of the upper extremities, which results from entrapment of the median nerve at the wrist?

A. Carpal tunnel syndrome- Carpal tunnel syndrome is commonly due to repetitive hand activities.

B. Ganglion- A ganglion, a collection of gelatinous material near the tendon sheaths and joints, appears as a round, firm, cystic swelling, usually on the dorsum of the wrist.

C. Dupuytren's contracture- Dupuytren's contracture is a slowly progressive contracture of the palmar fascia.

D. Impingement syndrome- Impingement syndrome is associated with the shoulder and may progress to a rotator cuff tear.

451. When the nurse notes that the patient's left great toe deviates laterally, she recognizes that the patient has a

A. hallux valgus.-
Hallux valgus is commonly referred to as a bunion.

B. hammertoe.- Hammertoes are usually pulled upward.

C. pes cavus.- Pes cavus refers to a foot with an abnormally high arch and a fixed equinus deformity of the forefoot.

D. flatfoot.- In flatfoot, the patient demonstrates a diminished longitudinal arch of the foot.


452. Localized rapid bone turnover, most commonly affecting the skull, femur, tibia, pelvic bones, and vertebrae, characterizes which of the following bone disorders?

A. Osteitis deformans-
Osteitis deformans (Paget's disease) results in bone that is highly vascularized and structurally weak, predisposing to pathologic fractures.

B. Osteomalacia- Osteomalacia is a metabolic bone disease characterized by inadequate mineralization of bone.

C. Osteoporosis- Osteoporosis is characterized by reduction of total bone mass and a change in bone structure which increases susceptibility to fracture.

D. Osteomyelitis- Osteomyelitis is an infection of bone that comes from extension of soft tissue infection, direct bone contamination, or hematogenous spread.

453. Most cases of osteomyelitis are caused by which of the following microorganisms?

A. Staphylococcus- Staphylococcus aureus causes 70-80% of bone infections.

B. Proteus species- While Proteus species are frequently found in osteomyelitis, they do not cause the majority of bone infections.

C. Pseudomonas species- While Pseudomonas species are frequently found in osteomyelitis, they do not cause the majority of bone infections.

D. Escherichia coli- While E. coli is frequently found in osteomyelitis, it does not cause the majority of bone infections

454.Which of the following statements reflects information to be included when teaching the patient about plantar fasciitis?

A. Management of plantar fasciitis includes stretching exercises.
Management also includes wearing shoes with support and cushioning to relieve pain, orthotic devices (e.g., heel cups, arch supports), and the use of non-steroidal anti-inflammatory drugs (NSAIDs).

B. Plantar fasciitis presents as an acute onset of pain localized to the ball of the foot that occurs when pressure is placed upon it and diminishes when pressure is released.
Plantar fasciitis, an inflammation of the foot-supporting fascia, presents as an acute onset of heel pain experienced with the first steps in the morning. The pain is localized to the anterior medial aspect of the heel and diminishes with gentle stretching of the foot and Achilles tendon.

C. The pain of plantar fasciitis diminishes with warm water soaks. -Plantar fasciitis, an inflammation of the foot-supporting fascia, presents as an acute onset of heel pain experienced with the first steps in the morning. The pain is localized to the anterior medial aspect of the heel and diminishes with gentle stretching of the foot and Achilles tendon.

D. Complications of plantar fasciitis include neuromuscular damage and decreased ankle range of motion.
Unresolved plantar fasciitis may progress to fascial tears at the heel and eventual development of heel spurs.

455. Lifestyle risk factors for osteoporosis include

A. lack of exposure to sunshine.-Lifestyle risk factors for osteoporosis include lack of exposure to sunshine, low calcium and vitamin D diet, cigarette smoking, use of alcohol and/or caffeine, and lack of weight-bearing exercise.

B. lack of aerobic exercise.- Lack of weight-bearing exercise, not aerobic exercise, is a lifestyle risk factor for osteoporosis.

C. a low protein, high fat diet. - A low calcium and vitamin D diet, not a low protein, high fat diet, is a lifestyle risk factor for osteoporosis.

D. an estrogen deficiency or menopause.- An estrogen deficiency or menopause is an individual, not lifestyle risk factor for osteoporosis. Other individual risk factors include female gender, white non-Hispanic or Asian race, increased age, low weight and body mass index, family history of osteoporosis, low initial bone mass, and contributing co-existing medical conditions and medications.

456.The nurse teaches the patient with a high risk for osteoporosis about risk-lowering strategies including which of the following statements?

A. Walk or perform weight-bearing exercises out of doors.
Risk-lowering strategies for osteoporosis include walking or exercising out of doors, performing a regular weight-bearing exercise regimen, increasing dietary calcium and vitamin D intake, smoking cessation, and consuming alcohol and caffeine consumption in moderation.

B. Increase fiber in the diet.- Risk-lowering strategies for osteoporosis include increasing dietary calcium and vitamin D intake,
walking or exercising out of doors, smoking cessation, consuming alcohol and caffeine consumption in moderation, and performing a regular weight-bearing exercise regimen.

C. Reduce stress.- Risk-lowering strategies for osteoporosis include walking or exercising out of doors, increasing dietary calcium and vitamin D intake, smoking cessation, consuming alcohol and caffeine consumption in moderation, and performing a regular weight-bearing exercise regimen.

D. Decrease the intake of vitamin A and D. - Risk-lowering strategies for osteoporosis include increasing dietary calcium and vitamin D intake,
walking or exercising out of doors, smoking cessation, consuming alcohol and caffeine consumption in moderation, and performing a regular weight-bearing exercise regimen.


457.Instructions for the patient with low back pain include which of the following?

A. When lifting, avoid overreaching.- Instructions for the patient with low back pain should include that when lifting, the patient should avoid overreaching. The patient should also keep the load close to the body, bend the knees and tighten the abdominal muscles, use a wide base of support, and use a back brace to protect the back.
B. When lifting, place the load away from the body.- When lifting, the patient with low back pain should keep the load close to the body.

C. When lifting, use a narrow base of support.- When lifting, the patient with low back pain should use a wide base of support.

D. When lifting, bend the knees and loosen the abdominal muscles.- When lifting, the patient with low back pain should bend the knees and tighten the abdominal muscles.

458. Dupuytren's contracture causes flexion of the

A. fourth and fifth fingers. - Dupuytren's contracture causes flexion of the fourth and fifth fingers, and frequently the middle finger.

B. thumb.- Dupuytren's contracture causes flexion of the fourth and fifth fingers, and frequently the middle finger.

C. index and middle fingers.- Dupuytren's contracture causes flexion of the fourth and fifth fingers, and frequently the middle finger.

D. ring finger. - Dupuytren's contracture causes flexion of the fourth and fifth fingers, and frequently the middle finger.

459. A metabolic bone disease characterized by inadequate mineralization of bone is

A. osteomalacia- Osteomalacia is a metabolic bone disease characterized by inadequate mineralization of bone.

B. osteoporosis- Osteoporosis is characterized by reduction of total bone mass and a change in bone structure which increases susceptibility to fracture.

C. osteomyelitis- Osteomyelitis is an infection of bone that comes from extension of soft tissue infection, direct bone contamination, or hematogenous spread.

D. osteoarthritis- Osteoarthritis (OA), also known as degenerative joint disease, is the most common and frequently disabling of the joint disorders. OA affects the articular cartilage, subchondral bone, and synovium.

460. Which of the following terms refers to an injury to ligaments and other soft tissues of a joint?

A. Sprain-
A sprain is caused by a wrenching or twisting motion.

B. Dislocation- Dislocation refers to the separation of joint surfaces.

C. Subluxation-Subluxation refers to partial separation or dislocation of joint surfaces.

D. Strain- Strain refers to a muscle pull or tear.

461. Which of the following terms refers to failure of fragments of a fractured bone to heal together?

A. Nonunion
- When nonunion occurs, the patient complains of persistent discomfort and movement at the fracture site.

B. Dislocation- Dislocation refers to the separation of joint surfaces.

C. Subluxation- Subluxation refers to partial separation or dislocation of joint surfaces.

D. Malunion- Malunion refers to growth of the fragments of a fractured bone in a faulty position, forming an imperfect union.

462. The Emergency Department nurse teaches patients with sports injuries to remember the acronym RICE, which stands for which of the following combinations of treatment?

A. Rest, ice, compression, elevation- RICE is used for the treatment of contusions, sprains, and strains.

B. Rest, ice, circulation, and examination- While circulation problems must be examined, the RICE treatment does not refer to circulation and examination.

C. Rotation, immersion, compression and elevation- Rotation of a joint is contraindicated when injury is suspected, and immersion of the area may be anatomically difficult.

D. Rotation, ice, compression, and examination- Rotation of a joint is contraindicated when injury is suspected, and examination, while indicated, does not provide treatment.

463. The nurse anticipates that the physician will perform joint aspiration and wrapping with compression elastic dressing for which of the following musculoskeletal problems?

A. Joint effusion- The described treatments are used with joint effusions and hemarthrosis.

B. Strain- A strain is treated by RICE.

A. Sprain- A sprain is treated by RICE.

B. Avascular necrosis- Avascular necrosis describes death of tissue due to insufficient blood supply and may be associated with steroid use.

464. When x-ray demonstrates a fracture in which bone has splintered into several pieces, that fracture is described as

A.
comminuted.- A comminuted fracture may require open reduction and internal fixation.

B. compound.- A compound fracture is one in which damage also involves the skin or mucous membranes.

C. depressed.- A depressed fracture is one in which fragments are driven inward.

D. impacted.-An impacted fracture is one in which a bone fragment is driven into another bone fragment.

465. When x-ray demonstrates a fracture in which the fragments of bone are driven inward, the fracture is described as

A. depressed.-
Depressed skull fractures occur as a result of blunt trauma.

B. compound.- A compound fracture is one in which damage also involves the skin or mucous membranes.

C. comminuted.- A comminuted fracture is one in which the bone has splintered into several pieces.

D. impacted.- An impacted fracture is one in which a bone fragment is driven into another bone fragment.

466. A fracture is termed pathologic when the fracture

A. occurs through an area of diseased bone.- Pathologic fractures can occur without the trauma of a fall.

B. results in a pulling away of a fragment of bone by a ligament or tendon and its attachment.- An avulsion fracture results in a pulling away of a fragment of bone by a ligament or tendon and its attachment.

C. presents as one side of the bone being broken and the other side being bent.
A greenstick fracture presents as one side of the bone being broken and the other side being bent.

D. involves damage to the skin or mucous membranes.- A compound fracture involves damage to the skin or mucous membranes.

467. The most common complication after knee arthroscopy is

A. joint effusion.- Joint effusion produces marked pain, and the physician may need to aspirate the joint to remove fluid and relieve the pressure.

B. infection.- Infection is not a common complication of arthroscopy.

C. knee giving way.-Complaints of the knee giving way are associated with functioning of the injured knee prior to arthroscopy.

D. knee locking.- Complaints of the knee locking are associated with functioning of the injured knee prior to arthroscopy.

468. When the patient who has experienced trauma to an extremity complains of severe burning pain, vasomotor changes, and muscles spasms in the injured extremity, the nurse recognizes that the patient is likely demonstrating signs of

A. reflex sympathetic dystrophy syndrome.- RSD is frequently chronic and occurs most often in women.

B. avascular necrosis of bone.- Avascular necrosis is manifested by pain and limited movement.

C. a reaction to an internal fixation device.- Pain and decreased function are the prime indicators of reaction to an internal fixation device.

D. heterotrophic ossification.- Heterotrophic ossification causes muscular pain and limited muscular contraction and movement.

469. Which of the following terms refers to a fracture in which one side of a bone is broken and the other side is bent?

A. Greenstick
- A greenstick fracture is a fracture in which one side of a bone is broken and the other side is bent.

B. Spiral- A spiral fracture is a fracture twisting around the shaft of the bone.

C. Avulsion- An avulsion is the pulling away of a fragment of bone by a ligament or tendon and its attachment.

D. Oblique- An oblique is a fracture occurring at an angle across the bone.

470. The nurse assesses subtle personality changes, restlessness, irritability, and confusion in a patient who has sustained a fracture. The nurse suspects

A. fat embolism syndrome.-
Cerebral disturbances in the patient with fat embolism syndrome include subtle personality changes, restlessness, irritability, and confusion.

B. compartment syndrome.- With compartment syndrome, the patient complains of deep, throbbing, unrelenting pain.

B. hypovolemic shock.- With hypovolemic shock, the patient would have a decreased blood pressure and increased pulse rate.

C. reflex sympathetic dystrophy syndrome. - Clinical manifestations of reflex sympathetic dystrophy syndrome include severe, burning pain, local edema, hyperesthesia, muscle spasms, and vasomotor skin changes.

471. A Colles' fracture is a fracture of the

A. distal radius.- A Colles' fracture is a fracture of the distal radius (wrist). It is usually the result of a fall on an open, dorsiflexed hand.

B. elbow.-A Colles' fracture is a fracture of the distal radius.

C. humeral shaft.- A Colles' fracture is a fracture of the distal radius.

D. clavicle.- A Colles' fracture is a fracture of the distal radius.

472. With fractures of the femoral neck, the leg is

A. shortened, adducted, and externally rotated. - With fractures of the femoral neck, the leg is shortened, adducted, and externally rotated.

B. shortened, abducted, and internally rotated. - With fractures of the femoral neck, the leg is shortened, adducted, and externally rotated.

C. adducted and internally rotated. - With fractures of the femoral neck, the leg is shortened, adducted, and externally rotated.

D. abducted and externally rotated.- With fractures of the femoral neck, the leg is shortened, adducted, and externally rotated.

473. Which of the following terms most precisely refers to an infection acquired in the hospital that was not present or incubating at the time of hospital admission?

A. Nosocomial infection-
A 1970 CDC study found that about one-third of nosocomial infections could be prevented when effective infection control programs were in place.

B. Primary bloodstream infection- A primary bloodstream infection is bacteremia or fungemia, which occurs without infection, identified at another anatomic site.

C. Secondary bloodstream infection- A secondary bloodstream infection is bacteremia of fungemia of another anatomic site, which serves as a source for bloodstream contamination.

D. Emerging infectious diseases- Emerging infectious diseases refer to diseases of infectious origin of which incidence in humans has increased within the past two decades or threaten to increase in the near future.

474.The usual incubation period (infection to first symptom) for AIDS is

A. 10 years.- HIV is transmitted through sexual, percutaneous, or perinatal contact.

B. 3--6 months.- The incubation period for HIV infection is greater than 3-6 months.

C. 1 year.- The incubation period for HIV infection is greater than 1 year.

D. 5 years.- The incubation period for HIV infection is greater than 5 years.

475. The usual incubation period (infection to first symptom) for hepatitis B is

A. 45-160 days.- Hepatitis B is responsible for more than 200 deaths of healthcare workers annually.

B. 15-50 days.-The incubation period for hepatitis B is 45-160 days.

C. 6-9 months.- The incubation period for hepatitis B is shorter than 6-9 months.

D. unclear.- The incubation periods for hepatitis D, E, and G are unclear.

476. Which of the following terms refers to a state of microorganisms being present within a host without causing host interference or interaction?

A. Colonization-
Understanding the principle of colonization facilitates interpretation of microbiologic reports.

B. Susceptible- A susceptible host is a host who does not possess immunity to a particular pathogen.

C. Immune- An immune host is a host who is not susceptible to a particular pathogen.

D. Infection- Infection refers to host interaction with an organism.

477. The nurse teaches the parent of the child with chickenpox that the child is no longer contagious to others when

A. the vesicles and pustules have crusted.-
When the lesions have crusted, the patient is no longer contagious to others.

B. the first rash appears.- The child remains contagious when the rash is present.

C. the fever disappears.- The child remains contagious if the fever occurs as the rash is progressing.

D. the rash is changing into vesicles, and pustules appear.-The child remains contagious when the rash is changing into vesicles and pustules.

478. Which of the following statements reflects the nursing management of the patient with West Nile Virus infection?

A. There is no treatment for West Nile Virus infection. Patients are supported by fluid replacement, airway management, and standard nursing care support during the time that the patient has meningitis symptoms.

B. The incubation period is three to five days. - The incubation period (from mosquito bite until onset of symptoms) is between 5–15 days.

C. Patients with West Nile virus present with gastrointestinal complaints, such as nausea, vomiting, diarrhea, and abdominal pain. - Most human infections are asymptomatic. When symptoms are present, headache and fever are most frequently reported. Less than one percent of those infected develop more severe illness, including meningitis.

D. Transmission of West Nile virus occurs from human-to-human.- Birds are the natural reservoir for the virus. Mosquitoes become infected when feeding on birds and can transmit the virus to animals and humans. There is no human-to-human transmission of virus.

479. Prophylaxis antibiotic for anthrax is given to people with symptoms who have been in a defined "hot zone" for a period of

A. 60 days.-
Those with symptoms who have been in the hot zone should be given 60 days of prophylactic antibiotic. The aim of prophylaxis is to assure that if spores were inhaled, bacteria will be killed immediately upon release from spores. Those who have symptoms of fever, cough, headache, chills, and especially evidence of mediastinal lymph node involvement should be treated with intravenous antibiotics and respiratory support, if needed.

B. 30 days.- Those with symptoms who have been in the hot zone should be given 60 days of prophylactic antibiotic

C. 14 days.-Those with symptoms who have been in the hot zone should be given 60 days of prophylactic antibiotic

D. 10 days. -Those with symptoms who have been in the hot zone should be given 60 days of prophylactic antibiotic

480. If a case of smallpox is suspected, the nurse should

A. call the CDC Emergency Preparedness Office.- Anyone suspecting a case of smallpox should call the CDC Emergency Preparedness Office at 770-488-7100. The CDC will respond by immmediate provision of diagnostic support and eventual release of vaccine if a case is confirmed. Until instructed otherwise by the CDC, healthcare providers should carefully establish isolation with negative pressure, and maintain thorough lists of all those who have contact with the patient.

B. immediately vaccinate the patient and anyone in contact with the patient. -The CDC will provide diagnostic support and will release the vaccine if the patient is confirmed to have smallpox.

C. establish isolation with positive pressure.-Isolation with negative pressure should be established.

D. Assess the patient for signs of a rash similar to chickenpox in appearance and progression. - The lesions associated with smallpox may appear similar in appearance, but the progression is very different from that of chickenpox. Smallpox lesions will appear to be at the same stage of development as the rash progresses from macules to papules to pustules to scabs. This progression is very different from that of chickenpox. With chickenpox, lesions appear at different developmental stages.

481. The six elements necessary for infection are a causative organism, a reservoir of available organisms, a portal or mode of exit from the reservoir, a mode of transmission from reservoir to host, a susceptible host, and a

A. mode of entry to host. -The six elements necessary for infection are a causative organism, a reservoir of available organisms, a portal or mode of exit from the reservoir, a mode of transmission from reservoir to host, a susceptible host, and a mode of entry to host.

B. mode of exit from the host. - A mode of entry to the host, not a mode of exit from the host, is necessary for infection.

C. virulent host. -The six elements necessary for infection are a causative organism, a reservoir of available organisms, a portal or mode of exit from the reservoir, a mode of transmission from reservoir to host, a susceptible host, and a mode of entry to host.

D. latent time period. -The six elements necessary for infection are a causative organism, a reservoir of available organisms, a portal or mode of exit from the reservoir, a mode of transmission from reservoir to host, a susceptible host, and a mode of entry to host.

482. Which of the following statements reflect what is known about the Ebola and Marburg viruses?

A. The diagnosis should be considered in a patient who has a febrile, hemorrhagic illness after traveling to Asia or Africa. - The diagnosis should be considered in a patient who has a febrile, hemorrhagic illness after traveling to Asia or Africa, or who has handled animals or animal carcasses from those parts of the world.

B. Treatment during the acute phase includes administration of acyclovir, and ventilator and dialysis support.- No antivirals have been approved or show promise against the viruses. Treatment must be largely supportive maintenance of the circulatory system and respiratory systems. It is likely that the infected patient would need ventilator and dialysis support through the acute phases of illness.

C. The viruses can be spread only by airborne exposure. - The viruses can be spread by exposure to blood or other body fluid, insect bite, and mucous membrane exposure.

D. Symptoms include severe lower abdominal pain, nausea, vomiting, and dehydration. - Symptoms include fever, rash, and encephalitis which progress rapidly to profound hemorrhage, organ destruction, and shock.

483. Bubonic plague occurs

A. after the organism enters through the skin.- Bubonic refers to enlarged lymph nodes that develop after the organism enters through the skin. Bubonic plague is the form seen most frequently, as the organism is transferred from rodents or other animals to humans by insect bite.

B. occurs after the organism is inhaled.. - Pneumonic plague occurs after the organism is inhaled. Only pneumonic plague can be contagious from person to person by an airborne route.

C. occurs when the organism causes a bloodstream infection. - Septicemic plague occurs when the organism causes a bloodstream infection usually secondary to either pneumonic or bubonic, but sometimes without either entity.

D. after the organism is transferred by human to human contact. - Bubonic plague is the form seen most frequently, as the organism is transferred from rodents or other animals to humans by insect bite.

484. The term given to the category of triage that refers to life-threatening or potentially life-threatening injury or illness requiring immediate treatment is

A. emergent.
- The patient triaged as emergent must be seen immediately.

B. urgent.- The triage category of urgent refers to minor illness or injury needing first-aid-level treatment.

C. immediate.-The triage category of immediate refers to non-acute, non-life-threatening injury or illness.

D. non-acute.- The triage category of immediate refers to non-acute, non-life-threatening injury or illness.

485. When the patient has been field triaged and categorized as blue, the nurse recognizes that the patient requires

A. fast-track or psychological support.-
When a patient is categorized as blue, field triage has identified fast-track or psychological support needs.

B. emergent care.- Field triaged patients who require emergent care will be categorized as red.

C. immediate care.-Field triaged patients who require immediate care will be categorized as yellow.

D. urgent care.-Field triaged patients who require urgent care will be categorized as green.

486. Which of the following guidelines is appropriate to helping family members cope with sudden death?

A. Show acceptance of the body by touching it, giving the family permission to touch.
The nurse should encourage the family to view and touch the body if they wish, since this action helps the family to integrate the loss.

B. Inform the family that the patient has passed on.- The nurse should avoid using euphemisms such as passed on.

C. Obtain orders for sedation for family members.-The nurse should avoid giving sedation to family members, since this may mask or delay the grieving process.

D. Provide details of the factors attendant to the sudden death. -The nurse should avoid volunteering unnecessary information (e.g., patient was drinking at the time of the accident).


487. Which of the following solutions should the nurse anticipate for fluid replacement in the male patient?

A. Lactated Ringer's solution- Replacement fluids may include isotonic electrolyte solutions and blood component therapy.

B. Type O negative blood- O negative blood is prepared for emergency use in women of childbearing age.

C. Dextrose 5% in water- Dextrose 5% in water should not be used to replace fluids in hypovolemic patients.

D. Hypertonic saline- Hypertonic saline is used only to treat severe symptomatic hyponatremia and should be used only in intensive care units.

488. Induction of vomiting is indicated for the accidental poisoning patient who has ingested

A. aspirin.- Overdose of aspirin should be treated with emesis or lavage, followed by ingestion of activated charcoal to absorb the aspirin.

B. rust remover.- Rust remover is an alkaline product, which is corrosive, and induced vomiting is contraindicated.

C. gasoline.- Gasoline is a petroleum distillate, and induced vomiting is contraindicated.

D. toilet bowl cleaner.- Toilet bowl cleaners are corrosive, and induced vomiting is contraindicated.

489. Which of the following phases of psychological reaction to rape is characterized by fear and flashbacks?

A. Heightening anxiety phase-
During the heightened anxiety phase, the patient demonstrates anxiety, hyperalertness, and psychosomatic reactions, in addition to fear and flashbacks.

B. Acute disorganization phase- The acute disorganization phase is characterized by shock, disbelief, guilt, humiliation, and anger.

C. Denial phase- The denial phase is characterized by an unwillingness to talk.

D. Reorganization phase- The reorganization phase occurs when the incident is put into perspective. Some patients never fully recover from rape trauma.

490. When preparing for an emergency bioterroism drill, the nurse instructs the drill volunteers that each biological agents requires specific patient management and medications to combat the virus, bacteria, or toxin. Which of the following statements reflect the patient management of variola virus (small pox)?

A. Small pox spreads rapidly and requires immediate isolation. -
Small pox is spread by droplet or direct contact. There are no antiviral agents effective against small pox, however vaccination within two to three days of exposure is protective.

B. Acyclovir is effective against smallpox. - There are no antiviral agents effective against small pox; however, vaccination within two to three days of exposure is protective.

C. Small pox is spread by inhalation of spores.- Small pox is spread by droplet or direct contact. It spreads rapidly and requires immediate isolation. Even in death, the disease can be transmitted.

D. Vaccination is effective only if administered within 12 to 24 hours of exposure.
Vaccination within two to three days of exposure of the small pox virus is protective. In four to five days, it may prevent death and should be administered with vaccinia immune globulin.

491. Which of the following statements reflect the nursing management of pulmonary anthrax (B. anthracis)?

A. Prophylaxis with fluoroquinone is suggested after exposure. -Treatment is with ciprofloxacin or doxycycline.

Airborne person-to-person transmission occurs.
Anthracis is a spore forming bacteria resulting in gastrointestinal, pulmonary, and skin symptoms. Symptoms are dependent upon contact, ingestion, or inhalation of the spores. Routine universal precautions are effective. Anthrax survives in the spore form for long periods making the body a potential source of infection for morticians.

B. Diagnosis is by pulmonary function testing and chest x-ray. - Blood cultures are required to confirm the bacteria's presence and diagnosis.

C. Pulmonary effects include respiratory failure, shock, and death within five to seven days after exposure. - The pulmonary effects include respiratory failure, shock, and death within 24-36 hours after exposure.

492. Which of the following terms refers to injuries that occur when a person is caught between objects, run over by a moving vehicle, or compressed by machinery?

A. Crush injuries
- Crush injuries are those that occur when a person is caught between objects, run over by a moving vehicle, or compressed by machinery.

B. Blunt trauma- Blunt trauma is commonly associated with extra-abdominal injuries to the chest, head, or extremities.

C. Penetrating abdominal injuries- Penetrating abdominal injuries include those such as gunshot wounds and stab wounds.

D. Intra-abdominal injuries- Intra-abdominal injuries are categorized as penetrating and blunt trauma.

493. A person suffering from carbon monoxide poisoning

A. appears intoxicated. - A person suffering from carbon monoxide poisoning appears intoxicated (from cerebral hypoxia). Other signs and symptoms include headache, muscular weakness, palpitation, dizziness, and mental confusion.

B. presents with severe hypertension. - A person suffering from carbon monoxide poisoning appears intoxicated (from cerebral hypoxia). Other signs and symptoms include headache, muscular weakness, palpitation, dizziness, and mental confusion.

C. appears hyperactive. - A person suffering from carbon monoxide poisoning appears intoxicated (from cerebral hypoxia). Other signs and symptoms include headache, muscular weakness, palpitation, dizziness, and mental confusion.

D. will always present with a cherry red skin coloring. - The skin coloring in the patient with carbon monoxide poisoning can range from pink to cherry red to cynanotic and pale and is not a reliable diagnostic sign.

494.Treatment of an acetaminophen overdose includes the administration of

A. N-acetylcysteine (Mucomyst). - Treatment of acetaminophen overdose includes administration of N-acetylcysteine (Mucomyst).

B. flumazenil (Romazicon). - Flumazenil is administered in the treatment of nonbarbituate sedative overdoses.

C.naloxone (Narcan)- Naloxone (Narcan) is administered in the treatment of narcotic overdoses.

D. diazepam (Valium). - Diazepam (Valium) may be administered to treat uncontrolled hyperactivity in the patient with a hallucinogen overdose.


495. Which of the following statements reflect the nursing management of the patient with a white phosphorus chemical burn?

A. Do not apply water to the burn. - Water should not be applied to burns from lye or white phosphorus because of the potential for an explosion or deepening of the burn.

B. Immediately drench the skin with running water from a shower, hose or faucet.
Water should not be applied to burns from lye or white phosphorus because of the potential for an explosion or deepening of the burn.

C. Alternate applications of water and ice to the burn. - Water should not be applied to burns from lye or white phosphorus because of the potential for an explosion or deepening of the burn.

D. Wash off the chemical using warm water, then flush the skin with cool water. - Water should not be applied to burns from lye or white phosphorus because of the potential for an explosion or deepening of the burn.

496. During a disaster, the nurse sees a victim with a green triage tag. The nurse knows that the person has

A. injuries that are minor and treatment can be delayed hours to days. -
A green triage tag (priority 3 or minimal) indicates injuries that are minor and treatment can be delayed hours to days.

B. injuries that are life-threatening but survivable with minimal intervention. - A red triage tag (priority 1 or immediate) indicates injuries that are life-threatening but survivable with minimal intervention.

C. injuries that are significant and require medical care, but can wait hours without threat to life or limb. - A yellow triage tag (priority 2 or delayed) indicates injuries that are significant and require medical care, but can wait hours without threat to life or limb.

D. indicates injuries that are extensive and chances of survival are unlikely even with definitive care.- A black triage tag (priority 4 or expectant) indicates injuries that are extensive and chances of survival are unlikely even with definitive care.

497. If a person has been exposed to radiation, presenting symptoms, such as nausea, vomiting, loss of appetite, diarrhea, or fatigue can be expected to occur within _______ hours after exposure?

A. 48 to 72 -
The prodromal phase (presenting symptoms) of radiation exposure occurs within 48 to 72 hours after exposure. Signs and symptoms include nausea, vomiting, loss of appetite, diarrhea, and fatigue. With high-dose radiation exposure, the signs and symptoms may include fever, respiratory distress, and increased excitability.

B. 6 to 12- The prodromal phase (presenting symptoms) of radiation exposure occurs within 48 to 72 hours after exposure.

C. 12 to 24 - The prodromal phase (presenting symptoms) of radiation exposure occurs within 48 to 72 hours after exposure.

D. 24 to 48 - The prodromal phase (presenting symptoms) of radiation exposure occurs within 48 to 72 hours after exposure.

498. Which of the following refers to a management tool for organizing personnel, facilities, equipment, and communication for any emergency situation?

A. The Incident Command System .The Incident Command System (ICS) is a management tool for organizing personnel, facilities, equipment and communication for any emergency situation. The federal government mandates that the ICS be used during emergencies. Under this structure, one person is designated as incident commander. This person must be continuously informed of all activities and informed about any deviation from the established plan. While the ICS is primarily a field structure and process, aspects of it are used at the level of an individual hospital’s emergency response plan as well.

B. at the state and local levels. The OEM is responsible for providing interagency coordination during an emergency. It maintains a corps of emergency management personnel, including responders, planners, and administrative and support staff.

C. National Disaster Medical System -National Disaster Medical System (NDMS). The NDMS has many medical support teams such as Disaster Medical Assistance Teams (DMATs) that provide medical personnel to set up and staff a field hospital.

D. The Hospital Emergency Preparedness Plan- The Hospital Emergency Preparedness Plan is a facility-specific plan for emergency preparedness required by the Joint Commission on Accreditation of Healthcare Organizations (JCAHO).

499. Which of the following terms refers to a process by which an individual receives education about recognition of stress reactions and management strategies for handling stress?

A. Defusing-
Defusing is a process by which the individual receives education about recognition of stress reactions and management strategies for handling stress. It is a component of critical incident stress management (CISM).

B. Debriefing -Debriefing is a more complicated intervention of critical incident stress management (CISM); it involves 2- to 3- hour process during which participants are asked about their emotional reactions to the incident, what symptoms they may be experiencing (e.g., flashbacks, difficulty sleeping, intrusive thoughts) and other psychological ramifications.

C. Follow-up-In follow-up, members of the critical incident stress management (CISM) team contact the participants of a debriefing and schedule a follow-up meeting if necessary. People with ongoing stress reactions are referred to mental health specialists.

D. Critical incident stress management - Critical incident stress management (CISM) is an approach to preventing and treating the emotional trauma that can affect emergency responders as a consequence of their jobs but that can also occur to anyone involved in a disaster or mass casualty incident.

500. The first step in decontamination is

A. removal of the patient's clothing and jewelry and then rinsing the patient with water. - To be effective, decontamination must include a minimum of two steps. The first step is removal of the patient’s clothing and jewelry and then rinsing the patient with water. The second step consists of a thorough soap and water wash and rinse.

B. a thorough soap and water wash and rinse of the patient. - A thorough soap and water wash and rinse of the patient is the second step in the decontamination process. The first is to remove the patient's clothing and jewelry and then rinsing the patient with water.

C. to immediately apply personal protective equipment.- To be effective, decontamination must include a minimum of two steps. The first step is removal of the patient’s clothing and jewelry and then rinsing the patient with water. The second step consists of a thorough soap and water wash and rinse.

D. to immediately apply a chemical decontamination foam to the area of contamination. - To be effective, decontamination must include a minimum of two steps. The first step is removal of the patient’s clothing and jewelry and then rinsing the patient with water. The second step consists of a thorough soap and water wash and rinse.

No comments:

Post a Comment

Categories

Amoebiasis (1) Anatomy and Physiology (42) ANATOMY AND PHYSIOLOGY Quick Review (1) ANATOMY AND PHYSIOLOGY Quick Review quiz (1) and Acid-Base Balance (3) and Dying (2) Anesthetics (2) Answers (13) antibiotics (2) antifungal (1) antiparasitics (1) Antiviral (1) Ascariasis (1) Asepsis (1) audio (2) audiobook (1) Basic Drill Answers (1) Basic Intravenous Therapy Lectures (1) Body systems (1) Bullets (1) Cancer (5) Cardiac Drugs (1) Cardiovascular (1) Cardiovascular Diseases (1) CBQ answers (1) CD A (2) CD A to Z (1) CD_A (3) CHN practice test (7) CHN practice test answers (7) Circulatory System (1) Common Board Questions (1) Common Lab Values (1) Common Laboratory tests (11) Communicable Disease Nursing (5) COMMUNICABLE DISEASES (6) Community Health Nursing (1) Comunication in Nursing (1) concepts (1) COPD (1) Coping mechanisms (1) CPR (4) Degenerative Disorders (2) Diabetes Mellitus (1) Diagnostic Procedure and tests (1) Diet (7) digestive system (1) Disorders (13) documentation and reporting (1) downloads (6) ebooks (3) Electrolyte (3) Emergency drugs (1) endocrine disorders (3) endocrine drugs (1) endocrine system (9) Endorcrine drugs (5) Family Planning (1) Fluid (3) Fluids and Electrolytes (36) FUNDAMENTALS OF NURSING (71) Gastrointestinal System (3) Git Bullets (1) GIT Disorders (5) GIT drugs (7) Grief (2) GUT (1) GUT drugs (3) handouts (1) Hematological drugs (3) Homeostasis (1) IMCI (1) immune sytem (1) increased intracranial pressure (1) Integumentary drugs (5) IV Therapy Lectures (4) Loss (2) LPN (2) LPN/LVN NCLEX (2) LRS Disorders: Infectious (4) LRS Disorders: Miscellaneous (5) Lung Cancer (4) LVN (2) maternal drill answers (7) Maternal Nursing (35) MCN (28) Medical and Surgical Nursing (61) Medical and Surgical Nursing Overview (1) Medical and Surgical Nursing Quiz (1) medications (1) MedSurg (8) MS drill answers (8) MS Drills (8) MS handouts (17) Muscular System (1) NCLEX hot topics (1) NCLEXPN (2) nervous system (1) Neuro Drugs (11) neurology (1) Neurology Anatomy and Physiology (1) NLE Practice Test (53) notes (1) NURSING (4) Nursing Bullets (3) Nursing Jurisprudence (1) Nursing Leadership and Management (1) Nursing Lectures (1) Nursing Process (1) Nursing Research (1) Nursing Research drill (1) Nursing Research drill answer (1) Nursing Slideshows (12) NURSING VIDEOS (1) Nutrition (8) Obstetric Nursing (6) OR (1) Orthopedic (1) Pain (1) Pain assessment (1) PALMER (2) Parkinson's disease (1) Pediatric Drills answers (10) Pediatrics Nursing (14) pentagon notes (2) Pericarditis (1) PHARMACOLOGY (75) Physical Assessment (11) Practice Tests (50) PRC (1) Psychiatric Nursing (18) Psychiatric Nursing Answers (7) Psychiatric Nursing Drills (7) Quizzes (5) Respiratory Disease (21) Respiratory Drugs (7) Respiratory System (3) Schizophrenia (1) self concept (1) skeletal system (1) Sleep (1) slideshow (13) stress (3) subjects (1) Surgery (1) Terms to know (1) Therapeutic Communication (1) Transcultural concepts quick review (1) Urinary System (1) video (13) Vital Signs (1)

share this blog

Share |